203996327-Conclusion-Es.pdf

May 9, 2018 | Author: Rob Sandoval Steban | Category: Epilepsy, Multiple Sclerosis, Coma, Neurology, Hypoglycemia


Comments



Description

NEUROLOGÍA1ª Vuelta CTO Medicina SEMIOLOGÍA. 7. Paciente con disminución de fuerza en miembro inferior derecho, con disminución de sensibilidad térmica en 1. Un paciente con lesión en el área motora puede presentar miembro inferior izquierdo y déficit de sensibilidad posi- todas, EXCEPTO una: cional en miembro inferior derecho. Se trata de: 1) Ausencia de reflejos miotáticos, en fases muy precoces. 1) Síndrome medular anterior. 2) Reflejos miotáticos exaltados. 2) Síndrome medular posterolateral. 3) Ausencia de reflejos cutáneo abdominales. 3) Síndrome medular central. 4) Respuesta cutáneo plantar flexora en fases avanzadas 4) Síndrome de hemisección medular derecha. del cuadro. 5) Síndrome de hemisección medular izquierda. 5) Disminución de la masa muscular del territorio afecta- do. 8. La pérdida de sensibilidad térmica y dolorosa bilateral entre los niveles D3 y D6, con sensibilidad conservada por 2. Un paciente presenta hemiparesia derecha que afecta a encima y debajo de la lesión, nos debe hacer sospechar el cara, brazo y pierna. Señale, de entre las siguientes opcio- diagnóstico de: nes, la localización más probable de la lesión: 1) Polineuropatía de fibras finas. 1) Hemimédula derecha. 2) Polirradiculopatía. 2) Bulbo medial derecho. 3) Lesión centromedular. 3) Protuberancia izquierda. 4) Lesión cordonal anterior. 4) Cápsula interna derecha. 5) Lesión cordonal posterior. 5) Cápsula interna izquierda. 9. Un varón de 60 años refiere, desde hace seis meses, déficit 3. La exploración de un paciente que consulta por diplopía motor progresivo en miembros inferiores, acompañado revela limitación en la mirada hacia fuera del ojo izquier- de sensación eléctrica cuando flexiona el cuello, que se do, paresia hemifacial izquierda y disminución de fuerza inicia a nivel cervical y se irradia distalmente. A la explo- en extremidades derechas. La lesión se sitúa en: ración se objetivan amiotrofias en musculatura intrínseca de las manos, reflejos bicipital y estilorradial normales, 1) Cápsula interna izquierda. hiperreflexia en miembros inferiores y respuesta cutáneo 2) Mesencéfalo izquierdo. plantar extensora bilateral. ¿Cuál de los siguientes diag- 3) Mesencéfalo derecho. nósticos le parece más probable?: 4) Protuberancia izquierda. 5) Conducto auditivo interno izquierdo. 1) Mielopatía cervical. 2) Neurolúes. 4. Paciente con hipoestesia hemifacial izquierda y hemicor- 3) Degeneración subaguda combinada de la médula. poral derecha. Señale la localización más probable de su 4) Síndrome espinal anterior. lesión: 5) Esclerosis lateral amiotrófica. 1) Corteza parietal izquierda. 10. Señale la definición correcta: 2) Tálamo izquierdo. 3) Mesencéfalo izquierdo. 1) Distonías: movimientos continuos, reptantes, que pue- 4) Bulbo izquierdo. den afectar a cualquier área corporal y que suelen 5) Ganglio de Gasser izquierdo. aumentar si el paciente intenta realizar un movimiento voluntario o hablar. 5. Varón de 63 años, hipertenso y diabético acude a Urgen- 2) Mioclonías: movimientos repetitivos, irregulares y este- cias por diplopía de instauración brusca, que desaparece reotipados que pueden afectar a diversos grupos mus- al taparse uno de los dos ojos. La exploración revela ptosis, culares. Preguntas TEST limitación en la supraelevación e infraversión de ojo 3) Corea: movimientos involuntarios de torsión rela- derecho, con pupilas normales. El diagnóstico más proba- cionados con contracciones musculares manteni- ble es: das. 4) Temblor: movimiento oscilatorio rítmico debido a 1) Herniación uncal derecha, secundaria a una hemorra- contracciones de tipo alternante de músculos agonistas gia intraparenquimatosa. y antagonistas. 2) Neuropatía isquémica del tercer par. 5) Atetosis: movimientos rápidos de amplitud variable 3) Aneurisma de la arteria comunicante posterior. debidos a contracciones simultáneas de músculos 4) Aneurisma de la arteria cerebelosa anterosuperior. agonistas y antagonistas. 5) Aneurisma de la arteria cerebral posterior. 11. Varón de 72 años que presenta a la exploración tono 6. Un paciente acude a su médico por haber observado muscular aumentado de forma generalizada, con resis- descenso de su párpado derecho con disminución de la tencia continua a la movilización pasiva y reflejos miotá- hendidura palpebral. La exploración revela una pupila ticos normales. De todas las siguientes, señale cuál le derecha miótica. En el diagnóstico diferencial incluirá parece la definición correcta de la alteración motora que todos, EXCEPTO uno de los siguientes: presenta el paciente: 1) Síndrome bulbar lateral o de Wallenberg. 1) Discinesia. 2) Siringomielia cervical. 2) Distonía. 3) Tumor apical pulmonar. 3) Rigidez. 4) Patología carotídea. 4) Espasticidad. 5) Neurolúes. 5) Paresia. 1 NEUROLOGÍA CTO Medicina 1ª Vuelta 12. Señale cuál es el tratamiento de elección en las distonías 18. Señale cuál de los siguientes síntomas NO corresponde a focales cérvico-faciales: una lesión de la corteza occipital: 1) Anticolinérgicos. 1) Prosopagnosia. 2) Benzodiacepinas. 2) Astereognosia. 3) Toxina botulínica. 3) Simultanagnosia. 4) Barbitúricos. 4) Hemianopsia homónima con respeto macular. 5) Tenotomía. 5) Apraxia óptica. 13. Un paciente de 57 años presenta oscilaciones en las manos 19. Señale dónde localizaría la lesión ante un paciente con cuando realiza actividades como intentar enhebrar una cuadrantanopsia homónima superior izquierda: aguja o sujetar un vaso mientras bebe. No presenta movi- mientos involuntarios cuando no realiza actividad con las 1) Lóbulo temporal izquierdo. manos. Señale cuál de estos términos le parece más 2) Lóbulo temporal derecho. correcto para definir este trastorno: 3) Lóbulo parietal izquierdo. 4) Lóbulo parietal derecho. 1) Temblor de reposo. 5) Cintilla óptica izquierda. 2) Temblor postural. 3) Temblor cinético. 20. ¿Cuál de los siguientes datos clínicos NO es característico 4) Temblor intencional. de la lesión del lóbulo parietal?: 5) Temblor rúbrico. 1) No reconocimiento de objetos a través del tacto (aste- 14. Un movimiento involuntario, caracterizado por sacudi- reognosia). das rápidas y arrítmicas secundarias a periodos silentes en 2) Negligencia del hemicuerpo contralateral en lesiones la actividad electromiográfica, se puede encontrar asocia- no dominantes. do a cualquiera de estas encefalopatías, SALVO: 3) No discriminación entre dos estímulos cutáneos en hemicuerpo contralateral. 1) Hipoglucémica. 4) Anestesia dolorosa en el hemicuerpo contralateral. 2) Hipercápnica. 5) Cuadrantanopsia homónima inferior contralateral. 3) Hepática. 4) Urémica. 21. Una mujer de 72 años, hipertensa, presenta bruscamente 5) Tóxica por anticomiciales. un trastorno del lenguaje, con habla incoherente, sustitu- yendo una palabra por otra, siendo incapaz de repetir 15. Paciente de 55 años que acude a su consulta porque en los palabras que se le dicen. Sin embargo, obedece órdenes últimos meses refiere un “temblorcillo” en la mano iz- verbales. La localización más probable de la lesión es: quierda cuando tiene la mano apoyada; si realiza cual- quier movimiento, este temblor desaparece. ¿Cuál sería su 1) Lóbulo frontal izquierdo. actitud?: 2) Girus supramarginalis izquierdo. 3) Lóbulo temporal izquierdo. 1) Por tratarse de un temblor postural, insistiría sobre sus 4) Área prefrontal izquierda. antecedentes familiares. 5) En torno al área de Wernicke. 2) Realizaría urgentemente una prueba de imagen para descartar una lesión cerebelosa hemisférica. 3) Pensaría que probablemente en los próximos años CUADROS CONFUSIONALES. ENCEFALOPATÍAS META- desarrollará una enfermedad de Parkinson. BÓLICAS. COMA. 4) Lo tranquilizaría, ya que probablemente sean “sus nervios”. 22. Un paciente de 70 años es traído a urgencias por sus 5) Buscaría signos que apunten a una ingesta alcohólica Preguntas TEST familiares por presentar un cuadro de instauración pro- importante. gresiva en el curso de varias horas consistente en inaten- ción, desorientación temporoespacial, incapacidad para 16. De entre los siguientes síntomas y signos, señale cuál NO retener información, lenguaje incoherente y alucinacio- corresponde a una lesión cerebelosa: nes. Señale la etiología más probable de este proceso: 1) Disartria. 1) Hematoma subdural subagudo. 2) Temblor intencional. 2) Tumoración hemisférica no dominante. 3) Temblor cinético. 3) Estatus parcial complejo. 4) Marcha atáxica. 4) Encefalitis herpética. 5) Hipertonía. 5) Trastorno metabólico. 17. Paciente que presenta de forma progresiva bipedestación y 23. NO se considera causa de estado comatoso: marcha inestable, con pasos desiguales, sin clara lateraliza- ción. No presenta dismetrías, disdiadococinesias ni oscila- 1) Encefalopatía urémica. ciones en extremidades. Señale el diagnóstico más probable: 2) Encefalopatía hepática. 3) Lesión compresiva mesencefálica. 1) Temblor esencial. 4) Lesión vascular protuberancial extensa. 2) Lesión ocupante de espacio en fosa posterior. 5) Lesión bulbar. 3) Atrofia aislada de vermis cerebeloso. 4) Lesión cordonal posterior. 24. La exploración de un paciente en coma revela una postura 5) Lesión mesencefálica. en extensión y adducción de miembros inferiores, con 2 NEUROLOGÍA 1ª Vuelta CTO Medicina extensión, adducción y rotación interna de miembros 3) Afectar de forma precoz el córtex cerebeloso, los superiores, que aparece con la estimulación dolorosa. ganglios basales y el área hipocampal. ¿Cuál de los siguientes hallazgos es más compatible con la 4) Causar daños tisulares cerebrales en 5 minutos tras el situación clínica descrita?: inicio de la anoxia. 5) Aparición de síntomas días después de sufrir el proceso 1) Patrón respiratorio rítmico con pausas de apnea y isquémico. rachas de hiperventilación. 2) Reflejos oculocefálicos conservados. 30. Paciente de 18 años que es traído a urgencias tras un 3) Pupilas de tamaño y reactividad normales. cuadro de cinco días de evolución de trastorno de la 4) Hematoma en fosa posterior. conducta y pérdida de memoria, en el que en el curso de 5) Respuesta a órdenes verbales. las últimas 48 horas se ha objetivado fiebre de 39,5 ºC, cefalea holocraneal, y disminución del nivel de concien- 25. Un paciente en coma presenta desviación tónica de la cia. El examen del LCR revela pleocitosis linfocitaria con un mirada hacia la izquierda y disminución de la movilidad en nivel de proteinas elevado. Respecto al diagnóstico y hemicuerpo izquierdo ante estímulos dolorosos. Como tratamiento, señale la FALSA: causa de su coma sospechará: 1) Ante la sospecha clínica, incluso sin estudio del LCR, 1) Lesión hemisférica izquierda. estaría indicado el tratamiento empírico con aciclovir. 2) Lesión protuberancial derecha. 2) Se trata de la causa más frecuente de encefalitis focal 3) Lesión bulbar derecha. esporádica. 4) Hipoglucemia severa. 3) En la TC o RMN podemos encontrar datos de inflama- 5) Encefalopatía hipóxica. ción y edema sobre todo en los lóbulos temporal anterior y frontal inferior. 26. Un paciente alcohólico de 49 años, con antecedentes de 4) El método más sensible y específico para el diagnóstico HTA, diabetes y fumador importante, es traído al servicio es el cultivo viral del líquido cefalorraquídeo. de Urgencias por alteración severa del nivel de conciencia. 5) Los pacientes inmunodeprimidos pueden tener una En la exploración clínica inicial se objetiva parálisis pseu- respuesta inflamatoria reducida en el cerebro, con un dobulbar y cuadriparesia espástica. ¿Cuál es el diagnósti- curso clínico insidioso. co más probable?: 31. ¿Cuál de los siguientes datos NO es característico del coma 1) Enfermedad de Wernicke. hipoglucémico?: 2) Enfermedad de Marchiafava-Bignami. 3) Mielinólisis central pontina. 1) Aparición de los primeros síntomas al cabo de minutos 4) Psicosis de Korsakoff. de hipoglucemia severa mantenida. 5) Polineuritis alcohólica. 2) La ingesta de antidiabéticos orales puede ser causa de coma hipoglucémico. 27. Un paciente es traído al hospital en situación clínica de 3) Puede cursar con hemiparesia, especialmente en an- coma. A la exploración se le objetiva patrón respiratorio de cianos. hiperventilación rítmica y pupilas isocóricas de tamaño 4) Se presenta como una desconexión brusca del medio, medio y reactividad normal. El diagnóstico más probable es: sin síntomas previos. 5) La sobredosis de insulina es una causa frecuente. 1) Lesión hemisférica profunda. 2) Lesión mesencefálica. 32. Un paciente es traído a Urgencias por presentar diplopía, 3) Lesión pontina. importante inestabilidad en la marcha y disminución del nivel 4) Lesión bulbar. de conciencia con lenguaje incoherente. Entre sus antece- 5) Coma acidótico. dentes destaca alcoholismo. De las siguientes opciones, ¿cuál le parece la medida terapéutica inicial más adecuada?: Preguntas TEST 28. Usted está valorando a un paciente que ingresó hace 24 horas por un cuadro confusional con lenguaje incoheren- 1) Observación clínica sin tratamiento. te y ataxia. A la exploración se le objetiva estado alerta con 2) Punción lumbar y tratamiento con aciclovir empírico. incapacidad para realizar cualquier movimiento volunta- 3) Administración de vitamina B1 parenteral. rio, salvo parpadeo débil y movimientos verticales ocula- 4) Sueroterapia con sueros glucosados abundantes. res. Su opinión sería: 5) Administración de neurolépticos parenterales. 1) Debe realizarse TC craneal para descartar lesión di- encefálica ocupante de espacio. AMNESIAS. DEMENCIAS. 2) Debe realizarse RM cerebral centrada en fosa posterior por ser ésta la prueba de imagen de elección. 33. Sobre los defectos de memoria, señale la opción FALSA: 3) Debe tratársele de forma inmediata con vitamina B1 parenteral. 1) La memoria inmediata depende fundamentalmente 4) Deben revisarse los hallazgos analíticos al ingreso y la del nivel de atención. sueroterapia pautada. 2) La amnesia global transitoria afecta básicamente al 5) La causa más probable es coma de origen conversivo. recuerdo de datos recientes. 3) El tratamiento de los cuadros de amnesia se basa en el 29. La encefalopatía anoxicoisquémica NO se caracteriza bloqueo de los receptores gabaérgicos. por: 4) La amnesia anterógrada es uno de los hallazgos carac- terísticos del síndrome de Korsakoff. 1) Buena regeneración posterior de las áreas afectadas. 5) Una lesión bilateral extensa de lóbulos temporales 2) Su origen puede ser un infarto agudo de miocardio. puede cursar con un síndrome amnésico. 3 resulta muy que más le atraían. Señale. 1) Crisis generalizada no convulsiva. 2) TAC craneal. excepto lenguaje 3) Existen alteraciones en la marcha. trastorno de la marcha y sacudidas tando continuamente "¿dónde estoy?". No déficits neurológicos. EPILEPSIA. 1) Enfermedad de Pick. 4 . El deterioro de funciones superiores caracterizado por número de casos. 45. 5) Afasias. 2) Enfermedad de Creutzfeldt-Jakob. minutos. La diferencia entre crisis parciales simples y complejas es: 37. por tratarse de una 1) La mitad de los pacientes afectados muestran signos de crisis parcial compleja. 44. afectación de primera motoneurona. Señale cuál de las siguientes enfermedades cursa con 4) Crisis de ausencia. 3) Enfermedad de Alzheimer. un fenómeno isquémico cerebral. pregun. ¿Cuál de las 4) Cursa con demencia subcortical. 5) Pueden existir antecedentes familiares en un reducido 35. Sobre la enfermedad de Binswanger. señale la FALSA: 36. ni trastorno de pequeños pasos”. operado hace un año. es carac- terístico de todas. 42. Un paciente de 25 años ha comenzado a presentar episo- dios repetidos de pérdida de conciencia consistentes en 38. ni de los esfínteres aparentemente. Paciente de 65 años consulta por un cuadro progresivo de brusca presenta pérdida de la memoria reciente. Se sigue de corta confu- 1) Deterioro del estado de alerta y atención. Un paciente ha sido diagnosticado de demencia cortical. Un paciente de 70 años. deterioro de memoria. que aunque no es patognomónica. 2) 3) Inhibidores de la acetilcolinesterasa cerebral. cos involuntarios que comienzan en la mano derecha y 5) Enfermedad de Parkinson. Acude a urgencias una paciente de 65 años que de manera 40. con antecedente de meningioma 2) Enfermedad de Huntington. 2) El deterioro severo se produce en la mayoría de los 3) Pediría una RM para descartar una encefalitis herpética pacientes antes de un año desde el inicio de los temporal. 2) Enfermedad de Huntington. agnosias o afasia. 4) Recomendaría una valoración psiquiátrica. 3) Conservación de la conciencia y percepción del am- 1) Agonistas de receptores dopaminérgicos. como hiperrefle- 2) Tranquilizaría a la familia. bresaltos. No datos de parkinsonismo. enfermedad de Alzheimer?: 2) Duración de la crisis. sin factores de riesgo cardiovascular. sensación epigástrica ascendente seguida de desconexión Usted NO esperaría encontrar en la exploración: del medio. de olvidos frecuentes e incapacidad para retener las 2) Presenta leucoaraiosis en las pruebas de imagen cere- cosas como antes. la marcha. 5) Serología luética. con 41. menos de 6 horas el cuadro remita sin ningún problema. En la exploración musculares breves y arrítmicas desencadenadas por so- no se objetiva ningún signo de focalidad neurológica. Se trata probablemente de: 3) Apraxias. con una marcha “a empobrecido. Preguntas TEST 4) Antioxidantes (idebenone). olvidos frecuentes. demencia de características corticales: 5) No presenta semiología de crisis comicial. Una de estas patologías NO está producida por priones: ausencia de francas apraxias. 2) Crisis parcial simple. 4) 5) Foco en lóbulo temporal. trastornos de concentración y enlentecimiento en los procesos de pensamiento. 4) Estadio lacunar. 3) Más de la mitad de los pacientes suelen presentar algún 5) Iniciaría antiagregación con clopidogrel por tratarse de signo parkinsoniano. 5) Demencia multiinfarto. 4) Insomnio familiar fatal. sión posterior. 39. en relación con la patología que proba- ¿Cuál sería su actitud?: blemente sufre el paciente. 3) Estudio de electrolitos en suero. 4) El EEG no es útil para el diagnóstico. porque probablemente en xia e hipertonía elástica. Mujer de 67 años. NEUROLOGÍA CTO Medicina 1ª Vuelta 34. 1) Enfermedad de Binswanger. 5) Factores neurotrópicos. Etiología subyacente. siguientes medidas no está recomendada de rutina?: 5) El tratamiento antihipertensivo adecuado puede rever- tir los síntomas. automatismos deglutorios y falta de respuesta a estímulos externos. 43. pérdida de interés por las actividades bral. 1) Aparece en pacientes hipertensos mal controlados de que presenta un cuadro progresivo en los últimos dos larga evolución. años. episodios repetidos consistentes en movimientos clóni- 4) Enfermedad de Binswanger. 5) Encefalopatía espongiforme. 3) Crisis parcial compleja. biente. 4) Agnosias. 3) Enfermedad de Strachan. 4) Estudio genético cromosoma 14 (preselinina I). La duración de estos episodios es de 2-3 2) Deterioro de la memoria de fijación. e incluso desorientación en lugares característica de esta patología. Inhibidores de la MAO-B. síntomas. refiere desde hace 2 semanas 3) Enfermedad de Friedreich. MENOS una de estas patologías: 1) Enfermedad de Gerstmann-Straussler-Scheinker. 1) Analítica con función tiroidea. mirada fija. ¿Cuál de los siguientes es el fármaco más útil para la 1) Presencia o no de manifestaciones psíquicas. cotidianos. la respuesta INCORRECTA: 1) Iniciaría tratamiento antiepiléptico. febriles a largo plazo con bajas dosis de valproico. y movimientos tónico-clónicos generalizados. 1) La causa más probable del caso es una lesión ocupante 4) El fenobarbital en niños puede producir un síndrome de espacio a nivel cortical. En el síndrome de West es característico: niveles plasmáticos del fármaco. se inicia tratamiento 55. Sobre el caso. diagnosticada de epilepsia genera- 1) Parestesias hemicorporales. 2) Se trata de una crisis simple motora. Es correcto. zación ocular derecha. 3) Crisis generalizadas mioclónicas. Señale cuál de las siguientes es un patrón característico de 5) Fenobarbital i. de pocos segundos de duración. 2) Iniciar tratamiento con ácido valproico.v. 2) Iniciar perfusión lenta con diacepam intravenoso. familiares. Un paciente de 25 años ha presentado una crisis comicial hemicuerpo derecho. sin claro factor des- secuelas.v. 3) Primero fenobarbital i.v. que desaparece tras 48 horas. de hiperactividad. 4) El episodio tónico-clónico generalizado puede deber. por ser éste el fármaco de 1) Conservación del desarrollo psicomotor a lo largo de elección para su tipo de epilepsia.v. Se encuentra en tratamiento 3) Elevación del brazo con desviación cefálica hacia ese con carbamacepina desde los 20 años y no ha presentado brazo. 3) Añadir ácido valproico. 4) El foco epiléptico parece localizarse en el lóbulo 4) Iniciar tratamiento con vigabatrina. 5 . 5) Felbamato: aplasia medular. de unos 5 2) Valproato: caída de pelo. izquierdo. ¿Cuál es la actitud que hay que adoptar 4) Carbamacepina: reducción concéntrica del campo en este momento?: visual. presenta un episodio de pérdida de conciencia. Un varón de 45 años comienza de modo brusco con 51. medida específica. 4) Aumentar la dosis de carbamacepina. 1) Solicitar RMN urgente. 4) Primero valproato i. con T 38. Mujer de 22 años acude a consulta por un episodio en el antipirético. difenilhidantoína 5) En el caso de tratarse de una lesión tumoral. Duran unos 3 minutos y ceden sin de tipo tónico-clónico generalizado. consulta por estar 2) Alucinaciones visuales en forma de luces centelleantes. que en el contexto de un cuadro gastro- intestinal. Preguntas TEST la enfermedad. 47. 1) Suspender progresivamente el tratamiento. crisis originadas en el lóbulo frontal: 53. el EEG interictal mos afirmar que: y la RM cerebral son normales. 4) Hipsarritmia durante las crisis. parietal izquierdo. Pode. fenitoína i. 1) Esperar a una segunda crisis para instaurar tratamiento.v. 54. mal. 3) Fenobarbital: sedación. que el paciente 52. La actitud terapéutica 4) Sensación de bolo gástrico que asciende seguido de recomendada sería: alteración del nivel de conciencia. valproato sódico se a una generalización a partir del foco hemisférico i.v. y si no cede. dolorimiento a la movili- 5) Inducir al paciente un coma barbitúrico. con movimientos tónico-clónicos generalizados. que en una ocasión son 1) El tratamiento en monoterapia consigue el control de seguidos de pérdida brusca del nivel de conciencia junto las crisis en el 20% de los pacientes aproximadamente.5ºC. rante el embarazo. importancia. 1) Primero clonacepam i. Señale cuál de los siguientes efectos adversos NO es característico de cada fármaco: 49. lo más i. no se toma ninguna ENFERMEDADES DESMIELINIZANTES. NEUROLOGÍA 1ª Vuelta CTO Medicina progresan por miembro superior hasta afectar a todo el 50. Una mujer de 30 años. penthotal sódico i. fenitoína i. Refiere 2) La carbamacepina es el fármaco de segunda elección no haber sufrido nunca accidentes o contusiones de en las crisis de ausencia típicas. 5) Se recomienda aborto terapéutico. 5) Iniciar tratamiento en caso de presentar antecedentes 5) Supone un signo inequívoco de recidiva tumoral. 46. lizada tónico-clónica desde los 20. 2) Está indicada la realización urgente de una TC craneal 5) Es habitual el tratamiento profiláctico de las crisis en busca del diagnóstico. probable es que se trate de una metástasis. La exploración es normal. La actitud terapéutica recomendada sería: 1) Se trata de una crisis clónica secundariamente genera- lizada. 2) Mantener el tratamiento a la dosis actual y vigilar los 48. e intubación. y si no cede. minutos de duración. encadenante. 3) El origen más probable de la crisis es un traumatismo craneoencefálico de baja intensidad. 5) Parpadeo bilateral con mirada fija. y si no cede.v. junto a la hoz. 3) Es una crisis parcial compleja. 1) Fenitoína: hiperplasia gingival. y si no cede. La exploración neurológica es nor.. 3) Como la crisis ha cedido sola. que presentaba visión borrosa. 3) Iniciar tratamiento con etosuximida. es INCORRECTO: 3) La etosuximida es el tratamiento de elección para las crisis parciales complejas. 5) Buen control terapéutico con fenitoína. 4) Si la fiebre supera los 40ºC. No hay deterioro de conciencia asociado.v. El tratamiento de elección en el estatus epiléptico es: haya olvidado.v. crisis desde los 25 años de edad. embarazada de 4 semanas.v. por ser habitual 2) Existencia de patología cerebral subyacente en la una caída de los niveles plasmáticos del fármaco du- mayoría de los casos. 2) Primero diacepam i. en relación al tratamiento de la epilepsia: movimientos tónicos en miembro superior derecho. si no cede. Niño de 6 años. do. el curso de una esclerosis múltiple: 3) En el LCR esperaría encontrar bandas oligoclonales con aumento de las gammaglobulinas. con conservación de motilidad ocular vertical 1) Estudio del LCR. 1) En una TC craneal se podrá objetivar atrofia de la 2) En las formas remitentes-recurrentes puede llegar a cabeza del núcleo caudado. señale la respuesta mientos extraños. progresiva. presenta diplopía de 1 semana de presenta retraso escolar. 1) Depresión. 2) Es habitual que las lesiones desmielinizantes presenten rias. Es FALSO sobre la encefalomielitis diseminada aguda: en la marcha y dificultad en la articulación de las palabras. 1) Se trata probablemente de un caso de esclerosis múltiple. 6 . 4) Esclerosis múltiple. 1) Brote agudo de esclerosis múltiple. por ser ésta la enfermedad vírica nativa más 57. No tiene antecedentes personales destacables. toda probabilidad habría presentado un cuadro simi- teral. 3) Clásicamente se asocia con la infección por el virus del 1) Esclerosis lateral amiotrófica. trastornos de memoria. Señale cuál de entre los 3) TC/RM. 5) Potenciales evocados. Señale cuál de los siguientes síntomas es más frecuente en frecuente en países occidentales. Con respecto al tratamiento para modificar el curso de la su abuelo paterno "perdió la cabeza" y que tenía movi- enfermedad en la esclerosis múltiple. INCORRECTA: Señale. 4) Si el padre del paciente hubiera vivido lo suficiente. que parece hacer de forma volun- 2) EEG. que cedió en unos 4 días. con 5) La administración de estos fármacos es por vía paren. 2) Mielinolisis central pontina 59. En la exploración presenta apertura dimiento diagnóstico NO le aportará ningún dato?: ocular espontánea. 3) El tratamiento de elección es con neurolépticos (blo- 4) El tratamiento con interferon puede ayudar a la recu. acetato de glatiramer. Un paciente de 47 años presenta un cuadro progresivo de 5) No existe tratamiento que modifique el curso clínico de decaimiento. y movi- la enfermedad. peración de secuelas de brotes previos. nivel de conciencia. respecto a las pruebas complementarias. crisis generalizadas y ataxia evolución. Se trata seguidas de período de aplanamiento de la actividad de: cerebral. la enfermedad. siguientes le parece el diagnóstico más probable: 4) Fondo de ojo. señale la 3) Intoxicación etílica aguda respuesta correcta: 4) Encefalopatía de Wernicke. 5) Es un cuadro frecuente y habitualmente autolimita- 4) Movimientos coreoatetósicos involuntarios. la varicela es el agente etioló- 3) Una vasculitis cerebral. es altamente efectivo. tristeza. taria en respuesta a orden verbal. 56. 62. gico más frecuentemente asociado. 1) Lesión fascicular del tercer par derecho. 2) 3) Los corticoides disminuyen el número de brotes. Preguntas TEST 4) El tratamiento concomitante con corticoides e interfe- rón está contraindicado. 1) Los corticoides ayudan a mejorar secuelas previas. 5) El interferón beta es útil para el control sintomático de 5) Panencefalitis esclerosante subaguda. La exploración muestra limitación en la aproxi. El uso de interferón no está indicado en el primer brote. 2) La causa es una infección vírica. tetraparesia. voluntaria y parpadeo. 2) Una facomatosis. 4) Lesión del fascículo longitudinal medial izquierdo. 2) El pronóstico de esta enfermedad es de una superviven- 3) Están disponibles los interferones beta 1a y beta 1b y el cia de entre 10 y 25 años. 4) Si se aplica en fases iniciales. En las exploraciones complementarias desta- mación del ojo derecho con nistagmo en ojo izquierdo ca un EEG con descargas de ondas lentas de alto voltaje cuando intenta la mirada lateral a la izquierda. 4) En países desarrollados. la varicela. PATOLOGÍA EXTRAPIRAMIDAL. El diagnóstico de la esclerosis múltiple es sobre todo clínico. la respuesta FALSA: 1) No hay tratamiento específico para las formas progre- sivas primarias. 63. 3) Neuralgia del trigémino. NEUROLOGÍA CTO Medicina 1ª Vuelta Refiere que hace 3 meses tuvo un cuadro de inestabilidad 61. Sobre el tratamiento de la esclerosis múltiple. 5) Psicosis de Korsakoff. sarampión. Una joven con antecedente de trastorno sensitivo hemi. Usted es consultado para valorar a un niño de 10 años que corporal hace 1 año. reducirse un 30% de los brotes. sobre el diagnóstico más probable de este pacien- te. ausencia de movimien- tos faciales. ¿qué proce. hospital tras ser encontrado en la calle con alteración del pero. señale la respuesta correcta: 3) Lesión del fascículo longitudinal medial derecho. queantes de receptores dopaminérgicos). Diagnóstico que le sugiere: captación de contraste. el tratamiento específico 2) Crisis epilépticas. Sabe que 60. En la RM cerebral realizada se aprecian múltiples lesiones en sustancia blanca y moderada atrofia cortical. En relación con la enfermedad que probablemente pre- 2) Lesión nuclear del tercer par derecho. Se le realiza un estudio con RM. que demuestra placas de desmielinización periependima. Su padre falleció joven en un accidente. mientos continuos involuntarios de forma generalizada. 1) Es una enfermedad de curso monofásico. lar. 64. senta. la más probable. 5) Oftalmoplejia internuclear bilateral. Un paciente con antecedentes de alcoholismo es traído al 58. 5) Signo de Lhermitte. NO es 4) Existe una alteración genética en el cromosoma X. 5) En el tratamiento farmacológico no se incluyen los jos presentes algo hipoactivos y simétricos. 74. 3) Atrofia olivopontocerebelosa. NEUROLOGÍA 1ª Vuelta CTO Medicina 5) El único aminoácido que se encuentra alterado en el hace años. demencia subcortical y 3) Atrofia olivopontocerebelosa. 3) Enfermedad por cuerpos de Lewy difusos. nivel de conciencia. Con respecto a la enfermedad de Parkinson idiopática. bradicinesia. señale cuál de los siguientes diagnósticos le 73. 4) Bromocriptina... con la dosis empleada. esca.). 3) Presenta una dilatación selectiva de las astas frontales 71. No se objetiva antagonistas dopaminérgicos. 1) La estimulación de ambos núcleos subtalámicos es la 2) Enfermedad de Huntington (variante rígida o enferme. bradicinesia. sugerente de enfermedad de Parkinson idiopática?: 5) No existe medicación que detenga la progresión de la enfermedad. 1) Enfermedad de Parkinson. junto con dificultad para la marcha. 5) Los fenómenos «on-off» empeoran al añadir carbido- 2) Síndrome de Steele-Richardson-Olszewski. junto a alucinaciones visuales ocasiona. 4) A la exploración es frecuente encontrar una hipertonía mia facial. mejor diana terapéutica para mejorar. en conjunto. 3) Los cambios en la personalidad son frecuentes en las les y caídas al suelo en varias ocasiones. versión de la mirada. ¿Cuál es su sospecha diagnóstica?: 65. que se encuentran especial- mente en el tronco del encéfalo. En la historia 1) El fenómeno wearing-off es la reaparición de síntomas clínica destaca la presencia de síncopes posturales desde parkinsonianos por descenso de las concentraciones plasmáticas de L-dopa. la dad de Westphal). secundarios del tratamiento a largo plazo con L-dopa en la enfermedad de Parkinson: 70. sintomático. 75. Respecto a la L-DOPA. señale la correcta: 3) Enfermedad de Balo. afectación cerebelosa. ¿Cuál de entre los siguientes datos clínicos. son los cuerpos de Lewy. señale la parece más probable: correcta: 1) Enfermedad de Parkinson. 2) Bradicinesia. con resistencia continua a los movimientos pasivos. 7 . no contraindica la cirugía. 3) El fenómeno «wearing-off» consiste en un aumento de so temblor y marcha atáxica con aumento de base de movimientos coreicos antes de la siguiente dosis. sintomatología del paciente. 4) Enfermedad de Shy-Drager. 2) Síndrome de Steele-Richardson-Olszewski. sustentación. rigidez cervical en extensión. edad. 2) Se trata de una cirugía muy agresiva. 72. señale la respuesta INCORRECTA: 3) Haloperidol. Diagnóstico que le sugiere: 5) Siempre que sea posible. sin pérdida del fases evolucionadas de la enfermedad. 2) El diagnóstico puede establecerse con la clínica y los 5) Hidrocefalia normotensa. aquellos pacientes con deterioro cognitivo y avanzada 5) Enfermedad de Alzheimer. Un varón de 65 años consulta por dificultad para iniciar la micción. rigidez y sistema nervioso central es la dopamina. aumento del tono muscular de forma global. incontinencia e impotencia. 69. pa al tratamiento. Respecto a la estimulación cerebral profunda.. antecedentes familiares. 1) Enfermedad de Parkinson. 1) Asocia movimientos coreicos. 3) Reflejos miotáticos normales. 1) Fluoxetina. 67. 66. 2) Reserpina. Ante estos signos. su diagnóstico de presunción será: 4) Las distonías suelen coincidir con el pico plasmático de L-DOPA. Preguntas TEST aumento de los reflejos miotáticos y parálisis de la infra. cuando se de los ventrículos laterales. ¿Cuál de los siguientes fármacos NO es útil en el tratamien. elástica. Señale la respuesta FALSA con relación a los efectos 5) Hidrocefalia normotensa. 4) Demencia por cuerpos de Lewy. dormido. en pocos meses. 4) Conservación de funciones corticales. Paciente de 71 años remitido a consulta de neurología por 2) El temblor de reposo característico se asocia en nume- cambios de comportamiento de instauración progresiva rosas ocasiones a un temblor postural. 2) Atrofia olivopontocerebelosa. refle. 4) Enfermedad de Shy-Drager. Un varón de 60 años presenta un cuadro clínico consis. to de la enfermedad de Huntington?: 5) Caídas frecuentes. 2) Es poco útil en los parkinsonismos no idiopáticos. es mejor realizar una lesión. Ante un paciente que presenta rigidez. 1) La aparición de efectos secundarios no se relaciona 5) Parálisis supranuclear progresiva. trastornos psiquiátricos. movimientos involuntarios ni temblor de reposo. enfermedad de Parkinson idiopática (piramidalismo. que implantar un sistema de estimulación. 3) Durante la cirugía el paciente está completamente 68. y se reserva para 4) Enfermedad de Creutzfeldt-Jakob. tente en lentitud motora. aunque sí puede pautarse tratamiento 1) Temblor de reposo. presentan desde el inicio del cuadro sindrómico. Es INCORRECTO sobre la enfermedad de Huntington: 1) Enfermedad de Parkinson. 1) El marcador anatomopatológico más característico 5) Tetrabenacina. En la exploración se aprecia hipomi. 4) El hecho de encontrar en la clínica datos atípicos de la inestabilidad postural con frecuentes caídas hacia atrás. Paciente de 65 años que presenta oscilaciones en las 1) Crisis comicial afectando al área frontal derecha aso- manos cuando realiza determinadas tareas como escribir. 3) Desinhibición o moria. Señale 3) Miastenia gravis. clínico descrito: 5) Embolismo sobre el territorio de la arteria cerebral posterior derecha. Entre sus antecedentes personales hiperreflexia. Destaca una hipoestesia hemicorporal derecha que inclu- 2) Los síntomas comienzan antes de los 25 años de edad. Paciente de 76 años que sufre de forma brusca dificultad Preguntas TEST 79. 4) Si se encuentra temblor cinético en la exploración. de pérdida de fuerza en miembros derechos. 5) La principal causa de muerte son las infecciones respi. 1) Hemorragia intraparenquimatosa corticosubcortical de localización temporoparietal derecha. 4) El tratamiento del fenómeno on-off se basa en aumen. ni trastornos de la marcha. ye hemicara. HTA de difícil control terapéutico y sustitución de la que sospechar como diagnóstico probable: cabeza femoral izquierda por prótesis hace 5 años. usar cubiertos o afeitarse. 3) El tratamiento con beta-bloqueantes puede ser una 1) Hemianopsia homónima contralateral. 5) Hay que descartar que el paciente esté siguiendo algún 5) Apraxia de la marcha en lesiones bilaterales. 1) Con frecuencia se encuentran en la historia antece- dentes familiares de síntomas similares. La explora- 5) Estos fenómenos no suelen aparecer en las primeras ción revela desviación conjugada de los ojos a la izquierda fases de la enfermedad. La exploración neurológica es normal. asociando tra de forma progresiva paresia distal en miembros con dificultad para hablar. importante de la patología cerebrovascular isquémica: 5) Infarto isquémico carotídeo derecho extenso. Es traído hoy a Urgencias por cuadro brusco de L-dopa. Un paciente de 56 años presentó hace dos meses un IAM tar de forma sostenida las concentraciones plasmáticas anteroseptal. amiotrofia y fasciculaciones. Varón de 57 años. 4) Hemiparesia contralateral de predominio crural. 83. 5) Es urgente descartar patología ateromatosa carotídea. No presen. Señale cuál de las siguientes patologías NO se asocia con teria cerebral media izquierda. En la exploración no se aprecia déficit motor y la comprensión y expresión del lenguaje están conservadas. para sostener cosas con la mano izquierda. limitación en la amplitud o rapidez izquierda. 5) Tumor medular dorsal. Mejora con la ingesta moderada 2) Embolismo cardíaco sobre la arteria basilar con lesión de alcohol. NO se incluye entre los síndromes lacunares. 4) Si la prueba de imagen cerebral urgente es normal. uno de los 2) Hipercolesterolemia. 3) Infarto lacunar en ganglios basales derechos. parcial 4) Malformaciones esqueléticas. sin deterioro cognitivo asociado. 2) Abulia. siguientes: 8 . ojo izquierdo. lo 2) Miocardiopatía. lo más probable es encontrar en un infarto de la arteria cerebral anterior?: que desarrolle un parkinsonismo en los próximos años. 3) El tratamiento del fenómeno wearing-off se basa en 5) Diabetes mellitus. señale la respuesta correcta: 4) Las respuestas cutáneo plantares son extensoras. A la exploración presenta desviación de la comisura bucal 1) Atrofia muscular espinobulbar. buena opción como terapia sintomática. terior izquierda. 2) Presenta probablemente isquemia en territorio de ar- 78. Señale cuál de los siguientes es el factor de riesgo más 4) Infarto lacunar en cápsula interna derecha. ¿Cuál de los siguientes signos y síntomas NO esperaría 2) Dada la existencia de rueda dentada. Se trata de: 76. y hemianopsia derecha por amenaza. Con respecto al diagnóstico más probable de 3) Suele haber hipo-arreflexia generalizada. Señale la 4) Embolismo sobre el territorio de la arteria cerebral opción que le parece INCORRECTA con respecto al caso posterior izquierda. el diagnóstico más probable: 4) Esclerosis lateral amiotrófica. habrá que descartar patología de fosa posterior.habría co. de instauración respuesta INCORRECTA: brusca. 1) Es la ataxia hereditaria más frecuente. 1) Deterioro cognitivo. 5) Diabetes mellitus. salvo isquémica de la hemiprotuberancia derecha. con función destacan: hipercolesterolemia leve en tratamiento dietéti- esfinteriana íntegra. 4) Tabaquismo. mantener concentraciones plasmáticas estables. 82. NEUROLOGÍA CTO Medicina 1ª Vuelta 2) El fenómeno on-off son fluctuaciones motoras que no 3) Tratamiento con anticonceptivos orales. La exploración sensitiva es normal. cuya exploración neurológica demues. más probable es que se trate de un estatus epiléptico 3) Trastornos de conducción cardíaca. 85. tienen relación con los niveles plasmáticos de L-dopa. de ramas parietales. 84. ciando parálisis de Todd postcrítica. En relación con la enfermedad de Friedreich. la enfermedad de Friedreich: 3) Los datos clínicos sugieren afectación en el territorio de la arteria cerebral posterior izquierda. este enfermo. 1) HTA. 80. una discreta rueda dentada en mano derecha. hacia la derecha con capacidad conservada para cerrar el 2) Enfermedad de Kugelberg-Welander. Paciente de 58 años que acude a Urgencias por pérdida 77. 1) Es preciso la realización de una RM cerebral ante la ratorias de repetición. de los movimientos. 81. señale la de visión en el hemicampo derecho. 2) Lesión isquémica en territorio de arteria cerebral pos- PATOLOGÍA VASCULAR CEREBRAL. 3) Embolismo sobre territorio de la arteria cerebral media ta temblor de reposo. tratamiento que pueda justificar la sintomatología. sospecha de patología de fosa posterior. 000/mm3. día. 4) Embolia paradójica a través de un foramen oval per. se debe 92. to de elección es antiagregación. 89. Acerca del tratamiento fibrinolítico con t-PA. neuropatías diabéticas: 3) Si en el estudio angiográfico se aprecia estenosis de menos del 30% en la carótida sintomática. 5) La amiotrofia diabética es de predominio proximal en 87. ría en primer lugar?: 4) Una presión arterial mayor de 185/110 mmHg en una determinación. que le cedieron con un masaje. Un varón de 30 años presenta un trastorno en la relajación 1) La disminución en la amplitud de los potenciales de de ambas manos. 2) Si se aprecia estenosis en el eco-Doppler. más ostensible en las minutos. sin soplos. SISTEMA NERVIOSO PERIFÉRICO. 3) Punción lumbar. 1) La más frecuente es la polineuropatía distal simétrica. de 10 días de evolución. toria grave. talámico de 1. agresiva. 3) Ictus sensitivo puro. Señale cuál es la causa más probable de isquemia en el 3) Plasmaféresis repetidas en la fase inicial de la enfermedad. 2) Administración de ciclofosfamida a dosis de 2 mg/kg/ día. auscultación cardíaca rítmica sin soplos. 2) La dispersión de los potenciales de acción es caracte- 2) Ictus motor puro. un caso de Guillain-Barré es: Preguntas TEST 2) Malformación vascular. el tratamien. ¿Qué exploración realiza- 3) Historia de hemorragias intracraneales. puesta INCORRECTA: 3) Mononeuritis múltiple. A propósito de la distrofia muscular progresiva de tipo meable. 86. Carótidas rítmicas y simétricas. en las neuropatías axonales y en las desmielinizantes. 1) Su transmisión es autosómica recesiva. el tratamiento se realiza con anticoagula. 5) Rotura de aneurisma fusiforme. La TC craneal de un paciente muestra un hematoma 4) EEG. 3) Rotura de aneurisma sacular. ventilación mecánica en caso de insuficiencia respira- 2) Aterotrombosis de grandes vasos extracraneales. 88. Paciente de 58 años que acude por presentar episodio de 5) No existe lesión axonal y desmielinizante concomitan- pérdida de visión en ojo izquierdo. 95. El examen físico 2) Pericarditis postinfarto de miocardio. afirmación que NO debe considerarse un criterio de exclusión: 93. Duchenne. 2) La mononeuropatía craneal más frecuente es el III par 4) Si se detecta estenosis entre el 30 y el 50% de la carótida incompleto. La principal medida terapéutica que se debe adoptar ante 1) Angiopatía amiloide. 3) La biopsia es patognomónica. 5) Vigilancia de la función respiratoria e intubación y 1) Aterotrombosis de grandes vasos intracraneales. 1) Electromiografía. 2) Se inicia por las extremidades superiores. cia o las diarreas son sugerentes de neuropatía vegeta- 5) En el caso de encontrarse una estenosis de más del 70 tiva. rística de la afectación axonal. Señale la res. 2) Neuropatía sensitiva distal. que le empeora con el frío. 4) Los bloqueos de conducción son sugerentes de lesión axonal. % en la carótida sintomática. de inicio en campo temente en un mismo nervio. de duración total de unos dos minutos. frontal y debilidad facial bilateral. 90. 4) Aumenta la creatinina urinaria. enunciado correcto: 96. Refiere haber presentado días antes 91. Fondo de ojo: sin hallazgos. Señale el enunciado INCORRECTO con respecto a las realizar angiografía posteriormente. señale la respuesta correcta: 5) Trombosis in situ. 1) Polineuropatía desmielinizante aguda. la impoten- ción oral.5 cm de diámetro. 1) Administración de prednisona a dosis de 1 mg/kg peso/ 4) Hipertensión. 4) Neuropatía amiotrófica proximal. 3) Síntomas como la hipotensión ortostática. Señale la causa más 5) TC cerebral. Señale cuál de las siguientes neuropatías periféricas NO se episodio de escasos minutos de duración de hormigueos ha relacionado con la infección por VIH: en la mano derecha. 2) Arteriografía cerebral. eco-Doppler de troncos supraaórticos. 3) La velocidad de conducción se encuentra disminuida 4) Ataxia-hemiparesia. 1) La primera prueba diagnóstica a realizar es un estudio 5) Polirradiculitis. 5) Disartria-mano torpe. probable de esta patología: 94. señale el 5) Las alteraciones predominan en las fibras tipo I. NEUROLOGÍA 1ª Vuelta CTO Medicina 1) Hemianopsia homónima. superior. territorio de la arteria cerebral media: 4) Medidas fisioterapéuticas adecuadas. demuestra ausencia de reflejos. En el diagnóstico diferencial de las neuropatías. señale la cintura pélvica. inferiores. calvicie acción es típica de la afectación axonal. Varón de 25 años que presenta una infección respiratoria de vías altas. 3) Embolia cardiogénica. hay que plantearse la 4) Son potencialmente reversibles si se tratan de forma posibilidad quirúrgica. A la exploración. sintomática. Quince días más tarde acude al hospital por 1) Mejoría rápida de la sintomatología en los primeros pérdida de fuerza en extremidades. 5) Recuento plaquetario inferior a 100. El diagnóstico clínico es: 9 . de unos 45 minutos de duración y que se acompaña de lagrimeo. 4) Electroneurografía. NEUROLOGÍA CTO Medicina 1ª Vuelta 1) Enfermedad de Becker. Varón de 30 años con debilidad muscular de predominio vespertino que empeora con el ejercicio y mejora con el reposo. El paciente ya ha acudido varias veces a urgencias y está en tratamiento con Verapamilo a dosis de 80 mg/6 horas para disminuir la frecuencia de los ataques. 3) Es preciso la realización de una RX de tórax para confirmar el diagnóstico. no presentaba alteraciones sensitivas ni amiotro- fias y los reflejos musculares profundos estaban conser- vados. inyección con- juntival y disminución de la hendidura palpebral en el mismo lado. 5) Miopatía hipertiroidea. que refiere visión doble y caída de los párpados de unos meses de evolución. como una puña- lada en el ojo derecho. 3) Distrofia facio-escápulo-humeral. 5) Cefalea en racimos. Es INCORRECTO en relación con la miastenia gravis: 1) La aparición de ptosis y debilidad fluctuantes a lo largo del día deben hacer sospechar el diagnóstico. extraoculares y en los de las extremi- dades. Un varón de 45 años presenta episodios diarios de cefalea consistentes en dolor periocular intenso. 100. La exploración sensitiva es normal. rinorrea. 5) Estimulación repetitiva. 3) Electromiografía de fibra simple. 4) Migraña oftálmica. ptosis palpebral y diplopía. 97. 5) Distrofia oculofaríngea. 10 . 2) Arteritis de la temporal. 2) La clínica y los hallazgos neurofisiológicos pueden ser suficientes para establecer el diagnóstico. La paciente dice encontrarse unos días mejor que otros y mucho mejor al levantarse por la mañana que por la noche. 3) Miastenia gravis. Preguntas TEST CEFALEAS. 2) Síndrome miasténico de Lambert-Eaton. En la explora- ción se puso de manifiesto fatigabilidad muscular en los músculos de la cara. 2) Determinación de anticuerpos antirreceptor de acetil- colina. 98. 99. A estos síntomas iniciales se les añadió dificultad para tragar y debilidad en las extremidades inferiores.¿Qué prueba NO utilizaría para con- firmar el diagnóstico?: 1) Test de Tensilon. 3) Hemicránea paroxística. 4) La alteración tímica más frecuentemente asociada es la hiperplasia. Paciente de 25 años de edad. Los reflejos miotáticos están conservados y no hay amiotrofias. 5) En las formas oculares puras el tratamiento de elección son los anticolinesterásicos. 2) Enfermedad de Steinert. ¿Cuál es el diagnóstico más probable?: 1) Botulismo. Lo más probable es que se trate de: 1) Neuralgia del trigémino atípica. 4) Esclerosis múltiple. 4) Distrofia muscular de las cinturas. En fases avanzadas puede encontrarse disminución de la masa Las diplopías que se describen como una limitación en la motili- muscular (opción 5). se dividen en superiores (D6-9). hipertonía elástica. No sucede igual en neuromuscular de un músculo concreto de los encargados de mo- las lesiones de la segunda motoneurona. dad del globo ocular en una dirección concreta apuntan a lesión condicionada por la menor actividad muscular.. que se sitúa en la protuberancia. en este caso izquierda. la lesión estará situada en el tronco del encéfalo. Allí se encuentran los cuerpos des derechas. y del movimiento de adducción (hacia la línea me- nan por una conexión monosináptica (entre una neurona sensitiva y dia). punta de la nariz (infraversión y rotación interna). lesional como para la lateralidad. que en el caso concreto de la pared del abdomen. En el caso de los síndromes cruzados sensitivos. La lesión es homolateral a la cara En fases muy precoces de lesión de primera motoneurona los afectada. inferior del ojo. especialmente en las lesiones medulares. dependiente del grado de pérdida de fuerza. pregunta. por lo que la opción incorrecta es la 4. paresia espástica. ya que la 1234565789 57 . Diagnóstico diferencial de lesión de 1ª y 2ª motoneurona. si se preservan las segundas. donde se lesiona la vía piramidal antes de primeras motoneuronas. que tenemos suficientes datos para dar con la opción correcta. el origen más probable es medular al estímulo en la piel abdominal. motoneuronas. apunta a una lesión nuclear en la protuberancia izquier- 11) e inferiores (D11-L1) y suelen abolirse en lesiones de las primeras da. NEUROLOGÍA 1ª Vuelta CTO Medicina Pregunta 1. dan lugar a una decusarse en el bulbo. sitivo. mientras que los reflejos cutáneos sue. medios (D9. y tardan días o semanas en exaltarse. junto al núcleo del VII par izquierdo y la vía corticoespinal. Las lesiones de las siones del tronco del encéfalo. Si el paciente tiene limitación anteriores. neuronales de las primeras motoneuronas encargadas de inervar el Los síndromes cruzados son la presentación característica de las le- hemicuerpo contralateral de forma voluntaria.R: 4 Ante la presencia de un síndrome cruzado. y no se comportan de la misma manera que los una acción de abducción casi pura. Los reflejos cutáneos abdominales reflejan la respuesta en la mirada externa con el ojo izquierdo. clonus y respuesta cutáneo-plantar lesión viene definida por la afectación craneofacial. y los núcleos motores del tronco del encéfalo. o extraocular).R: 4 que el resto de la exploración que presenta el paciente indica un El área motora ocupa la circunvolución prerrolándica o área 4 síndrome cruzado. el IV par contribuye a un movimiento similar al de mirar a la una segunda motoneurona). será una afectación baja. El III par se encarga de la motilidad vertical superior e Los reflejos de estiramiento muscular o miotáticos se desencade. sea éste motor o sen- Pregunta 1. con lo piramidal (médula). con reflejos de estiramiento muscular aumentados Ante la presencia de un síndrome cruzado. la localización de la (opción 2). en este caso el VII. con una contracción de la la afectación del VI par izquierdo.. y el VI par realiza len ser polisinápticos. con afectación facial izquierda y de extremida- de Brodmann en el lóbulo frontal. en las que la amiotrofia es ver el globo ocular (lo que se denomina motilidad ocular extrínseca precoz y más intensa. Pregunta 4. sobre todo en lesiones de la vía afectado. tanto para el nivel extensora (signo de Babinski). y el nivel lo indica el par craneal reflejos están abolidos (opción 1). 234565789 57 porción central del trigémino (el núcleo sensitivo central del trigémi- no) se sitúa en su mayor parte en el bulbo y en la parte más inferior de 123456789 36782 7. 4 7 7. 4678 574 678683 67 la protuberancia. Dado que entre las respuestas no está la opción de 9 36782 7. a pesar de 53 6 873 75. la respuesta no puede ser más que la 4. 68 256 65 8684836 87 76 Hay que recordar. con respecto a los síndromes sensitivos. que 1256 65 85236   una lesión parietal no produce anestesia hemicorporal. 4 7 la protuberancia. sino trastornos en las 467853 .5  encontrarse allí la corteza sensitiva primaria. 567 4678536 . esto es. en el reconocimiento adecuado de los estímulos sensitivos (agnosias sensitivas). 73 7 8.567868 77 sensibilidades combinadas.  634  73 7 8.  634  84!6 Las lesiones talámicas sí producen anestesia hemicorporal !76 8" #57$5 completa. que incluye la hemicara. suele acompañarse de episodios lancinantes de dolor es- 68 7. y que si se establece de forma definitiva. 5. 4 . 567 % 8!575 8 7. 5. 4 . en realidad es un Pregunta 6.R: 5 relación con la desaferentización sensitiva. siendo la motoneuronas. la hemicara indica el lado de la lesión. y el corteza frontal): dan lugar a defectos hemicorporales contralate. por tanto.) y debemos pregunta. La afectación de las extremidades se un III par. que se sitúan en el mismo lado del músculo al causa más frecuente en nuestro medio la neuropatía isquémica del que inervan. en los que aparece sintomatología en un hemicuer. por lo que dan lugar a alrededor del nervio. o del que aún no ha tenido lugar ninguna decusación. dado La presencia de ptosis y miosis es altamente sugerente de síndro- me de Horner por lesión del simpático cervical. es diagnóstica de un equilibrio simpático-parasimpático. simpático (provoca dilatación). La existencia de una pupila midriática indica lesión de la vía En el caso de la pregunta. La inervación simpá- 1 . síndromes cruzados. produce por la lesión de la vía corticoespinal en su trayecto por el Recuerda que las fibras pupilomotoras están localizadas periféricamente tronco del encéfalo. las otras se acompa- ñarán de dificultad en los movimientos del globo ocular y de ptosis. se sitúa medial. Hay que pensar en brazo y pierna en el mismo hemicuerpo. y por eso es la 5 la trayecto del III par (seno cavernoso.R: 2 • Lesiones troncoencefálicas: típicamente dan lugar a los síndro. aún sin decusar. de unas pupilas rales que incluyen la hemicara y las extremidades. En los diabético. al tener un defecto en la hemicara. mes cruzados. hay que recordar los portadora de la información del dolor y de la temperatura. órbita). permanente del estímulo parasimpático (produce constricción).. Pregunta 3. puesto (mesencefálico posterior). Aunque existe una parte de Esta disposición anatómica hace que en la lesión bulbar lateral se vea axones de primeras motoneuronas que descienden por el cor. por lo que producen defectos ipsilaterales. basarnos exclusivamente en la distribución. tono. y de movi- mientos pseudoatetoides de la mano afectada. La presencia. y el paciente presenta bratoria que asciende por los cordones posteriores. y en el dón anterior y se decusan en la médula. y que una lesión compresiva se caracteriza por defectos contralaterales. Cuando un paciente presenta un déficit motor del que no sabe. En este caso se trata de una afectación del III par aislada. del recto superior e inferior. Comentarios TEST una hemiparesia ipsilateral a la lesión. mientras que la afectación facial se pro. Por lo que respecta a las lesiones bulbares. sabemos que se trata de lesiones a nivel del núcleo parasimpático o de Edinger-Westphal una lesión superior al tronco del encéfalo.567 pontáneo en el hemicuerpo anestesiado (dolor talámico).R: 4 Aunque el motivo de la consulta es la diplopía. Las lesiones del núcleo respuesta correcta.. Siempre que se nos presenta po y en la hemicara opuesta. etc. pueden tener afectación exclusivamente pupilar. hay que recordar que en el bulbo la vía espinotalámica. parasimpática con conservación de la simpática... discurre distintos patrones: por la porción lateral. afectada la sensibilidad termoalgésica y no la propioceptiva. lo siguiente que hay que valorar es cómo está la pupila. corticoespinal se decusa en el bulbo y se sitúa en el cordón lateral ipsilateral al hemicuerpo que va a inervar. Pregunta 5. medias.R: 5 dato que no es imprescindible para responder correctamente. la mayor parte de la vía síndrome bulbar medial suceda al contrario. las segundas par. una midriasis asociada a la debilidad de la musculatura que inerva el III duce por lesión de los núcleos motores. y contralateral. probablemente en Pregunta 2. La afectación del elevador del párpado. como es el caso de la mos sus características exploratorias (reflejos. nos revela la afectación del III par craneal. mientras que la información posicional y vi- • Lesiones medulares: no afectan a la cara. es decir. El equilibro en el diámetro pupilar viene determinado por la acción • Lesiones por encima del tronco del encéfalo (cápsula interna. de la porción mesencefálica del III par. medular posterolateral En miembros superiores se objetivan amiotrofias. siendo extraño que transversa presente un defecto proximal y no distal. muy cerca del centro de la médula. lo fundamental es recordar las principales vías y su dispo. con dilatación (hemisección medular) del conducto ependimario y lesión de las vías adyacentes. encuentran su segunda neurona sensitiva en el asta posterior medular. lo más probable es que cursara con hiperreflexia generalizada.R: 3 La pérdida de sensibilidad térmica y dolorosa nos debe hacer Si la lesión es distal a la bifurcación carotídea. lo que indica afectación de los cordones poste- riores medulares a nivel cervical. y decusándose para alcanzar el tálamo con. dado que lesiones traumáticas o compresivas ocasiones por una lesión isquémica en el territorio de la arteria espi- pueden dar síndromes aproximados que no encajen por completo nal anterior. con un trayecto ascendente. constantes. un trayecto ascendente. lo que sugiere lesión de segunda motoneurona. de esfínteres ni sensitivas. trate de una lesión funicular (del cordón). • Anhidrosis. En el bulbo la vía se sitúa lateral. con lo que se produce una compresión de curso tralateral)... to incluiría pues: • Disminución de hendidura palpebral. NEUROLOGÍA CTO Medicina 1ª Vuelta tica se encarga de la dilatación pupilar. El hecho de que se trate de un nivel lesional suspendido (con lares) y del tronco del encéfalo.R: 4 El axón de esta segunda neurona sensitiva de la vía se decusa con Pregunta 7. segmento en que la médula alcanza neuronas sensitivas. la que no se relaciona con el nervio periférico. Son fibras III y IV. Sd. describe es cada uno de los defectos neurológicos. motivado por un déficit de vitamina B12. dado que es recogida por unas fibras independien- síndrome de Horner es la neurolúes. en Comentarios TEST este caso no es compatible. Pregunta 10. por lo que cada cor- dón lateral lleva las vías motoras para el hemicuerpo ipsilateral. Sd. No afecta en lo que conocemos previamente. inicial. Pregunta 8. de la musculatura tarsal (con. La clínica aislada de anestesia termoalgésica bilateral y simétrica debe hacer sospechar una lesión centromedular. Sd. Una lesión de esta vía a nivel medular cursaría con discurre por separado de la pupila a partir de ese punto. bratoria. mal pronóstico en lo que respecta a su recuperación. dado que si fuera por encima de los segmentos medulares cervicales. los cordones posteriores y cursa con una plejia bilateral aguda. lesión. cordonal posterior lateral amiotrófica (ELA): de curso progresivo en la edad adulta. del vértice pulmonar afectarían a toda la parte inferior del hemicuerpo desde el nivel (tumores pulmonares). En un paciente con clínica de afectación de primeras y segundas motoneuronas hay que considerar la posibilidad de una esclerosis Sd. Síndromes medulares. por paresia del mús. esta patología asocia otro tipo tes del resto de sensibilidad táctil. La causa más fre- Sd. contralateral. que irriga los dos tercios anteriores medulares. la sintomatología que presentará un paciente del que conozcamos la • Enoftalmos (aparente para algunos autores. Brown-Sequard cuente de lesión centromedular es la Siringomielia. lo que indica lesión de la vía piramidal. pasando por la comisura anterior medular. la anhidrosis y la sospechar una lesión de la vía espinotalámica. o interpretar la distribución de la misma si lo que se nos culo orbitario). pobremente de alteración pupilar: la pupila de Argyll-Robertson. Su de. las neuronas motoras presentes a ese nivel.. cusación se produce en el bulbo raquídeo. lo que explica la amiotro- zar el nivel correspondiente de la segunda motoneurona. su máximo grosor. fia coincidente con la lesión. medular anterior La degeneración subaguda combinada de la médula presenta deterioro de cordones laterales y posteriores medulares. hasta alcanzar el cordón anterior y lateral contralaterales. Unas pistas para en- segunda neurona sensitiva en el asta posterior medular. por la que discurre vasodilatación no estarán presentes. medular central que se acompaña de reflejos vivos y signo de Babinski. de presión y tacto hasta el tálamo (pasando por unos La mielopatía cervical se produce por una disminución del espa- núcleos en el bulbo raquídeo. me bulbar medial. mielinizadas algunas. de curso ascendente. La lesión de las neuronas III y IV periféricas daría lugar a un defecto Mielopatía con patrón polineuropático. de la médula cervical (lesiones centromedu. El síndrome espinal anterior se produce en la mayoría de las sición en la médula. por lo que la amiotrofia en Aunque algunos síndromes medulares son muy característicos y manos no apoya el diagnóstico. Conociendo las principales vías medulares. sitúan en la porción anterior-lateral medular. de • Cordones posteriores: conducen la sensibilidad posicional.y la espasticidad en miembros inferiores –cordones laterales-) y a den en su mayor parte por el cordón lateral medular hasta alcan. que tras entrar por la raíz posterior medular. Cada cordón posterior contiene la información del lentamente progresivo. res. vi.R: 4 • Vía espinotalámica: las fibras sensitivas del nervio periférico Las descripciones de los trastornos del movimiento que cursan que recogen las señales del dolor y la temperatura se decusan con exceso de movimiento (hipercinesias) no siempre hacen fácil- en la médula. dado que dicha inervación esta sensibilidad. Pregunta 9. La sensibilidad termoalgésica también puede verse afectada en el De las opciones de la pregunta. y el paciente presenta signo de Lhermitte. donde se encuentran las segundas cio en el canal medular cervical. tras encontrar su mente identificable el movimiento en cuestión. en guante y calcetín.R: 1 Tenemos un paciente con déficit motor en miembros inferiores. y se focar estos trastornos: 2 . como lo sería entre D3 y D6. finas. no es difícil adivinar • Miosis.. Pregunta 7. la vía tribuye a la elevación del párpado) y de las glándulas sudoríparas y espinotalámica en la médula conduce información sensitiva los vasos sanguíneos de la hemicara. afectando a las vías ascendentes y descen- hemicuerpo ipsilateral. dado que la ELA debe cursar sin altera- ciones cognitivas. tos por debajo del nivel lesional (debido al trayecto ascendente de en el diagnóstico diferencial del síndrome de Horner hay que incluir los axones que acceden a la vía en el interior de la médula) y que las lesiones de la carótida interna (disección). Por lo tanto. El síndrome de Horner comple. No cursa con afectación de segundas motoneuronas. El hecho de que en miembros superiores no haya asimismo hiperreflexia sugiere que la lesión es medular. • Vasodilatación. por áreas íntegras por encima y por debajo de la lesión) descarta que se lo que se ve afectada en el síndrome de Wallenberg y no en el síndro. dentes medulares (lo que justifica el Lhermitte –cordones posterio- • Vía piramidal: los axones de las primeras motoneuronas descien. alteraciones sensitivas termoalgésicas que comenzarían dos segmen- Dado el complejo trayecto del simpático hasta alcanzar la pupila. con mínimo recorrido. • Las sacudidas musculares. movimientos anormales.R: 2 nan en el área facial. mientras en las lesiones mesencefálicas coexis- exacerba en situaciones de cansancio. NEUROLOGÍA 1ª Vuelta CTO Medicina • Las contracciones musculares mantenidas típicas son las distonías: da en determinados ganglios basales. bradicinesia y temblor escalones terapéuticos: de reposo. probablemente. puede aparecer en enfermos de Parkinson aso- oníricas de caída. mayor al comienzo del movimiento y en movimientos rápidos. No es útil en las distonías que afectan a mu. nista y antagonista. El desuso to clínico de otras encefalopatías tóxico-metabólicas. en el que predomina esta forma de debut. benzodiacepinas. establecen como fijas. con distintos tipos de herencia cos como la lesión o estimulación cerebral profunda selecciona. más que a una recuperación completa. de inicio típicamente en la edad infantil. lentos. En el parkinsonismo. la causa La descripción de un temblor que sólo aparece cuando no se (primarias. aunque oca- también produce hipotonía. Todo el mun. por actividad alternante agonista-anta. observándose las sacudidas porque es incapaz de mantener la con una resistencia inferior a la normal al movimiento pasivo. (temblor rúbrico). englobarlo dentro de un síndrome clínico: mientos coreicos son rápidos. de amplitud variable. con las descripciones de situaciones en las que pueden estar en relación con: aparece el temblor. dentes familiares. Sólo el hecho de que plantar es flexora. y es típico de la patología cerebelosa y mesencefálica las oscilaciones. si el paciente refiere un temblor postural de instauración chos músculos. autosómica dominante con alta penetrancia). Los temblores se distinguen en distintas • El temblor cinético aparece en los movimientos con amplio re- clasificaciones. hasta que se los casos de enfermedad de Parkinson farmacorresistente. en efecto hay que preguntar por los antece- • Cirugía: en las distonías que no responden a los tratamientos des. Es el trastorno del movimiento más común. Si la espasticidad fijas (manos adelante) o con mínimo recorrido (escribir. con la peculiaridad de que en los casos cere- do tiene un cierto grado de temblor (temblor fisiológico) que se belosos es aislado. posible. talón- músculo (fenómeno de navaja). postural y cinético en el mismo paciente. recordar que el temblor cinético e intencional se suele se ejerce una fuerza continua. la frecuencia de corrido. según la forma de presentación. Es. Pregunta 15. obteniéndose un beneficio que per. • Toxina botulínica: es la mejor opción para la distonía cervical. La asterixis se explora pidiendo al paciente que mantenga una La disminución del tono muscular (hipotonía) es una pérdida del posición fija contra gravedad (generalmente las manos en dorsifle- tono normal en la que los músculos aparecen fláccidos y blandos. que precede a una disminución del nivel de conciencia y entrada Se han utilizado distintas clasificaciones. similar a la que se realiza en tienen una instauración lenta y progresiva en el tiempo. rodilla) mientras que el temblor postural se describe en posiciones cos vivos y respuesta cutaneoplantar extensora. sionalmente puede presentarse en pacientes con lesiones hemisféri- cas cerebrales. etc. Comentarios TEST las miopatías. las basa. ejerce acción muscular en la parte afectada (dejada caer a favor de das en los defectos genéticos asociados. • Rigidez: existe un aumento de tono muscular mantenido en mus. nes rítmicas debidas a la actividad alternante de la musculatura ago- miento farmacológico. tropezón. o apoyada) es diagnóstica de temblor de reposo. El temblor intencional es el empeoramiento o la aparición del temblor al final de un movimiento. todo el mundo presentamos. Pregunta 14. xión). La El temblor es un trastorno del movimiento definido por oscilacio- inmensa mayoría son simples y benignos. cabe la duda de si se trata de un temblor cinético. etc. postural. Se presenta típicamente en el temblor periférico.. gonista. que marcan la edad de comienzo (precoces si lo hacen antes de los 25 años y tardías a las que aparecen más tarde). es la forma más frecuente de temblor de causa nagógicas (al coger el sueño) que suelen coincidir con imágenes farmacológica. mulantes. aunque pueden afectar a cualquier área corporal. de predominio en manos.. 3 . cuadro de inquietud psicomotora. Pue- • Los movimientos continuos son coreoatetósicos: los atetósicos de clasificarse siguiendo varios criterios (ver figura en página siguien- son movimientos continuos. en realidad puede aparecer en el contex- hipotonía algo más leve que la descrita anteriormente. contorsiones y. limitando la actividad funcional del sujeto.. pueden ser subsidiarios de procedimientos quirúrgi. segmentarias. servir agua). tos. usar cubier- es muy intensa y mantenida puede llegar a dar lugar a una con. el más frecuente es el temblor siste semanas-meses. es en la mayoría de los casos familiar. que también • El temblor de reposo es visible cuando la parte afectada está en pueden afectar a cualquier parte del cuerpo y que cuando son reposo (con apoyo o sometida a efecto de la gravedad) y hay que generalizados aparecen como un baile (chorea = baile) pensar en un síndrome parkinsoniano. asimétrico (en fases iniciales tiepilépticos y un largo etcétera. si no se corrige a tiempo.R: 1 culatura flexora y extensora. Pregunta 13. • Los movimientos repetitivos estereotipados son los tics: predomi. secundarias) y. ya que la sospecha sería un temblor esencial. pero lo más útil te. por • Fármacos: agentes dopaminérgicos (agonistas dopaminérgicos. Los refle. esencial benigno. • Espasticidad: presente cuando se pierde la función inhibitoria dado que hay acción. los que vienen definidos continua a la movilización pasiva en cualquier dirección. Las postura. an. si Como ayuda. anticolinérgicos.R: 3 La encefalopatía hipoglucémica característicamente tiene una fase La distonía es un trastorno del movimiento que se define por inicial. el más objetivo es según la frecuencia del temblor. el con lo cual. en el electromiograma. dado que las acciones que ejerce la vía piramidal sobre la segunda neurona motora. y son arrítmicas. o un temblor postural.R: 3 tómica (focales. litio. gravedad..R: 3 matología de un temblor cinético. Las lesiones cerebelosas pueden cursar con una denominándose flapping. salvo que coexista una lesión de la vía piramidal. que critos arriba. que predominan distalmen. Se acompaña de reflejos miotáti. con lo que aparece una resistencia Dentro de los movimientos involuntarios. con un contracciones musculares sostenidas que producen posturas anor. Es son mantenimiento de posturas casi fijas. taquicardia. hereditarias. la distribución ana. se especifique que aparecen periodos silentes (o de silencio eléctrico. consumo de esti. o las patologías de base asociadas. (la más frecuente. Se inyecta directamente mano levanta la sospecha de enfermedad de Parkinson. nos da el diagnóstico sindrómico de asterixis. es. los movi. la mera aparición de un temblor de reposo en una blefaroespasmo y la distonía espástica. generalizadas). Todo el mundo tiene mioclonías hip. estrés. Los aumentos del tono muscular (hipertonías) En esta pregunta. El tono muscular viene definido por la resistencia del músculo a la movilización pasiva. breves y saltatorias son típi. se produce una relajación del describir en maniobras de exploración de ataxia (dedo-nariz. marcada por el aumento de catecolaminas circulantes. hemidistonías. y va encaminado a la mayor recuperación funcional niano. sudoración y midriasis males. más si se especifica que son rápidas y arrítmicas. jos de estiramiento muscular son normales y la respuesta cutáneo. baclofén. De los distintos tipos de temblor. medular o postura contra gravedad. ciado al temblor de reposo típico. y no precisan trata. y puede ser una primera sinto- Pregunta 11. más recientemente. siendo las más útiles las progresiva en coma hipoglucémico. otra parte.. en el músculo seleccionado. tracción mantenida (distonía espástica). de reposo. • El temblor postural aparece cuando se intenta mantener una cas de las mioclonías: pueden tener origen cortical. te). y éste a su El tratamiento de las distonías depende de la distribución y la vez conlleva inmediatamente la sospecha de un síndrome parkinso- intensidad. son el temblor. que no hay actividad muscular en determinados momentos) rítmicas de la resistencia (fenómeno de rueda dentada). lentamente progresiva. ten temblor de reposo. sólo tiembla una mano) e inicialmente sin otros signos o síntomas. arrítmicas. causas más frecuentes son las lesiones de la segunda motoneurona o Inicialmente descrita en relación con la insuficiencia hepática. como sacudidas musculares deben hacernos pensar en mioclonías. y es el temblor fisiológico que • Las oscilaciones rítmicas. el más frecuente es la Enfermedad de Parkinson. en ocasiones. Pregunta 12. con un temblor levodopa). Se dispone de tres De los síndromes que cursan con rigidez. la rigidez se acompaña de interrupciones es decir. independiente. por lo que el movimiento resultante resulta irregular. difi- bilidad profunda. Si se acompaña de signos atáxicos de extremidades. de Pregunta 17. pero aparece lateraliza- final (temblor intencional). un Romberg) y los pasos desiguales. pero si dular y espasmódico (asinergia). para pensar en Las lesiones cerebelosas cursan con ataxia. no se encuentran más signos cerebelosos que en la bipedestación Una lesión cerebelosa puede presentar un temblor que no es de y en la marcha. Generalmente se Comentarios TEST dentatorrubrotalámicas. ya que sólo debe considerarse como tal • Las lesiones occipitales que afectan en parte a la porción poste- a la inestabilidad que aparece al cerrar los ojos y desaparece al rior del cuerpo calloso. sólo problemas con el final del movimiento: los pies golpean mente de cuál sea su origen. degeneración subaguda combinada de la médula: operados de que es más característica la atrofia vermiana. fuertemente en el suelo (marcha taloneante). hay que pensar en una atrofia aislada de vermis. Cursa con Romberg positivo. que se acompaña habitualmente de se caracteriza por una aparente debilidad en miembros inferiores: alucinaciones y de anosognosia (el paciente niega que no ve). pen. a lo sumo una disminución del tono muscular que es • Lesiones de corteza visual primaria: son exclusivamente visuales. postural. con lo que puede resultar difícil complejos con alteraciones en la percepción o nominación de los diferenciar esta marcha de la que se presenta en el déficit de sensi. asociada a lesiones occipitales izquierdas). colores.. generar una lesión ocupante de espacio en fosa posterior. puede cursar con un temblor será positivo hacia el mismo lado de la lateralización de la marcha. prosopagnosia con dificultad para reconocer las caras. esto no es un dificultad para ver objetos de forma simultánea (simultanagnosia). La astereognosia o incapacidad para reconocer objetos por me- Considerarla si se refieren trastornos sensitivos asociados (sobre dio del tacto es típico de lesiones de la corteza parietal. sino quietud emocional y mejoran con la tranquilidad. y las correcciones que se hacen suelen ser pensaremos en una lesión cerebelosa completa.. Clasificación y tratamiento del temblor En pacientes con antecedente de alcoholismo se pueden encon. • Ataxia cerebelosa: es una marcha insegura. rección. con visión «en túnel» (hemianopsia con respeto macular bila- Hay que recordar que la marcha atáxica por lesión cerebelosa teral) o con ceguera cortical. NEUROLOGÍA CTO Medicina 1ª Vuelta Prenguta 13. típica- mente en relación con lesión de cordones posteriores medulares. es decir. yor dificultad al intentar caminar con ojos cerrados. reposo (postural o cinético) que característicamente se acentúa al • Ataxia vestibular: la marcha es inestable. El caso de la pregun- ta podría serlo si no es por la descripción de bipedestación ines- Pregunta 16. todo de sensibilidad posicional. El Romberg mesencefálico. anemia macrocítica o megaloblástica). con sensación de giro de objetos. Pregunta 18. en 4 . y temblores de carácter estómago. en fases de deprivación alcohólica. xión: el paciente puede ser capaz de escribir pero no de leer (alexia sin agrafia. menos intensa de la que presentan las lesiones de segunda pueden cursar con cuadrantanopsias o hemianopsia. y con ojos abiertos la marcha no tiene claras lateralizaciones. el momento equivocado. por lo que la lesión de las mismas a nivel acompañará de vértigo. Entran en acción distintos componentes del movimiento en cerrados). pacientes con alteraciones en la vitamina B12 (pensando en una trar temblores de tipo cinético por lesión cerebelosa crónica. Aparecen errores en la amplitud. esto justifica que la respuesta falsa sea la 5.. parece relacionarse con la lesión de vías ción de forma constante hacia el mismo lado. • Ataxia sensitiva: por pérdida de la sensibilidad posicional. inestable con ojos abiertos y cerrados (aunque algo peor con ojos mientos. signo de Romberg positivo. ileítis terminal. El paciente con lesión cerebelosa presenta ma. táctil profunda o vibratoria).R: 2 En reposo no se pueden objetivar alteraciones del movimiento. con pasos desiguales. antecedentes de sífilis (tabes dorsal). la extremidad que avanza lo hace con movimientos erráticos y al • Lesiones de áreas asociativas visuales: incluyen síndrome visuales apoyar puede golpear en el suelo.R: 5 table (que no hace coincidir con el cierre ocular. si son bilate- motoneurona. rales.R: 3 ver los objetos pero no de cogerlos con la mano (desconexión Ante la descripción de un paciente con marcha atáxica tenemos visuomanual) o incapaz de dar nombre a un objeto o a un color que establecer el diagnóstico diferencial básicamente entre unos que se le muestra de forma visual (por desconexión del área visual cuadros concretos: con el área sensitiva del lenguaje). como podría excesivas. Las lesiones en el lóbulo occipital pueden dar lugar a distintos en la valoración del tono muscular pueden no encontrarse signos tipos de clínica según la localización exacta de la lesión: patológicos. leve. mala organi. la fuerza y la frecuencia empleada en los distintos movi. pueden originar síndromes de descone- abrirlos. cerca del núcleo rojo. muy similar (temblor rúbrico) que se diferencia del cerebeloso por asociar un componente de temblor de reposo del que aquél carece. cultad para mantener la mirada fija en un punto (apraxia óptica). aun. la di. Todos los temblores posturales se acentúan en situaciones de in. zación de los movimientos. puedan conllevar trastornos más amplios de la percep. junto al lóbulo parietal y sobre la cisura de Silvio. a lo que a veces se le denomina negligencia: el paciente puede el entorno. de la pregunta) nos obliga a pensar en lesiones infracorticales: si se chas – déficit izquierdo y al revés) e invertido (lesiones inferiores – habla de anestesia. como en el caso de la pregunta. • La descripción del déficit visual siempre hace referencia al área Por el contrario. hay que pensar en el tálamo en los casos de déficit superior y al revés). la lesión táctil superficial –roces-. dolor. lo que no corresponde con ra o dolor o forma. ganando de forma progresiva a medida que nos acercamos a la En los pacientes en los que la principal alteración es la compren- corteza occipital. de modo que un defecto plenamente congruente sión de las órdenes que se les pide verbalmente que ejecuten. Defectos campimétricos y lesiones de la vía óptica paraquiasmástica 5 . De ahí que una exploración sensitiva completa este diagnóstico. La corteza sensitiva del lóbulo parietal es la encargada de recibir Algunos pacientes.. pero más que las de paña de un lenguaje fluente (indicando que no hay afectación de cintillas ópticas. si además se acom- son menos congruentes que las occipitales. cualquier referencia a alteraciones en una sensi- que el paciente deja de ver en su campo visual. Estos síntomas son más frecuentes en las lesio- visual: nes no dominantes (derechas). Estas áreas se extienden más allá de las nimos. hemianestesia completa o en lesiones medulares en casos de aneste- • La homonimia y la heteronimia hacen referencia a si el defecto en sias con un nivel bilateral. y los primeros la información so- bre las vías sensitivas.R: 2 da: defecto homónimo. Si al cerrar un ojo el defecto es superior La localización del lenguaje dentro del hemisferio izquierdo (en izquierdo y al cerrar el ojo contralateral. ción global de la sensibilidad más allá del dato aislado de temperatu. siendo estos últimos los que suponen con los del lenguaje oral. si fluente. La congruencia entre los campos visuales de ambos ojos se va la lesión es sólo de áreas anteriores: afasia motora. esto supone que bilidad concreta (táctil superficial. a pesar de ser fluente. dolorosa en el caso el defecto que lo origina será siempre contralateral (lesiones dere. vibratoria. confusional.. se sugiere una lesión cortical. dicho cuadro es la inatención. no hacer caso a agitación. Pregunta 21. Aunque el déficit visual coincida en ambos ojos. e indica lesión en áreas anteriores del lenguaje. habla fluida con parafasias. pre es idéntico: si el área que se pierde en el campo superior En los pacientes en los que está alterada la producción del lengua- izquierdo (en el caso de la pregunta) es mayor en un ojo que en je. tienen. reconocer un dibujo hecho sobre la piel. aunque tienden a ser más graves. suele afectar a la circunvolución supramarginal. sería la encargada de la percep. tenemos una afasia sensitiva. un objeto. reconocer va. posición) y el análisis de datos combinados (reconocer de Silvio. la pérdida de visión está el que se encuentran las funciones del lenguaje en la mayor parte de en otro cuadrante. la población) no es de una certeza anatómica indiscutible. será ininteligible. con importantes altera- Pregunta 20. táctil profundo –toques-. pero la comprensión está conservada.R: 5 Otras funciones perceptivas del lóbulo parietal hacen que sus La descripción del caso clínico es la característica de un cuadro lesiones. o con cierta disminución del nivel de conciencia. con una emisión de palabras escasa o nula se habla de afasia no otro.. siempre que no tenga que responder verbalmente. Puede presentar un cuadro de exceso de actividad. con lo que hablar de dan. tenemos una cuadrantanopsia homónima incongruente. al final de la cisura ra. presentan un los estímulos sensitivos combinados. si el mismo el defecto es idéntico. áreas anteriores del lenguaje). clásicas áreas de Wernicke y de Broca respectivamente. cierra el ojo izquierdo tiene pérdida de visión superior izquierda y si cierra el ojo derecho tiene pérdida de visión superior izquier. con Comentarios TEST no reconocer el hemicuerpo afectado como suyo. más allá del trastorno concreto del reconocimiento de da. es un defecto heterónimo. indica que congruencia o incongruencia no tiene sentido. • La congruencia e incongruencia tienen valor en los defectos homó. en el caso de la pregunta. como el paciente con afasia sensitiva. los déficits en el lenguaje escrito suelen coincidir rios estímulos simultáneos). del lóbulo frontal. es congruente. describe un paciente que si aislada de la sensibilidad termoalgésica (vía espinotalámica). el área anterior del lenguaje. Se sitúa Los defectos heterónimos son típicos de las lesiones quiasmáticas en torno a la cisura de Silvio: áreas temporales posterosuperiores (o de un paciente con dos lesiones distintas) mientras que los perisilvianas serían el área posterior del habla y zonas posterobasales déficits homónimos son siempre retroquiasmáticos. NEUROLOGÍA 1ª Vuelta CTO Medicina Pregunta 19. El len- guaje.R: 2 estímulos que vienen del lado afectado e incluso no ser claramente Conviene tener claros algunos conceptos en cuanto al campo conscientes del déficit. Lo que determina fundamentalmente la existencia de tos aislados. que habla de «anestesia el campo visual coincide o no en ambos ojos en la misma parte dolorosa» podríamos pensar en una lesión talámica o en una lesión del campo. vibración. En la opción 4. Las alteraciones campimé- paciente conserva la capacidad de comprender órdenes que se le tricas heterónimas no son superponibles. no siem. etc. temperatu. la baja interacción del paciente con ción. las lesiones de las radiaciones ópticas sospecha lesión de áreas posteriores del lenguaje.. En las afasias. una exploración cortical parietal. sin embargo importantes alteraciones de la deba incluir el análisis de datos concretos (recepción de estímulo repetición: a esta afasia se la denomina de conducción. Pregunta 19. Pregunta 22.R: 4 ciones semánticas y sintácticas. cuya definición co. Pregunta 22. dado que por mente con la dificultad de contacto con el explorador. conciencia esté relativamente conservado. descritas en la tabla. como está descrito en la opción 1. siendo la más frecuente los trastornos En el caso de la pregunta. puede no encontrarse ningún tipo ner nuevos datos no indican que el paciente tenga un deterioro de de movimiento. para distinguir la decorticación de la descerebración. sudoración. nes de las vías motoras (típicamente protuberanciales: síndrome de El cuadro confusional es una clínica común a una infinidad de cautiverio). incluyendo la memoria. con lo que la lo que respecta a los miembros inferiores son iguales. existiendo una hipotonía generalizada. NEUROLOGÍA CTO Medicina 1ª Vuelta El resto de los síntomas en realidad pueden relacionarse directa. el cálcu. Una lesión ocupante de espacio en la fosa posterior puede causar una compresión de tronco de forma directa o indirecta (herniaciones). tante profundo (respuesta 5 falsa) con alteraciones de los reflejos y las rresponde a un cuadro confusional con hiperactividad vegetativa funciones del tronco (falsas la 2 y la 3) y alteraciones en el patrón (taquicardia. Si el coma es muy profundo. En casos en funciones superiores de base. desorientación. el lenguaje incoherente y la incapacidad para rete. pero el nivel de de procesos mentales. sino que está alterada la elaboración los que no haya respuestas motoras a los estímulos.. etiologías. esperaríamos encontrar una situación de coma bas- Consideración aparte merece el delirium.R: 2 123456478749 9. hipertensión arterial) y cuyo representante respiratorio que no corresponderían a un ritmo de Cheyne-Stokes típico es el delirium tremens por deprivación alcohólica. descerebración. Causas de cuadro confusional Pregunta 25. hay que sospechar lesio- lo y la orientación. en el que se describe una postura de tóxico-metabólicos. el lenguaje. . 5. 89 589529 2459. 9 5. 9. 9 Las desviaciones forzadas de la mirada son típicas de dos localiza- 9. . 89 123456478749 245. 89 ciones distintas: la frontal y la protuberancial. no resultando difícil 123456789 26 . 8567 768 92826 764 2 7882 86 76 9 distinguir entre ambas: 222 6. 6789 • La lesión frontal cursa con una desviación oculocefálica hacia el 12 789 2 . 6786  67892 . 69689278. 67869 lado del lóbulo frontal lesionado (con una lesión frontal izquier- 12!696 2769692 76 9278" 929. 89289. 7#. 6789 da el paciente viene mirando hacia la izquierda). lo que suele 12$ 6 8 7% 6. 69 corresponder con el lado contrario de la hemiparesia (una lesión 12&978 9 764 22' 2 76 26 766 .  frontal izquierda cursa con una hemiparesia derecha). En resu- 12(8 6 927 929'  7 86 29''  men: el paciente con lesión frontal viene mirando hacia el hemi- 12) % . 82. 8 2782. 88* 6. 26+'68282687766.  cuerpo que puede mover. • La lesión protuberancial cursa con desviación ocular (más que 123456478749 9. 96. 969 tuberancial derecha presenta desviación ocular hacia la izquier- 12(68 '76 2 .89 oculocefálica) hacia el lado contrario a la lesión (una lesión pro- 12.  768 92687. 8#. lo que suele corresponder con el hemicuerpo deficitario (la 12) .67 9 da). disfagia). no es muy compatible con una lesión da). las ancianos. mediana edad o ancianos habituados al consumo de alcohol (vino). caracterizada por signos de pa- taria sin alteración del nivel de conciencia. ocu. con lo nico o cianuro. ción de la vía piramidal bilateral. lo mo. Suele haber una cierta lesiones frontales pueden asociar otros déficits (afasia motora en he- hipotonía. paraparesia o tetraparesia. dado que un daño a este nivel.R: 4 estados de hiponatremia. trastornos del ritmo respiratorio y con frecuencia re. Bignami es una degeneración primaria del cuerpo calloso. e importante alteración del nivel de conciencia. pueden generar una situación de coma. que puede ser mortal en pocas horas. En resumen: el paciente con lesión protu- El coma es una situación de mala función cerebral global. infarto (trombosis de la basilar). que puede cursar con hemiparesia. El nivel berancial viene mirando hacia el hemicuerpo parético. que aunque no muy correcto sugiere un paciente estabi. La mielinólisis central pontina es una enfermedad afecta a los núcleos respiratorios.. la distribución del déficit generalmente es de En las lesiones difusas corticales. Los únicos semiológicamente: pueden tener alteraciones pupilares. oculocefálicos. nivel. a cualquier toma más frecuente es la demencia. que sirve inespecífica. ya que se decusará en el bulbo raquídeo: una lesión protuberancial derecha cursa con una he- Pregunta 23. las funciones y reflejos del tronco cerebral están conservadas. en el que ya no hay SRA. Excepción notable es la hipoglucemia severa en tuberanciales asociarán probablemente alteraciones pupilares. mientras que la frontal no tiene por qué alterar el o multifocal de la corteza o una lesión directa de la SRA. alta.8567 768 9 lesión de la vía piramidal a su paso por la protuberancia genera una hemiparesia contralateral. periores. Ge- neralmente aparece 2-6 días después de la corrección rápida de Pregunta 24. Tiene un pobre pro- aparecen en lesiones del SNC. el término «estado El Síndrome de “locked-in” o cautiverio puede ser secundario a comatoso». predominio faciobraquial en las lesiones frontales.. y los verticales de la mirada. la tónica es la ausencia de foca. el Síndrome de Wallenberg. hiponatremia rápidamente corregida. No siempre permiten la localización nóstico y no hay tratamiento efectivo. La causa más frecuente es una lovestibulares). conservando el parpadeo y los movimientos oculares verticales. La alteración anatomopatológica flexión de miembros superiores. (zona medial. Los síntomas mentales están casi siempre presentes y 6 . Cursa con tetraplejia y afectación de la motilidad ocular hori- que produce es una alteración primaria del ritmo respiratorio. ojos. Si la lesión no parpadeo. La enfermedad de Marchiafava- acción de descerebración (extensora) a estímulos dolorosos. deo. con adducción. tumor. Cuando encontramos a un paciente en entre ambas: la lesión protuberancial es probable que deje al pa- estado de coma hay que pensar en un trastorno difuso generalizado ciente en coma. Aparece en lesiones mesencefálicas o protuberanciales altas. hemorragia o traumatis- bulbar. También aparece en pacientes desnutridos y se desconoce si su La decorticación cursa con extensión de miembros inferiores y patogenia es tóxica o metabólica. las lesiones pro- lidad neurológica. Se trata de una mielinólisis central pontina. Encontramos afecta- presores del SNC) o hiperventilatorio (acidosis). y el patrón respiratorio es muy variable: puede ser normal. pero sí pueden ser útiles en el Bignami. La presentación más frecuente es la demencia que asumimos que el dato más importante es la flexión. mielinólisis central pontina (hiponatremia rápidamente recupera- lizado en situación de coma. alteración movimientos que tienen respetados estos pacientes son los de parpa- o abolición de los reflejos del tronco (corneales. de tipo Pregunta 26. con bordes dorsal y ventral conservados). Sólo conservan la motilidad ocular en el plano vertical y el mente inestable. con hipoventilación en algunas intoxicaciones (de. arsé- hablan de hiperpronación asociada y otros de supinación. pero también se ha descrito asociada a Las respuestas motoras anómalas son movimientos reflejos que alcoholismo crónico y a trasplante hepático. puede haber movimientos horizontales pendulares de los misferio dominante). cuyo sín- Aunque todas las lesiones del tronco del encéfalo. referida en la exploración del pa- Comentarios TEST Las lesiones del tronco del encéfalo son muy ricas ciente. con hiperpronación de las su. algunos autores es similar a la encontrada en la toxicidad por alcohol metílico. desmielinizante del tronco cerebral. La enfermedad de Marchiafava- exacta de la lesión que las origina. como puede suceder en rálisis pseudobulbar (disartria. zontal. nivel de conciencia. que se pre- La descerebración o postura extensora cursa con extensión y sentó inicialmente con especial frecuencia en varones italianos de adducción de las cuatro extremidades..R: 5 miparesia izquierda). consiste en una degeneración primaria del cuerpo calloso abordaje diagnóstico. el paciente tendrá focalidad defici. de conciencia depende del normal funcionamiento de la sustancia reticular activadora (SRA) troncoencefálica y su proyección difusa a Por supuesto hay otra serie de datos que nos ayudan a diferenciar toda la corteza cerebral.R: 3 Cheyne-Stokes. Tiene una que conocemos como encefalopatías. que puede provocar una lesión de en ese orden y que tras la terapia de reposición el orden de recupera- sustancia blanca protuberancial. sión. esto troncoencefálica. oculares horizontales conjugados tienen su área de coordina- ción en la protuberancia..R: 4 hematíes (encefalitis necrohemorrágica). Un EEG con actividad focal hiperventilación rítmica nos debe hacer pensar en una encefalopa. asfixia. cefalea. mientras que los movimientos oculares afectación difusa de la corteza. puede ser normal en fases muy iniciales o cuando el paciente está inmunodeprimido. En algunos casos hay un aumento en la cifra de Pregunta 28. Pregunta 29.R: 1 etc. Respuestas pupilares características de las lesiones encefálicas. De forma típica.R: 3 Las lesiones que pueden dar esta clínica pueden ser de múltiples La tríada de oftalmoplejia externa (alteraciones de la motilidad del etiologías. aunque no todas las zonas del encéfalo sufren Pregunta 27. la gluconeogénesis impide pares. por aumento de sensibili. 7 . boca seca. ción de los síntomas también es el mismo: primero la oftalmoparesia. Las áreas cerebrales lesionadas no tienen una regeneración en lo que respecta a tejido neuronal: se produce una cicatriz glial que puede dar lugar a crisis epilépticas tardías. síntomas de focalidad neurológica. que se instaura de forma subaguda. se trate o no de un diabético conocido en tratamiento (pero El síndrome de cautiverio está originado por una lesión desmieli. globo ocular sin afectación pupilar o de la acomodación). que cursa con Cursa con fiebre. pero es la acidosis metabólica de cualquier origen la que se hiperproteinorraquia y glucosa normal en el LCR. midriasis y agitación. comenzando a producirse lesiones irreversibles tras escasos minutos de hipoxia. Una RM cerebral puede mos- trar desmielinización difusa protuberancial. A nivel mesencefálico se origina también el movimiento que se llegue a esa situación de forma fisiológica. y éstas aparecen en los días si- . donde se sitúan los III dado que en un organismo sin patología. una hipoglucemia lo suficientemente mantenida como para produ- naza) o incluso que es capaz de mantener una conversación si cir una situación de coma.. en el que se producen los • Estado alerta: esto implica que es capaz de interaccionar con el cambios metabólicos suficientes para permitir que se consuman cuer- entorno. en el que. con secuelas como deterioro cognitivo. Es la posibilidad de sufrir un daño cerebral irreversible. rada en el plano vertical. sin afectar al nivel de conciencia. a pesar de ser horizontal. Es exclusiva de diabéticos tratados. incluyendo la idiopática. de oscilar desde el estado confusional al coma profundo y signos y La hiperventilación rítmica es un signo clásico del coma acidótico. denominándose respiración de Kussmaul. importante morbimortalidad. especialmente si se conoce este antecedente) una de las primeras me- nizante que afecta a las vías motoras descendentes de la protuberan. órgano selectivo y sólo utiliza glucosa como fuente de energía. ya que se alteran en lesiones del incompleta y le quede alguna secuela tras una hipoxia cerebral es tronco del encéfalo y aportan información de dónde puede estar más frecuente que presente ataxia. de ondas agudas periódicas de predominio en región temporal – tía. La respuesta es que Ante un paciente en coma del que no podemos obtener cifras de tiene un síndrome de cautiverio.R: 5 en la misma medida. tornos motores extrapiramidales. frontal.. Este dato induce a pensar en una censo de los niveles de glucemia. con adducción de ambos ojos en condiciones de ayuno prolongado. por muy rápidamente que se produzca el des- nado número de parpadeos. lo LCR por reacción en cadena de la polimerasa (PCR). probablemente significa que sigue objetos con la mi. La teoría dice que la tríada se instaura rregida demasiado rápidamente. signos meníngeos y. lo que es más ca- ritmo regular de hiperventilación-apnea. el cuerpo tiene una primera reac- conservación de la sustancia reticular activadora (SRA) ción de stress que consiste en la liberación de catecolaminas. la exploración de las pupilas y su opción 3. NEUROLOGÍA 1ª Vuelta CTO Medicina tienen características de estados maníacos. cia. vas indica que el tronco cerebral del paciente permanece sin lesión.R: 4 III pares para los movimientos de addución (por los fascículos El coma hipoglucémico es una encefalopatía metabólica. alteraciones de memoria y tras- situada la lesión. lleva a un cuadro clínico hiperadrenérgico. Las áreas más sensibles son las descritas en la Ante un paciente comatoso. ataxia y ciente que ha presentado antes un cuadro confusional que ha sido cuadro confusional es la típica de la encefalopatía de Wernicke por atendido médicamente debe hacer pensar en una hiponatremia co. férica profunda es la respiración de Cheyne-Stokes. de tal manera que un paciente que se recupere de forma reactividad es especialmente útil. glucemia. y desde allí se dirigen las órdenes a los Pregunta 31. sólo inter. se le da la oportunidad de contestar “sí” o “no” con un determi. paranoides. posteriormente la ataxia y por último el cuadro confusional. pero el antecedente de una pa. que está situada en el tronco del encéfalo.. como la disfasia. apraxias o hemiparesias. guientes. generalmente en minutos-horas (de- de la protuberancia como límite superior puede no tener lesión de la pendiendo del ritmo de acción de la terapia hipoglucemiante). que es seguido de la dismi- Cómo un paciente con un déficit motor que incluye la parte alta nución del nivel de conciencia. debido a una desmielinización de las áreas afectadas. deterioro del nivel de conciencia que pue- dad de los centros inspiratorios a la concentración de CO2. déficit de vitamina B1 (tiamina). La existencia de pupilas isocóricas y normorreacti. La hipoxia cerebral puede producirse por cualquier causa que impida la llegada de flujo sanguíneo oxigenado al cerebro. con longitudinales mediales). recu- peran aparentemente sin secuelas. SRA. racterístico. Comentarios TEST verticales se organizan en el mesencéfalo. Las convulsiones son bastante frecuentes y se suelen asociar a El cerebro es el tejido con mayor sensibilidad a la falta de perfu- afasias. Pregunta 32. que se produce cierre palpebral ante Tanto los antidiabéticos orales como la insulina pueden producir la aproximación de un objeto al campo visual (reflejo de ame. de encefalitis. El cerebro es un de convergencia. no es exclusiva de la aci. El método de laboratorio más sensible y específico para el diag- ción del tronco cerebral. Pregunta 27. salvo vienen los terceros pares. En la TC la encefalitis La descripción del cuadro clínico es lo que nos debe llevar la herpética puede dar lugar a zonas de hipodensidad y efecto de masa máxima atención: en regiones frontotemporales que pueden captar contraste. lo que obliga a realizar desde el primer momento de una parada cardiaca manio- bras de resucitación. porque si se recupera flujo pasados 10-15 mi- nutos. depresivos. con sudoración. didas a tomar es la administración de glucosa por vía endovenosa. La celularidad relaciona típicamente con ella. El cuadro denominado encefalopatía hipóxica retardada se pre- senta en pacientes que. Pregunta 30. El hecho de que este paciente presente un patrón respiratorio de trastornos de la personalidad o disfasia. pos cetónicos. o compresión vascular. Típicamente hay pleocitosis linfocitaria (>95% de los casos) con dosis. es otro dato más que apoya el diagnóstico. taqui- cardia. tras sufrir el proceso de hipoxia aguda. el paciente probablemente permanezca en situación de coma. La RM es • Incapacidad para realizar cualquier movimiento voluntario sal. la técnica radiológica de elección para detectar cambios de señal en vo parpadeo y movimientos oculares verticales: los movimientos el parénquima cerebral en las encefalitis.. ya que el patrón más frecuentemente asociado a la lesión hemis. nos queda la posibilidad de una lesión nóstico de la encefalitis herpética es la detección de ADN vírico en el hemisférica profunda o una afectación difusa de toda la corteza.R: 4 Ante un paciente que presenta una situación de coma sin afecta.. sea por parada cardiaca de cualquier origen. . cuando es preciso). demencia cortical y subcortical Aunque hay distintas clasificaciones. por La memoria es la capacidad para almacenar datos (y rescatarlos el contrario. AMNESIAS. El enunciado de la pregunta describe una demencia subcortical. y se van en un paciente con sospecha de Wernicke antes de la tiamina. nica cerebral definida. es práctico recordar algunos datos básicos: 1232456789 5. de la vitamina por vía intramuscular es prácticamente inmediata. parenteral sobre la enteral. con presencia de signos de malnutrición o de alcoholismo previos a la conservación del nivel de conciencia y en ausencia de una lesión orgá- presentación de la nueva clínica. afasia. DEMENCIAS. es la administración de un combinado polivitamínico. apraxias y agnosias en au- que puede desencadenar o empeorar la encefalopatía. Se quedan por tanto fuera de esta definición los trastornos de nistración de 100 mg de tiamina por vía parenteral. sencia de claros déficits neurológicos focales. acalculia. la disponibilidad alguna función intelectual aislada (memoria. Diagnóstico diferencial entre tantes alteraciones de la memoria. dado que estamos hablando con un Una vez diagnosticado el paciente de demencia. NEUROLOGÍA CTO Medicina 1ª Vuelta En la práctica lo que nos interesa es no dejar pasar ninguna ence. Pregunta 35. ya que la caren. • Subcorticales: hay fundamentalmente un enlentecimiento global cia de otras vitaminas también puede producir síntomas neurológicos. Habrá que estar atentos a la tación de distintas áreas) de las funciones intelectuales previas. Pregunta 33. mencias: Hay que intentar evitar la administración de glucosa intravenosa • Corticales: el síntoma inicial es la pérdida de memoria. Ante la sospecha de síndrome de Wernicke está indicada la admi. de los procesos mentales. los teniendo en principio claras ventajas la intravenosa. pero sí la vía comas y los procesos secundarios (encefalopatías). ya desarrollando progresivamente afasia. no los defectos en la adquisición de dichas funciones (oligofrenias). sin claras alteraciones corticales (afasia. La definición de demencia incluye un deterioro global (con afec- falopatía de Wernicke sin sospecharla. El prototipo es la Lo más indicado en un paciente con déficit nutricional combinado enfermedad de Alzheimer. que es. apraxia.R: 3 entre las cuales no se encuentra la enfermedad de Alzheimer. En principio se puede asumir que es una función repartida de forma difusa por todo el cerebro. agnosia) y con fallos de memoria inicialmente menos llamativos. etc). también existen paciente malnutrido que probablemente asociará problemas de varios subtipos: la principal diferencia a recordar está entre las de- malabsorción digestiva. la demencia cortical por excelencia (y por frecuencia). aunque hay algunas áreas (hipocampo-cerebro basal) cuyas lesiones provocan impor. 657 1232456785. 657 • Memoria inmediata: se valora pidiendo al paciente que repita lo que se le dice. depende sobre todo de la atención. en pacientes 424967 123245465728979 . 4 8932 con alteraciones importantes del resto de funciones mnésicas. la  . 529. 85. 466. 425 5869 memoria inmediata se conserva si mantienen un buen nivel de conciencia. 16569 87794724626589472 62. 9499. 949899 • Memoria reciente: se valora pidiendo al paciente que retenga 42 . 965. 65. 9 54859 9 5. mientras se le encarga alguna 25667 9947282.9 algún dato durante varios minutos.  282. 48222 otra tarea. y pidiéndole que lo repita más tarde. • Memoria remota: se le pide al paciente que aporte datos de hace 4. 379 . 98894. 78562 54. 9354249. 2. 492. biográficos o históricos. años. 3. . 29 82685 8 75 95 Otra clasificación dividiría las amnesias en retrógrada. cuando se 296. 429 !. 897544 965 "582#949. Su estudio no está indicado Comentarios TEST • Aparición brusca y duración menor de 24 horas de rutina en el anciano con deterioro cognitivo en nuestro medio. su componente principal son proteínas fosforiladas. Lo proteína tau. no está afásico ni confuso. pero no olvida La enfermedad de Alzheimer es la causa más frecuente de de- quién es. puede realizar acciones com.. el EEG realizado generados.5939 olvidan datos que previamente se conocían. en su aparición. que más parece correlacionarse con el grado de demencia. • El paciente suele repetir las mismas preguntas. central de amiloide (placas “maduras”). fuera de los tratamientos globales para enfer. disartria. antes ni después de ictus que la población de su mismo rango de Las lesiones histológicas básicas son: edad. 8 . Su etiología es hasta hoy descono- plejas (vestirse.R: 3 Las lesiones anatomopatológicas básicas no son patognomónicas Hay que recordar que las apraxias. Pruebas complementarias funcionales. y la densidad de las lesiones lo que es diagnóstico. placas y en las paredes de los vasos de pequeño calibre cerebrales. La intensidad de esta degeneración es lo ciente no tiene otro tipo de crisis epilépticas). tales como el SPECT. Se encuentra numerosa Tratamiento no tiene. durante la crisis no muestra alteraciones que la justifiquen y el pa. lo que no se correspondería con ataque isquémico transitorio) • La degeneración neurofibrilar: acúmulos de neurofilamentos de- o epiléptico (pero la duración de las crisis es menor. por lo que en realidad no se alcanza “in vivo”. cuando el paciente no es capaz de retener nueva información.. a diferencia de otros síndromes (pérdida de En la Enfermedad de Alzheimer se desarrolla un déficit de todos fuerza. es el gen que tener claro que el cuadro típico tiene las siguientes características: de la preselinina I en el cromosoma 14. ten a la corteza hemisférica. Es necesario realizar un diagnóstico dife- rencial con otras causas de demencia como la vascular. las microgliales que con el paso del tiempo tienen un depósito ingesta de fármacos. Pregunta 34. siendo la causa más frecuente en nuestro medio la medades degenerativas que se acompañen de amnesias (demencias). y anterógrada.. mencia en el mundo occidental. El factor desencadenante del episodio puede ser muy variable: • Las placas neuríticas: acúmulo de neuronas distróficas con célu- esfuerzos físicos intensos. conducir) sin aparente dificultad cida. y es sólo su hallazgo de forma difusa síndromes clínicos que se presentan únicamente en procesos que afec. coito. Se desconoce la etiología de la amnesia global transitoria. habiéndose postulado numerosos factores que podrían influir • Predomina el defecto amnésico anterógrado: el paciente no re. anopsias) que pueden ser clínicamente comunes pese a los neurotransmisores. Se trata de un deterioro cognitivo progresivo con datos de afecta- cits aislados de memoria. diagnóstico de certeza.R: 4 Lo cierto es que actualmente no existe un tratamiento para los défi. que sí se conoce es el buen pronóstico: al paciente le queda como • Depósito de amiloide: el amiloide se forma a partir de una proteí- secuela una laguna de memoria correspondiente al tiempo de la na codificada en el cromosoma 21. El locus cromosómico cuya mutación se asocia con La amnesia global transitoria es lo que presenta el paciente.. Se ha que muestren una hipoperfusión frontotemporal pueden apoyar el postulado origen isquémico (pero el paciente no presenta más riesgo diagnóstico clínico. pero la incidencia de recurrencias es baja.. Pregunta 36. ataque de migraña. se deposita en el centro de las crisis.. puede presentar alguna Los hallazgos anatomopatológicos son los que proporcionan el laguna retrógrada. si bien los sistemas colinérgicos parecen los poder estar causados por lesiones de distintas localizaciones. Pregunta 35. metabólica. estrés emocional. enfermedad de Alzheimer. tiene datos durante más de 1-2 minutos. puesto que se desconoce la etiología. las agnosias y las afasias son de la Enfermedad de Alzheimer. Hay mayor frecuencia a los casos de Alzheimer de inicio precoz.R: 2 carencial. ción cortical. ubiquitina y amiloide. menos llamativa. cortical. cuan. que generalmente se producen a nivel digestivo. que puede mostrar un aumento ines- efectos secundarios. incapaz de extraer ese nombre de su memoria. hipertensión. En algunas familias. sino también por la pérdida de receptores. pero en este caso no sólo por la reducción presináptica • Trastorno de la marcha: aunque no está especificado el tipo de de neurotransmisor. Una modificación en la estructura terciaria de la proteína normal Las apraxias pueden describirse como que el paciente tiene difi. te generalmente producida en relación con complicaciones respira- torias. urgencia miccional. suele ser de características subcorticales.R: 1 permanece desconocido. Su código gené- una alteración establecida.R: 5 do asocian demencia. dado que ha sido el único mecanismo farmacológico que ha demostrado eficacia terapéutica ”in vivo”. aunque el cuadro suele cios tangibles frente a los potenciales efectos secundarios. no deje de la que aún no se conoce claramente su función. La conseguir el contacto con la proteína priónica. cluso esa memoria remota se altera y se pierde definitivamente. NEUROLOGÍA 1ª Vuelta CTO Medicina más afectados. de origen carencial. con el EEG compatible. mientras que el paciente aún es familiar fatal y alguna variante familiar de la enfermedad de capaz de recordar datos biográficos antiguos. re demencia incipiente (otra cosa sería un olvido global de forma • Y leucoaraiosis bilateral extensa en las pruebas de imagen. una demencia con trastorno de la marcha de tipo apráxico ayudaría a pensar en una demencia cortical.. No existe tratamiento para esta enfermedad. 9 . De hecho. Pregunta 38. sonalidad previa. Comentarios TEST Las demencias corticales se diferencian de las subcorticales en el La enfermedad de Strachan o neuritis jamaicana es una enferme- contexto clínico por una mayor alteración de las funciones “supe.. cálculo. con el hallaz- El diagnóstico de demencia se establece cuando existe afectación go de una espongiosis con escasa actividad inflamatoria. • Olvidos frecuentes de forma progresiva en paciente mayor: sugie. (sin cambios en la codificación genética) puede hacer que dicha pro- cultad para vestirse. ya que en su diagnóstico incluye que exis- La enfermedad de Pick es una demencia muy similar clínicamen. de forma progresiva. Hay. influyen en la enfermedad: formación de depósitos amiloides. manejar un mando a distancia). estas son las variantes afasia inicialmente es nominal: el paciente tiene un lenguaje espon. En el LCR. una llave). tomáticamente en una enfermedad de Creutzfeldt-Jakob. Esta modificación la puede rostros de los familiares.. (con dolor en extremidades y abolición de reflejos). diagnosticado de encefalopatía isquémica subcortical. las pruebas complementarias aumentarse de forma creciente. en especial y en fases iniciales se altera más son la enfermedad de Gerstmann-Straussler-Scheinker. se detecta la proteína 14-3-3. para realizar acciones simples que antes sabía teína se agregue en forma de un depósito amiloide resistente a la acti- (abrir una puerta con una llave. mente es transitorio por una causa que. una vez corregida. una mutación en el cromosoma 20 codifica corticales. mundo posee la proteína precursora de la proteína priónica (PrP). extrapiramidales. con muer- neurotransmisores. De modo que un paciente que presente: • Deterioro de características subcorticales con parkinsonismo. infecciosas: las variantes adquiridas de la enfermedad de C-J clásica. tamiento de la Enfermedad de Alzheimer. fenómenos oxidativos.R: 3 proceso.. aunque lo haya El mecanismo por el que la proteína consigue llegar hasta el siste- usado de forma espontánea. piramidales. y lo La enfermedad de Binswanger en realidad debería diagnosticarse mismo sucede con las demencias de origen vascular. muchas situaciones clínicas en las que resulta muy difí. disminu- no obstante. dad de carácter adquirido. La primera premisa para responder la pregunta es descifrar el • Factores de riesgo arteriosclerótico (hipertensión arterial. con lo que se produce la En el caso de las demencias corticales. deterioro de funciones superiores. dado que en fases avanzadas el deterioro es tan acusado que una la tríada típica es una demencia con mioclonía y un EEG caracterís- potenciación de la actividad colinérgica no consigue unos benefi. pero cuando se le pide que nombre alguna cosa es nueva variante del CJ y el kuru. Pregunta 42. lenguaje. básicamente En fases evolucionadas se produce un deterioro global. con la aparición de En la práctica. de forma anómala la proteína precursora. ataxia. diabe- cuadro clínico: tes. las alteraciones del comporta. más selectiva que en el Alzheimer. La dosis debe coherente. ponder a ninguno de los síndromes carenciales clásicos por déficit mientras que las demencias subcorticales conservan esas funciones.R: 1 El diagnóstico de certeza es el anatomopatológico. incontinencia de esfínteres. ma nervioso central y al propio interior de las células hasta conseguir la transformación de las proteínas precursoras en proteínas priónicas Pregunta 39. generando escasa actividad inflamatoria y una agnosias pueden ser evidentes en la incapacidad para reconocer los destrucción de características espongiformes. Una demencia progresiva con mioclonías debe hacer pensar au- Están indicados en las fases leve y moderada de la enfermedad. El diagnóstico de esta enfermedad se basa en el contexto clínico miento y la capacidad funcional global del sujeto. Todo el funciones superiores. trastornos de la marcha. Las vidad de las proteasas. mientras Pregunta 37. junto con nes de características “frontales”: desinhibición. que presentan en las pruebas de imagen una altera- En las pruebas de imagen aparece una atrofia selectiva frontal y ción en la densidad o la señal de la sustancia blanca que sugiere temporal inferior. lipohialinosis y esclerosis de arteriolas. principalmente tendremos enfermedad. en la que predominan las alteracio.. cal y la presencia de una prueba de imagen (TC o RM) sugerente. el insomnio la memoria reciente o de fijación. y con conservación del nivel de conciencia.R: 4 gidez. corticales y sub. sin que existan enfermedades subyacentes que puedan influir en ese Pregunta 41. pelagra). únicamente post mortem. trastorno. u objetos cotidianos (un reloj. con debut generalmente en la vida adulta (5ª década): problemas de memoria. otros El curso clínico es fatal en el transcurso de 6-12 meses. Todos comparten el meca. Se trata de pacientes con factores de riesgo vascular. ta una desmielinización difusa subcortical con ausencia de infartos te a la enfermedad de Alzheimer. dependiendo de la tolerancia de los suelen ser negativas. pero que aparece en la mayor parte intentado actuar sobre los distintos mecanismos que en principio de los casos. • Infartos lacunares previos. la táneo fluente. que no es Otras terapias no han demostrado eficacia. tico está en el cromosoma 20. ri- Pregunta 40. ante un paciente con deterioro cognitivo subcorti- neuronas degeneradas (células de Pick) o con inclusiones intraneu. lenguaje verborreico. Esto es válido para todos los tipos de demencia. completarse con signos cerebelosos. tico con las denominadas ondas trifásicas. a pesar de que se ha específica ni sensible al 100%.. de una vitamina (beri-beri. alteración de la per. se denomine enferme- neurofibrilar del Alzheimer (gránulos de Pick). aunque sin corres- riores”: memoria. En fases avanzadas in. Si no existe un Hay que recordar que los priones son partículas de naturaleza buen nivel de conciencia no se puede evaluar la afectación de las proteica y que se comportan como agentes infecciosos. y no es posible conocer si el deterioro simple. dad de Binswanger o no. ción de visión y audición. cil diferenciar entre ambas entidades. pecífico de las proteínas. Obtienen mejoría en todos los aspectos de la enfermedad: el crisis epilépticas y alteraciones de segunda neurona motora. Por lo general. cerebral. • Sacudidas musculares breves y arrítmicas: definición de mioclo- nismo de acción de ser inhibidores de la acetilcolinesterasa a nivel nías. extensos o de una gran número de infartos lacunares. similares a los de la degeneración considera que estamos ante el mismo cuadro. los trastornos denominados extrapiramidales. ¿desencadenadas por sobresaltos? mioclonías reflejas. queda brusca: amnesia global transitoria).R: 2 que una marcha a pequeños pasos apunta más a patología sub- Hoy existen en el mercado fármacos comercializados para el tra. La anatomía patológica es la diagnóstica. razón. etc. y que responde a la terapia polivitamínica. acciones motoras complejas. de distintas áreas de funcionamiento cerebral. isquemia coronaria). sin que se sepa la desmielinización y que se denomina leucoaraiosis. Creutzfeldt-Jakob. se ronales de filamentos anómalos. que presenta polineuropatía en el contexto de un funcionamiento globalmente enlentecido. es posible que haya habido una lesión quirúr. sino a un nivel inferior. pero probablemente intervenido. En la Pregunta 45. lo que inicial. pero a lo largo de la circunvolución motora que se necesite un estímulo especial para desencadenarlas. Hay que recordar que el síndrome de West debuta más frecuen- temente entre los 3 y los 6 meses y siempre en el primer año. El hecho de que se desplacen o extiendan a lo largo mejor. o ser ya parciales tónicas (se detalla que hay alteración del nivel de concien- parte de la crisis parcial compleja.. puesto que pensamos en un tumor. que ser “urgente” si lo entendemos como sinónimo de “inmediata”. la etiología subyacente no es determinante en el lógicamente. como ausencias. no es infrecuente que algunos Pregunta 43. corta duración (segundos. medio. El diagnóstico más da la realización de una prueba de imagen. de las cuales la más frecuente es la metástasis de un tumor valora datos clínicos que apoyen más un tipo u otro de demencia. occipital.R: 2 exacta. y es en Los episodios repetidos de movimientos involuntarios sugieren estos casos en los que el pronóstico a medio-largo plazo es peor: el crisis motoras. No es sencillo distinguir entre crisis de ausencia y las crisis parcia. hay que realizar un amplio despistaje para la patología de base. ne no olvidar que la hipsarritmia (el EEG que se considera parte de la se le llama “jacksoniana”. que suelen durar segundos. a efectos de un cia. pero una simples con síntomas psíquicos y las crisis con alteración del nivel de TC puede ser también diagnóstica. y convie- primaria del lóbulo frontal. como lo son la sensación epigástrica previa a se siguen de cefalea hemicraneal en el lado contrario a las fotopsias la crisis y la confusión posterior al episodio. dado que los síntomas motores comienzan en la mano clásico es con ACTH. ya que hay varios fármacos útiles: el Por último. la sintomatología. falsa).. lo que no implica que todas las crisis que tienen su origen en Pregunta 47. No necesariamente tiene difícil está entre las crisis de ausencia con las crisis parciales comple. dado que en muchas ocasiones los propios síntomas El síndrome clínico que presenta el paciente parecen ser crisis psíquicos pueden conducir a trastornos en el nivel de alerta. y básicamente nos focalidad occipital. sin necesidad de más tumor. síntomas nos hacen descartar el diagnóstico de ausencias típicas. que por definición comparten las crisis generalizadas. Recordar que se tratarán de lesiones con captación de contraste en nas puedan tener síntomas que obligan al diagnóstico diferencial anillo. encontraremos movimientos involuntarios en el hemicuerpo contra- mente nos reduce el diagnóstico diferencial.R: 3 En el lóbulo frontal se sitúa la corteza motora primaria. lateral.R: 2 mayor parte de los casos (60%) existe patología subyacente.R: 3 dicho lóbulo cursen con alteración del nivel de conciencia. Los espasmos que se consi- En cuanto a la causa. luego son crisis parciales complejas) y el hecho de que en una caso teórico. Pero preferiblemente habrá que ele- gir la opción de las crisis parciales complejas por ser mucho más Pregunta 48. de entre las opcio. ni el tipo exacto de los la anomalía eléctrica paroxística. que es más frecuentemente algún síntoma incompatible con la misma. que no están en la hoz (donde se cido. leptógeno. imprescindible la existencia de lesión ocupante de espacio. paciente no tiene crisis (opción 4. conciencia. Los síntomas que presentan las crisis epilépticas dependen.. escasos minutos). conviene recordar que la forma más frecuente de demencia vascular es la que se produce por infartos múltiples. está indica- también pueden presentar algunas crisis focales. mientras que el aura rente de ausencias. Las luces centelleantes paroxísticas en un hemicampo sugieren les complejas que cursan casi como ausencias. En cualquier caso. dado el antecedente de un meningioma deran más frecuentes varían según las series. suponer a la inversa dónde situar el foco epi- léptico. clínica característica del síndrome) es interictal. pero no es un visual migrañosa suele durar varios minutos (menos de media hora) y Comentarios TEST signo incompatible. y que en ocasiones no es Las causas de crisis epilépticas de debut en adulto joven son fun- fácil de separar de una enfermedad de Alzheimer. aunque no es privativa de es un término menos definido en el que se incluyen las crisis que las mismas. de toda la corteza. que está intervenido. Las crisis generalizadas no convulsivas no existen cefálica hacia el lado contrario. primario en otra localización. aunque algu. que premonitoria de una crisis generalizada. que pueden ser clínicamente muy similares. ya que cuando do. sin desplaza o extiende.R: 3 generalmente de predominio subcortical. En los casos (40%) das como las crisis parciales complejas asocian alteración del nivel en los que no se encuentra patología subyacente el pronóstico es de conciencia. la duración es poco suge. La sensación de bolo epigástrico ascendente puede ser tendríamos que conformarnos con el de ausencias atípicas. que se produce cuando se estimula como tales en las clasificaciones actuales de las crisis epilépticas. hay que establecer el diagnóstico diferencial en- tiene que hacer descartar la ausencia el hecho de que aparezca tre crisis y episodios de aura migrañosa. Dado El diagnóstico no es difícil desde un punto de vista sindrómico. gica o por compresión por el tumor previo que puede generar la Elegir entre los tratamientos que han demostrado eficacia en el crisis. suelen ser epilepsias crónicas de difícil tratamiento. De igual modo. habrá que descartar una recidiva del tumor. el más importante es la desconexión del tante efecto de masa. De éstos.. pero no hay generalización con afecta. y por eso El paciente presenta desconexión del medio. en la circunvolución progresión más acusada. Las crisis parciales son crisis focales eléctricamente. jas. de hecho. que espacio. Si no conllevan deterioro del nivel de conciencia. mente indica extensión de la actividad eléctrica anómala a lo largo ples. conociendo mismos implica mejor o peor pronóstico de la patología de base. síndrome de West no es fácil. de la función del área cerebral en el que se asienta curso clínico de los episodios comiciales. A este tipo de crisis. y suelen ser de De entre los datos del caso clínico. La mayoría de las crisis parciales complejas tienen foco en el lóbulo temporal. en el área correspondiente a las primeras motoneuronas encar- estamos en fase sintomática. Para distinguir damentalmente los traumatismos previos y las lesiones ocupantes de entre ambas se ha utilizado la escala isquémica de Hachinski. con edema digitiforme o vasogénico alrededor.. mientras que no necesitamos antecedente tumoral para encontrar una metástasis. NEUROLOGÍA CTO Medicina 1ª Vuelta No existe un tratamiento que revierta los síntomas. se necesita un traumatismo al menos moderado para poder considerarlo causante de un foco epi- EPILEPSIA. con impor- con crisis generalizadas. descritas por Jackson. corteza parietal contralateral. aunque el valproato y el clonacepam también derecha. indica que el daño cerebral está estable. motora. y que Ante la sospecha de una lesión ocupante de espacio. El control estricto de los factores de riesgo aspira a prevenir una sitúa la raíz del muslo). a la 3 o la 4. cuando el ción del resto de la corteza. se debe especificar que hay alteración del nivel de ocasión se presente un episodio tonicoclónico generalizado simple- conciencia para diagnosticar de crisis parciales complejas y no sim. La presencia de estos (aunque pueden existir auras sin migraña posterior).R: 3 tumores den sus primeros síntomas por metástasis intracraneales. Otro límite especialmente difícil se sitúa entre las crisis parciales La prueba de imagen que más datos aportará será la RM. y pueden aparecer en las crisis parciales complejas y en cursan predominantemente con pérdida del nivel de conciencia las denominadas ausencias atípicas. es decir. Pregunta 46. De todos modos. localizamos el punto de inicio en el lóbulo frontal izquier- 10 .. pero no puede dilatarse mucho. por eso el paciente mira al hemicuerpo que tiene los salvo que se les quiera aplicar ese nombre a lo que conocemos movimientos involuntarios. Podemos también. Pregunta 44. del mismo hemicuerpo indica que la actividad eléctrica también se Las crisis de espasmos se presentan generalmente al despertar. se paciente suele quedar con una afectación psicomotriz importante y tratarán de crisis parciales simples. la corteza. pero no es sean los flexores y los mixtos (flexo-extensores). y escasos síntomas motores. y las crisis parciales simples no alteran el nivel de Las parestesias hemicorporales paroxísticas sugieren un foco en conciencia. Las crisis suelen tener inicio y fin bruscos. asimismo cada lóbulo frontal coordina la desviación oculo- nes. gadas de la extremidad superior. ya que tanto las crisis generaliza. el manejo más ade.. No está indicado el tratamiento profiláctico con antico- ascensos térmicos bruscos. 1) El tratamiento en monoterapia (una vez seleccionado un fármaco de primera línea para el tipo de crisis) consigue un buen control Pregunta 49. El efecto riesgo de presentar crisis más adelante. Indicación de fármaco antiepiléptico • Fenitoína (difenilhidantoína) intravenosa: de inicio de acción algo en función del tipo de crisis. de las benzodiacepinas es de corta duración. Si son recurrentes. el pacien- visto estadísticamente que se presentan antecedentes familiares en te tiene la hiperactividad y es preciso tratarlo con estimulantes un gran número de casos. una actividad eléctrica paroxística que clínica. por periodos concretos de tiempo y con segui.R: 1 • Benzodiacepina: intravenosa en adultos. En algunos casos concretos. se sitúan en primer o segundo lugar: la etosuximida y el a esas edades para presentar crisis generalizadas en relación con las ácido valproico. dado que es suficiente con el control de los epilepsia. para conseguirlo... 12345675892 2 . lo que significa que el paciente tiene crisis no causadas Pregunta 52. dado que puede producir depresión respiratoria. aumenta la resistencia a que 3) La etosuximida es un fármaco cuya única indicación hoy día son los cambios de temperatura produzcan alteraciones eléctricas. primera línea sería la carbamacepina cuado es el control de la temperatura. del diacepam. de vida (1-5 años) y se relaciona con antecedentes familiares de dios críticos. en niños puede producirse el anticomiciales como profilaxis de crisis febriles. en el que parece que la Se desconocen por el momento los mecanismos básicos por los alteración básica es una tendencia al sueño con dificultad para cuales se es más sensible a los cambios de temperatura. debido a la especial sensibilidad que tiene el cerebro autores. Pregunta 50. que comienzan en la retina periférica (con la consiguiente De modo que. preferiblemente con 4) El fenobarbital cuenta entre sus efectos secundarios más frecuen- paracetamol. Hay distintas pautas de actuación ante un estatus epiléptico. el tratamiento (o uno de ellos ) de Dado que la mayoría ceden espontáneamente. sino por la suma de ésta a otra patología. pero generalmente se establece una actuación por escalones: Pregunta 50.R: 2 de las mismas en un alta porcentaje de pacientes (60-80%) Las crisis febriles típicas son un fenómeno muy frecuente en la 2) En las crisis de ausencia típicas hay dos fármacos que. con lo que en mu- chas ocasiones no se espera a que el paciente tenga una segunda crisis para pasar al segundo escalón. bajo determi. No es preciso el tratamiento de base crisis febriles infantiles en los padres y con algunas formas de con anticomiciales. más que de una corteza cerebral normal con mayor sensibili. puede administrar diacepam oral o rectal en situaciones de ascenso Esto no es muy distinto de lo que sucede con el síndrome de térmico. para que no tenga la necesidad de ser hiperactivo En las crisis típicas.. las crisis parciales complejas. en otros países de forma recurrente.R: 2 únicamente por la fiebre. En principio. que se va peorar las crisis de ausencia alcanzando a lo largo de la infancia. En nues- Hay que recordar que “epilepsia” significa tener crisis epilépticas tro medio se utiliza habitualmente el diacepam. y sí el control de los ascensos de tempera- Las denominadas crisis febriles “atípicas” sugieren patología de base tura en los procesos febriles. ante una primera crisis: NO TRATAR. dado que el daño Pregunta 51. tanto el desarrollo. En En la pregunta nos presentan un caso de una crisis febril típica. el uso de este fármaco está limitado a Se trata de una serie de generalidades que hay simplemente que casos seleccionados. La vigabatrina da mejor resultado que los otros en antecedentes familiares y lo que conocemos de la enfermedad. y el control de los síndromes de West asociados a esclerosis tuberosa. sin que esto signifique mayor dosis. reducción periférica del campo visual) y que obliga a realizar campimetrías de control a los pacientes tratados. pero se ha mantener el nivel de alerta y por eso. dad a la hipertermia. según los edad infantil. las crisis de ausencia típicas. por eso puede estar indicado iniciar el tratamiento a la primera pero su uso generalizado se ha seguido del hallazgo de lesiones en la crisis. 5) Las crisis febriles son un proceso frecuente en los primeros años rendimiento y funciones cerebrales son normales fuera de los episo. hay que tener cuidado con la mente conlleve una crisis epiléptica. rectal en niños. por lo que hay quien prefiere hablar de “crisis con fiebre”. La fenitoína y la carbamacepina pueden em- subidas térmicas.R: 4 es irreversible. Con la madurez completa de la corteza. pero febril. se puede que tiene una repercusión clínica en el contexto de un síndrome administrar diacepam al principio de cada proceso febril. NEUROLOGÍA 1ª Vuelta CTO Medicina puede ser eficaces. se efecto paradójico de desencadenar un cuadro de hiperactividad. no es una práctica generalizada. No está indicado el tratamiento continuado con tes la sedación y la somnolencia. como el comportamiento. miciales a largo plazo. retina. En cualquier caso. disponen de loracepam por vía intravenosa y se utiliza en lugar nadas circunstancias. Cualquier cerebro puede tener. más lento. pero de vida media larga. así como el EEG. permite que posteriormente Comentarios TEST se mantenga el tratamiento a largo plazo con el mismo fármaco. hiperactividad e hipercinesia primario. recordar: miento campimétrico. sin que controle ningún otro tipo. aunque no en todos los países. Si con la administración de fenitoína en 1234526 789 1. 27323 284 También está admitido el ácido valproico intravenoso. pasar al siguien- 5446542 326522 1.    dosis máxima (hasta 25 mg/kg) no ceden las crisis. R: 2 " 3$  % 8 1 89# 7 .. si es preciso. con anestesia general si es preciso. En algunos casos se precisa otro 452 2 !542 1   tipo de barbitúricos. • Barbitúricos: el fenobarbital como primera opción. Su adminis- 452  !542 1 " tración generalmente ha de ser en la UCI o con posibilidad inme- diata de intubación. 546542 89# 1 Pregunta 53. 1 789 8 te escalón. si no aparecie- 542 1   89# ra el tema del embarazo. un paciente epiléptico con buen control con un fármaco durante años. en este caso. En principio. la opción correcta sería la 1. si está cinco años sin crisis (como en este caso) " 3$  &22 1   89# con pruebas complementarias negativas. 1234567897 76 . podría ser candidato a un intento de lenta retirada de medicación. 7545  75456 57. 8 6523459  76 . no hemos evitado las es bajo o no es posible determinarlo. paciente presente de nuevo crisis. tenemos que decidirnos entre asumir el riesgo de que se puede estimar el riesgo de presentar nuevas crisis por los malformaciones o el riesgo de crisis durante el embarazo. 7 Un embarazo con tratamiento farmacológico en general tiene  47 8 ! 75"45 !7978 75#$45 97 75$% 45 &'( 75#245 9789 71 mayor riesgo de malformaciones fetales. es decir. tratamiento. las primeras semanas. porque una retirada brusca puede hacer que el negativo. incluso de una forma más agresiva Los síndromes epilépticos merecen una consideración aparte. Esta balan- 11 . cuando del tratamiento: si hacemos una lenta retirada. Estas se producen en la fase de organogénesis. dado el no desestimable número de casos en los que se La retirada de un fármaco anticomicial debe hacerse de forma presenta una crisis aislada con un estudio complementario normal o muy lenta (meses). no está indicado iniciar un malformaciones. que las que motivaron el inicio del tratamiento. eso obligaría a una suspensión brusca en el riesgo estimado de presentar nuevas crisis en el futuro. pero ya estamos en el contexto de una patología hereditaria de la Visto lo anterior. con lo que si quere- mos evitar ese riesgo hay que suspender el fármaco al menos durante La decisión de instaurar un tratamiento anticomicial está basada la primera fase del embarazo. la información ascendente por el fascículo longitudi- sometido a un fármaco. aparece un nistagmo en el ojo que Es cierto que puede haber una disminución de niveles del fárma.R: 4 en el fascículo longitudinal medial del ojo que no mira hacia adentro. que la madre embarazada sufra una crisis. si no se menciona el fascículo. que el de nal medial facilita el movimiento de abducción del ojo contralateral. aparece un ojo que no adduce (no mira hacia adentro) y un ojo que abduce (mira hacia fuera) pero con nistagmo. por lo que. en el que y menos a priori.R: 1 Entre los síntomas menos frecuentes en el curso de una esclerosis 12345678349 . ocular... en la protuberancia de ese lado. Efectos secundarios de la medicación anticomicial. podemos situar la lesión Pregunta 54. al producirse la lesión. o. pero no se suele precisar un aumento de dosis. en especial si es en monoterapia. NEUROLOGÍA CTO Medicina 1ª Vuelta za se inclina hacia lo primero: es menor el riesgo para el feto de estar Asimismo. En resumen: ante la presencia. sí abduce. co durante el embarazo por aumento del metabolismo y del volu. Pregunta 54. como único defecto de motilidad men de distribución. Pregunta 57. de alteración en la mirada horizontal hacia un lado. 6  . 4679376793 37 6 94 múltiple se encuentran los fenómenos paroxísticos: dolores o pares- 76385. 8785 4. 87. 85 58 7"6 68 tesias lancinantes que recuerdan las neuralgias (como la neuralgia 7 944 73 54.  7"6 68 del trigémino) en principio asumidas como originadas por lesiones y 7. 8. 73 9. aunque a veces no hay forma de 71.  7"6 68 áreas de desaferentización sensitiva. 53. Generalmente responden bien al tratamiento con 7 53 23 3 4. 7"6 68 justificar la causa. 746 75.  . como 7776869 38 las neuralgias esenciales).8 inmunomoduladores (antiepilépticos. antidepresivos tricíclicos. 7!3 . 4 786 63 4. 7. 5. .  7"6 68 La existencia de cuadros de actividad neuronal anómala (como las 7"3 . 56# 388 7"6 68 crisis epilépticas) son infrecuentes en la EM (un 3%) y se producen en 7$ 895867%3 . 8. 7 .  73 56( . 777 3 . relación con lesiones activas que se sitúen próximas a la sustancia gris. Lo mismo sucedería en los movimien- 7$3 .5587998&'3 postulándose que la irritación cortical que produce la inflamación activa sería la causa de las crisis. 6 3. 7. 634.  7). en lesiones próximas a los ganglios basales. bien 7776 38 . 646 tos coreoatetósicos. 7. 4 3. sobre todo los visuales. que a la exploración se acompaña de signos de lesión En el EEG se valora la actividad cortical.R: 3 cas o por la administración de contraste. de tal forma que la proteinorraquia total no aumente. Los trastornos del control de los esfínteres son muy infrecuentes al limitándose a apoyar o a alejar el diagnóstico. muy útiles para el estudio de la desmielinización en el sistema nervio- La disfasia aislada no encaja dentro del curso clínico de la escle. so central. se detectan anticuerpos en el LCR que no par. nes posteriores a nivel cervical. con el área mesen. rosis múltiple. Los hallazgos en las pruebas complementarias no son por sí mis- merosas lesiones diseminadas. que de Los trastornos afectivos. para el estudio de Comentarios TEST edema de papila (papilitis) o ser normal (neuritis retrobulbar). La prue- disminución de la agudeza visual. con lo que puede significar un signo de en brotes. son comu- vasculitis. longitudinal medial.. El signo de Lhermitte es un signo clásico de afectación de cordo- ENFERMEDADES DESMIELINIZANTES. puede demás sugerente de EM. y no cursa cuando hay lesión medular. puesto que es capaz de distinguir un mayor número alteración en este sentido. resultan apreciará una atrofia de la papila. bien por el uso de determinadas secuencias específi- Pregunta 56. Este es el hallazgo que aparece de forma más fuera en reposo.R: 4 descendente. a los ventrículos. nes en la EM. tratamiento preventivo con litio u otros estabilizadores del ánimo. periependimarias. ha relacionado con la carga lesional o la existencia de lesiones en El síntoma de comienzo más frecuente de la EM es la alteración áreas concretas. sí es la menos sugeren- El cuadro clínico se produce por la interrupción del fascículo te de la enfermedad y obliga a plantear antes otros diagnósticos. El déficit motor también es frecuente como síntoma de debut. y su localización. ante la presencia de una clínica por lo múltiple. con la aparición de un dolor agudo (tipo descarga eléctrica) que recorre la columna vertebral en sentido Pregunta 55. esto implica una síntesis ante una lesión del fascículo derecho se encuentra una imposibili. y en aquella aparece motoneurona. Pasadas unas semanas. La prueba más útil para el diagnóstico es hoy por hoy la resonan- medad.R: 2 generalizada. siendo las lesiones. en el fondo de ojo se óptico. Los potenciales evocados. se dice que si es bilateral y en un paciente joven. que puede presentarse como franca paresia focal o como debilidad Pregunta 58. especialmente la depresión. debut. sobre todo la resonancia. sino que puede presentarse La esclerosis lateral es una enfermedad que afecta a la en lesiones medulares cervicales de otro origen. y yor número de casos en los causados por la esclerosis múltiple. aunque información para la addución del ojo ipsilateral. En el LCR iremos a frecuentes al debut: lo más frecuente es un escotoma central con buscar las típicas bandas oligoclonales fundamentalmente. La localización de las lesiones. se objetiva un ma- las que se incluyen la esclerosis tuberosa. lo que indica que se han sintetizado el ojo puede hacer movimientos verticales y no está desviado hacia dentro del SNC. son más sugerentes de esclerosis múltiple. pero no hay que confundir la oftalmoplejia con la lesión del III: están presentes en el suero. Es un movimiento (el del recto interno) que induce el III del mismo paciente. por tanto se presenta con datos motores. alterada en la esclerosis múltiple. Los hallazgos del LCR apuntan hacia una actividad inflamatoria municación la zona de organización de la motilidad horizontal con. de lesiones e incluso diferenciar entre lesiones desmielinizantes acti- vas o antiguas. 12 . que recorre la protuberancia. cos causados por la terapia corticoidea. Una facomatosis es un grupo de enfermedades hereditarias. El fondo de ojo puede presentar ba de imagen resulta útil. aunque no se recomienda el Puede consistir en parestesias (descritas como pinchazos u hormi. mos diagnósticos ni descartan el diagnóstico de esclerosis múltiple. con palidez de la misma. aunque no sucede lo mismo a lo largo del curso de la enfer. si bien no descarta el diagnóstico. como sucedería con una lesión del par III. pegadas mal pronóstico si aparece en el debut de la enfermedad. • Aumento del índice de IgG: en el total de proteínas de un líquido El fascículo longitudinal medial de cada lado es el que lleva la normal aparecen más inmunoglobulinas de lo habitual. constante en la enfermedad.. De este modo. salvo que esté asociada a deterioro cognitivo por nu. No es exclusivo de la EM.558 por interrupción de vías inhibidoras. La panencefalitis esclerosante subaguda no cursa en bro. También pueden aparecer otros síntomas psiquiátri- de la sensibilidad. el hallazgo La oftalmoplejia internuclear es muy característica de la esclerosis de una RM cerebral normal. excesiva de IgG. dad para la addución del ojo derecho cuando se intenta la mirada • Bandas oligoclonales: al estudiar las IgG en el LCR y en el suero conjugada. entre que producen un grado similar de discapacidad. y se otras. ya que se trata de una afectación de Las alteraciones visuales son características. lo que se produce hasta en el 45% de los casos. En el FO podemos valorar el nervio más frecuente lo último. bien por irritación. dado que no es lo más frecuente. aunque sean menos la sustancia blanca del sistema nervioso central. que se encuentra junto a los sextos pares. de carácter congénito. zados dentro del SNC: esto se demuestra mediante dos mediciones: cefálica próxima a los terceros pares. puesto considerarse casi patognomónico. que podría ser normal en fases muy precoces. dentro del SNC cuando aparecen anticuerpos que han sido sinteti- jugada. Valorada la incidencia de depresión en otras patologías tes. gueo) o hipoestesias (acorchamientos).. que no tiene por qué estar piramidal. la neurofibromatosis. poniendo en co. donde más de la mitad de los pacientes sufre algún tipo de cia magnética. mente signifique una reducción de las lesiones aparentes. que hayan presentado dos brotes en los dos últimos Las causas son múltiples. de la desmielinización. En el LCR se detectan títulos de anti- tamientos. con interferón beta o con otro fármaco. ha valorado que el tratamiento con corticoides en un primer brote de neuritis óptica retrasaría la aparición del segundo brote. con lo que el paciente no está Comentarios TEST de glatiramer.R: 2 La descripción clínica de un paciente con aparente buen nivel de Pregunta 60. no está indicado el tratamiento. ni alteraciones en el EEG. los que se consideran más útiles son los visuales. mientras que para la virus vivos atenuados. mo- venosa en altas dosis conseguiría este efecto. Incluso se El tratamiento. efecto. de esclerosis múltiple. Se administran por vía subcutánea o intramuscular con distintas lo que incluye los movimientos oculares horizontales. conseguiríamos un menor daño y una mejor La clínica puede variar desde un síndrome meningo-encefalo- recuperación. pero no tiene motilidad voluntaria por debajo de ese nivel. que han demostrado reducción de un 30% de los comatoso. pero no sue- administra el interferón. por lo que.R: 5 Para contestar esta pregunta basta conocer cuál es la utilidad del Pregunta 64. oligoclonales. crisis epilépticas). sin modificar la intensidad de éstos. Parece que el tratamiento con corticoterapia intra.. ninguna enfermedad monofásica se beneficiará de involuntarios hay que pensar que se trate de movimientos coreicos. debería hacernos pensar en alteraciones dege- de brotes. lo que da lugar a una deshi- progresivas primarias (en las que hay una lesión continua.v.R: 3 sin que se sepa la razón y sin que se hayan repetido los resultados En un paciente de esta edad que previamente está bien y comien- en brotes de otro tipo za con un cuadro de deterioro progresivo hay que pensar en enfer- • La disminución del número de brotes: en las formas recurrentes. pero que estaban funcionando a pesar de la desmielinización. se deterioro mental y tetraplejia de instauración progresiva. mielítico grave con afectación de muchos sistemas (cerebeloso.. ya que no hay brotes que reducir.. mientos continuos involuntarios es el dato más significativo. un tratamiento de este tipo. len presentar crisis epilépticas. de forma genérica. Pregunta 61. si conseguimos disminuir la actividad inflamato. sin brotes) dratación y destrucción de las células a dicho nivel. con menos frecuencia no existe indicación de tratar. mioclonías. ria lo antes posible. Un paciente adulto en el que debutan movimientos continuos Por lo tanto. Se ha descrito mejoría de las lesiones desmielinizantes en la base.R: 5 interferón para dejar reducidas a dos las opciones a pensar. la más frecuente. probablemente por activación de la inmunidad celular. y tras la administración de suero antitetánico valoración del nervio óptico lo es el estudio del potencial evocado (no del toxoide).. El interferón beta.. que. que resonancia magnética tras el tratamiento con interferón. y hoy por hoy no hay tratamiento que consiga una regene.. que hace que el paciente tenga menos mientos continuos generalizados sin especificar área predominante ni episodios. 13 . con el que se puede mantener una conversación en la El tratamiento modificador de la historia natural de la esclerosis que las contestaciones son a base de parpadeos. macos para disminuir el número de brotes cuando aún no se En este paciente en el que expresamente se nos describe una sabe el número de brotes que espontáneamente presentará el alteración característica del EEG hay que pensar en una panencefa- paciente. No tiene buen tratamiento. se diagnostica de forma blanca. hasta un cuadro leve similar a un primer brote el grado de recuperación es el mismo en tratados y no tratados. a patología congénita y no degenerativa de inicio tardío. en clínico. Clásicamente era la infec- vista conceptual. con lo que a largo plazo tendríamos una menor carga nerativas de la sustancia blanca: las leucodistrofias. con corticoterapia intravenosa. el hecho de que se describa un paciente con movi- múltiples remitentes-recurrentes para disminuir el número de brotes. espacio. pañado de coma. Pregunta 62. el caso de presentar un nuevo brote tiene indicación de ser tratado Se produce una encefalitis difusa de las sustancias gris y blanca. brotes. la vacuna del sarampión. y hoy por hoy. Pregunta 59. Se produce en pacientes que han sufrido el sarampión (también es posible en vacunados. una reactividad cruzada con antígenos de la mielina. pero no virus. En este objetivo han demostrado eficacia varios fármacos. con lo que probable. por lo que no se trata a partir de un único brote. el paciente debe recibir terapia con corticoides. es la imagen carac- múltiple hoy por hoy está basado en la reducción del número de terística de un síndrome de cautiverio. salvo que se identifique la patología de miento. En efecto. Los movi- Es un inmunomodulador. ración neuronal. sino que será preciso tratar con cortico. aunque a largo plazo tor. por lo que no se recuperan con ningún trata. con pocos signos inflamatorios. dado que para el Las vacunaciones que se han asociado con este cuadro son las de resto de los sistemas la RM es más sensible. No existe ningún tratamiento y la evolución es mortal en el curso nización. sensitivo. y hay que recordar como curiosa la años. mielinólisis central pontina yatrogénica por una recuperación de- En las formas monosintomáticas (un único brote) y las formas masiado rápida de una hiponatremia. Este síndrome se produce por lesiones protuberanciales extensas siendo los más usados en la actualidad el interferón beta y el acetato que respeten la sustancia reticular. se produce una desmielinización difusa de la sustancia enlentecimiento en alguno de los sistemas.R: 3 Hoy las infecciones espontáneas con las que se asocia de forma El tratamiento de la esclerosis múltiple se basa en dos puntos: más frecuente esta patología son las infecciones respiratorias de etio- • La disminución de la intensidad del brote: desde un punto de logía desconocida. como se ha dicho. en caso de velocidad de ejecución los podríamos denominar “coreoatetósicos”. se sustituya la vacuna de virus atenuados por una sin virus vivos. lesional y por lo tanto menos secuelas. con indirecta de desmielinización en algún punto de la vía. por menos efectos secundarios relativos. PATOLOGÍA EXTRAPIRAMIDAL. enfermedad. Puede aparecer una destrucción espontánea de la mielina o aso- Las secuelas de brotes previos indican lesiones residuales más allá ciado a otras lesiones desmielinizantes. a pacientes con formas recurrentes-remitentes de la ra craneofacial y el resto del cuerpo. ción por el virus del sarampión. presentan ceguera. litis esclerosante subaguda. las leucodistrofias son cuadros degenerativos propios de to de glatiramer y algunos inmunosupresores han obtenido este la edad infantil. medades degenerativas que. sordera. es con dosis altas de corticoides. hace que en países de baja prevalencia de sarampión. de 1-4 años. Hoy por hoy. También hay bandas La existencia de secuelas establecidas en un paciente con escle. pero en los tratados esa recuperación se produce antes. con lo que no se dispone de ningún tratamiento que ayude a mejorar secuelas establecidas desde brotes antiguos. y lo más importante es distinguirlo de un cuadro compresivo. pero sin puede causar el mismo déficit motor. dado que se describe con afectación de remitentes existen tratamientos capaces de disminuir el número la sustancia blanca. seguidas de la varicela. como cualquier otra inflamación aséptica activa del SNC. pero que generalmente irá acom- cambios en la situación funcional del sujeto. Si se detecta vacunación. presentar brote. rosis múltiple habla de lesión axonal residual añadida a la desmieli. cuerpos antisarampión altos. como ocurriría de tratarse de una lesión ocupante de brotes previstos en comparación con grupos control. alteraciones pupilares y en los reflejos de tronco.R: 4 conciencia. ya que los movimientos atetósicos con mayor frecuencia se asocian terapia i. Esto visual. Pregunta 63. auditivo. no siendo incompatibles ambos tra. NEUROLOGÍA 1ª Vuelta CTO Medicina Los potenciales evocados se basan en la detección de la actividad En el caso de la encefalomielitis diseminada aguda. Aunque la presencia de trastornos anímicos y de la memoria pueda El interferón beta es un fármaco que se administra en las esclerosis ser patológica. la musculatu- pautas y dosis. No está indicado el tratamiento con fár. motor). pero con una inci- Un paciente que esté sometido a tratamiento para modificar el dencia muchísimo menor) y cursan de la forma descrita en el caso curso de la enfermedad. que se pre- cortical que se produce con la estimulación de distintos sistemas senta entre días y escasas semanas después de una infección o una periféricos (visual. el aceta. R: 3 rrecta la opción 4. El tratamiento de la enfermedad tiene un objetivo meramente Entre el 60 y el 90 % de los pacientes presenta inestabilidad postural sintomático. tica. • Funciones corticales: cambios de comportamiento. dado que el diagnóstico definitivo se establece post-mortem y que nosticando como siempre si no se dispone del estudio genético. tetrabenacina). de la que aún no se conoce su función normal. serotoninérgico. La enfermedad de Balo es un tipo de típica: enfermedad desmielinizante. sin temblor zadas de la corteza cerebral. ello. tipo de antidepresivos (tricíclicos). y que (conservación de la conciencia): no sugerente de enf. separar los datos del caso clínico en son el 50%. sonianos preceden a los cognitivos en un año. pero que conlleva bradipsiquia. de conlleva una destrucción neuronal en distintos sistemas.. tico. De he- cho. • Que es de herencia autosómica dominante. algo hipoactivos. y por eso se considera co. un número no pequeño de pacientes (hasta un 40-50%) pre- Pregunta 66. entre los que se encuentran los neurolépticos clásicos (halope. Son casos Existen una serie de datos clínicos muy sugerentes de la existencia Comentarios TEST heredados del padre y no de la madre. aunque al principio del cuadro pueda confundirse y evolución más benigna. diagnosticarse erróneamente de Enfermedad de Parkinson. diente toma ese cromosoma. el locus ceruleus. alucinacio- nérgica. pero en este caso es selectiva: sólo está aumentada de tamaño de piramidalismo.R: 3 des de que los hijos de un paciente afecto padezcan la enfermedad Para no caer en la confusión. en algunas ocasiones se pueden La presencia de un cuadro parkinsoniano con rigidez y bradici- nesia nos debe hacer pensar siempre en primer lugar en una enfer- 14 . con reflejos miotáticos aumentados (incompati- el asta frontal. Todo ello compatible con enfermedad de En esta enfermedad se produce una alteración de la proteína Parkinson. alguno que jugar en una enfermedad con una hiperactividad do- paminérgica. pero con afectación en fases más avan. no suele presentarse hasta transcurridos cuatro-cinco años del co- Los síntomas coreicos se controlan con antagonistas dopaminér. mientras que el ble con enfermedad de Parkinson). En ocasiones es especialmente difícil el diagnóstico diferencial ma 4 establece el diagnóstico de certeza incluso en fase sintomá. de una parálisis supranuclear progresiva o enfermedad de Steele- 2) Variante de inicio tardío: la corea como signo predominante. co más llamativo que el coreico. NEUROLOGÍA CTO Medicina 1ª Vuelta Estamos por lo tanto ante un paciente con demencia.R: 5 • Que la atrofia del núcleo caudado (más acusada en la cabeza) Se trata de un parkinsonismo con ciertos datos característicos que hace que ese espacio se ocupe por el ventrículo lateral próximo. reflejos presen- de este caso también habría padecido la enfermedad. lo que es más llamativo en Se ha intentado actuar sobre otros sistemas de neurotransmisión la hiperextensión forzada del cuello que suelen presentar estos pa- afectados (gabaérgico. hipertonía plástica. parálisis vertical de la mirada es al mirar hacia abajo. afectivo y movimientos involuntarios: esto es suficiente para el diag. sin claros efectos beneficiosos. Pregunta 67. los agonistas dopaminérgicos no tienen papel rias. afecta a Igualmente son demasiados signos motores extrapiramidales para hombres y mujeres. aunque hoy día puede sobre los receptores presinápticos. • Extrapiramidal: hipomimia facial. tes. Pregunta 68. Sin embargo. con lo que. caídas). típico de una parálisis supranuclear progresiva. que no cursa con bradicinesia. Todo resto del ventrículo (astas temporales y occipitales) es normal. ya que los sistemas serotoninérgicos pare. incluso labilidad emocional. hoy día el estudio gené. Caídas frecuentes que no parecen crisis epilépticas huntingtina. Huntington se desconoce. si los síntomas parkin- cognitiva. por lo tanto. es 1) Variante juvenil (de Westphal): presenta un cuadro rigido-acinéti. la Eso en la TC semeja una dilatación ventricular como las hidrocefa. con un tono relativamente conservado en las extremidades.. primera). medad por Cuerpos de Lewy. movimientos oculares verticales con conservación de los reflejos ridol. nando la atrofia del estriado. cursaría con datos de afectación cerebelosa. Son demasiados los datos que hacen más que dudoso el diagnós- Pregunta 65. el núcleo subtalámico de reposo (lo que esperaríamos en una enfermedad de Parkinson) y otras áreas ni movimientos involuntarios (lo que presentaría una variante de Desde el punto de vista bioquímico. pero no con la clínica psiquiátrica ni con la De forma arbitraria se ha establecido que. colinérgica. estaremos ante un • Que presenta el fenómeno de anticipación: en la duplicación Parkinson-demencia. en busca de las repeticiones del triplete CAG en el cromoso. cio más precoz. para la formación de las células sexuales se produce un te al comienzo con el cuadro motor. La herencia es autosómica dominante. existiendo una hiperfunción dopami. y tienen peor pronóstico. además de ser más joven que el paciente de la pregunta) relacionado es la dopamina. lo consideraremos una Enfer- aumento en el número de copias CAG. incluso entonces. limitando la liberación de más establecerse con certeza en ausencia de antecedentes o incluso de dopamina. irritabilidad. que La base neuroquímica de la depresión en la enfermedad de no suele ser una demencia completa. Ni el curso clínico ni la edad del Conviene recordar con respecto a esta enfermedad: paciente acompañan a la variante rígida de la enf. y las frecuentes caídas. nos tienen que llamar la atención: rigidez cervical en extensión. salvo para empeorarlos. mientras que si la demencia precede o coexis- genética. puede no quedar claro. La afectación típica de los movimientos oculares supranucleares medad. La muerte se produce por complicaciones asociadas (respirato- En principio. pero ante la presencia de antecedentes dos generaciones las distintas áreas de la exploración: atrás se tiene prácticamente la certeza de que el padre del paciente • Motor: hipertonía plástica (resistencia continua). el principal neurotransmisor Westphal. ya que hasta el momento no se dispone de ninguna al inicio. con el diagnóstico de Enfermedad de Parkinson. tiapride. dando lugar a caídas frecuentes. Parkinson. Tener en la memoria también dos variantes respecto a la forma como ataxia de la marcha. pero también se altera la neurotransmisión gabaérgica. Pregunta 69. colinérgico. entre la demencia por cuerpos de Lewy y un Parkinson-demencia.. Existe muy frecuentemente un deterioro cognitivo asociado. Puede haber una rigidez axial intensa. lo que ningún tratamiento consigue detener la evolución de la enfer.. aunque existen casos de debut más de la recaptación de serotonina (fluoxetina. que es la que está junto al caudado. sulpiride) y los depletores presinápticos de dopamina oculocefálicos. instauración subaguda. similares. si el descen. trastorno obtener beneficio con dosis bajas de agonistas dopaminérgicos. se puede seguir diag. por un efecto paradójico que se postula se produce al actuar nóstico de la enfermedad de Huntington. Richardson. con inhibidores selectivos mente en torno a los 50-60 años. la enfermedad debutará antes en él que en su antecesor. predomi. somatostatina) cientes. una enfermedad de Creutzfeldt-Jakob o de Alzheimer. cen estar afectados tanto en los pacientes deprimidos como en los La parálisis supranuclear progresiva tiene un rango de presenta- que no lo están. consistiendo en una limitación para los gicos. presenta datos lias. por lo que las posibilida. dada la aparición de datos anatomopatológicos Un mayor número de tripletes parece correlacionarse con el ini. mediante estudio genético. esto no quiere decir que sea preciso. la ataxia hereditaria más frecuente. más frecuente- se puede iniciar un tratamiento empírico.R: 4 sentan una cierta mejoría con la administración de levodopa. mienzo de los síntomas. la nes visuales. monofásica. de Huntington. paroxetina) o con otro precoz. que cursan de • Que aunque el diagnóstico se puede establecer por la clínica y una forma menos llamativa en lo motor (salvo las mioclonías en la los antecedentes familiares compatibles.R: 4 tico de enfermedad de Parkinson. En los pacientes que presenten síndrome depresivo ción muy similar al de la enfermedad de Parkinson. la somatostatina y la sustancia P. clínica.. (reserpina. La ataxia de Friedreich. La atrofia olivopontocerebelosa. y crisis epilépticas. dato poco compatible con molécula capaz de detener la muerte de las neuronas afectadas. hay que pen. En fases más avanzadas es cuando aparece la bradicinesia carac- dominante síntomas cerebelosos. el paciente generalmente no sufre caídas frecuentes hasta fases avanzadas de la enfermedad. Pero la presencia de marcha predomina el cuadro vegetativo sobre el motor.. ya que en la enfermedad de Parkinson idiopática no hay síntomas predominantemente genitourinarios: podríamos encontrar. y más limitante los síncopes de largo tiem- En el caso de la pregunta. ya se nos presenta la existencia de po de evolución. vegetativos.R: 4 dad. es poco satisfactorio. con nóstico. En el caso de la atrofia multisistémica que presenta de forma pre. unos reflejos vivos o exaltados obligaría a replantearse el diag- Nos están describiendo un cuadro de afectación vegetativa. el cuadro parkinsoniano. hay con considerar la presencia de una atrofia mul.R: 5 diendo de la sintomatología preponderante. por ser éstas las localizaciones en las que en un princi. lo que parece menos llamativo es sar en principio que no se trata de una enfermedad de Parkinson. postural. En un principio se consideraban distintas enfermedades depen. en una mano. por lo que podría corresponder a te que éstas se alteren. especialmente de Siempre que aparezca un cuadro parkinsoniano con síntomas los síncopes. por cuerpos de Lewy si la demencia aparece en fases iniciales. se de una enfermedad de Parkinson si el cuadro motor hubiera sido A pesar de las alteraciones posturales. podría tratar. salvo que haya además otras lesiones nos ante un cuadro de disautonomía periférica o bien de afectación asociadas. recomienda evitar hipotensiones y situaciones en que éstas pueden tisistémica: este es un cuadro que engloba la coexistencia de sínto.. cuando el temblor de reposo es el signo más frecuen. siendo en fases avanzadas cuando no es infrecuen- diagnóstico de patología medular. que pueden producir problemas por las caídas. hipertensor. se extiende a la mano contralateral. y en este caso es pre- tar síntomas de afectación cerebelosa. El tratamiento de los síntomas motores. Comentarios TEST atáxica automáticamente nos lleva a la lista de diagnósticos diferen. es el temblor de reposo. y que obliga al diagnóstico diferencial con la enfermedad Ante un síndrome parkinsoniano con disautonomía. dando lugar a lo que se denomina parkinson- un síndrome periférico si no fuera por los signos parkinsonianos. Este es el patrón típico de la enfermedad de Shy- escaso temblor. Si aparecen signos de afectación de otros sistemas. en la que te de inicio de la enf. Gene- parkinsoniano (ver figura en página siguiente). puesto que no puede presen. 15 . Diagnóstico diferencial del parkinsonismo medad de Parkinson. como en el resto de ciales en la que no cabe el Parkinson. Drager. junto con el aumento de tono muscular y la inestabilidad to-cerebelosa. cerebelosos y piramidales en distin. medular con afectación de segmentos inferiores. En este paciente. de Parkinson. la denominamos atrofia olivo-pon. que con el tiempo ban de presentaciones distintas de un mismo proceso. hasta que se demostró La presentación típica de la enfermedad de Parkinson idiopática que en realidad la anatomía patológica era la misma y que se trata. afectación de la vía piramidal. terística. que se encuadra dentro de la atrofia multisistémica. unilateral. ralmente los signos disautonómicos no suelen ser muy marcados. to grado en el mismo paciente. Se incompatibles. que es la causa más frecuente de síndrome compatible y previo a la aparición de los síntomas vegetativos. sin embargo. La capacidad cognitiva de los pacientes se conserva también du- La presencia del antecedente de síncopes no acompaña para el rante largo tiempo. demencia. ciso un buen control de los síntomas vegetativos. Pregunta 71. producirse y hay algunos autores que recomiendan tratamiento mas parkinsonianos. atrofias multisistémicas. NEUROLOGÍA 1ª Vuelta CTO Medicina Pregunta 69. aunque este extremo no está comúnmente aceptado. pio se creía estaba confinada la degeneración Los reflejos miotáticos suelen conservarse normales o mínima- mente disminuidos a lo largo de toda la evolución de la enferme- Pregunta 70. hipercinesia pico de dosis. durante años puede ser incluso el único dato. con lo que son frecuentes los antece- dentes familiares en la anamnesis. distonías fin de dosis. sobre todo las discinesias tardías. cerebral profunda (estimulación crónica del núcleo a alta frecuen. con presentaciones diferentes. y nos tenemos que ima- sintomatología de esta enfermedad. o el uso de medicación con liberación lenta. con fenómeno de rueda dentada.R: 4 tratamiento con betabloqueantes. mientras que en to. tarias son especialmente útiles para descartar otras patologías. En cualquier caso. antes de que se dispusiera de la L-Dopa.R: 4 suelen ser generalmente discinesias de predominio coreico. la ausencia de alteraciones en la las células y fibras nerviosas en el lugar de la lesión) y la estimulación marcha no apunta a un cuadro cerebeloso. etc). facilitado por los avances técni. junto a los efectos secundarios asociados con su admi- nistración. La dosis de que considerarse un signo independiente).R: 2 Comentarios TEST de la L-Dopa. El tratamiento quirúrgico de la enfermedad de Parkinson comen. o momento off. miento con este fármaco. generalmente combinadas Pregunta 73. que suele conservar las funciones superio. en las que el pacien- obligan a replantear el diagnóstico de enfermedad de Parkinson. que. Antes de dar este diagnóstico hay el VIM (lesión o estimulación). de síndrome parkinsoniano no idiopático. Fluctuaciones complejas son las que se presentan de modo do a las primeras motoneuronas (¿isquemia lacunar?). apareciendo fluctuaciones motoras (fenómeno “wearing off” esperable que el Parkinson. levodopa empleada sí influye en los posibles efectos secundarios. jos vivos. ganglios de la base (globus pálido medial y núcleo subtalámico) y el Por lo tanto. Esencialmente existen dos tipos de técnicas para el tratamiento quirúrgico de la enfermedad de Parkinson: las técnicas ablativas (le. interviene toda la extremidad. que produce un efecto similar al de la lesión pero reversible). administración de levodopa y recupera su situación funcional tras la generación estrío-nígrica) o que haya otras lesiones distintas afectan. no Hay que recordar que el diagnóstico de enfermedad de Parkinson distonías. tóxicas. y pueden deberse a múltiples causas. con objeto de mante- aparición de esta última en los años 60 decayó el interés por la ner un estímulo dopaminérgico continuo sin fluctuaciones. La pérdida de efectividad a largo plazo Pregunta 76. o de beneficio de dosis Pregunta 72. tanto la corteza como el área subcortical tiene afectación. El paciente ha de estar absorción. al inhibir la metabolización periférica de la L-Dopa. si es muy intenso. El fenómeno on-off no se relaciona con los niveles idiopática es eminentemente clínico. de la sintomatología del paciente.. que es lo característico de este cuadro. en general no responden a levodopa. con lo cual es 8 años. Básicamente aparecen fluctuaciones en la Por último. a la medicación o que presentan intolerancia a la misma o efectos • El temblor cinético aparece al realizar movimientos en los que secundarios importantes que limitan su uso. con sintomatología incapacitante. fenómenos “on-off”). Pero a lo largo mas y la reducción de dosis de L-dopa. aunque cupiera la duda de si se trata de sión mediante termocoagulación por radiofrecuencia que destruye un temblor postural o cinético. levodopaterapia crónica”. • El temblor de actitud es visible al mantener una postura o con mente jóvenes. recordar que los signos de afectación piramidal (refle. cirugía en esta enfermedad. y consiste en un empeoramiento Lewy. El 16 . con objeto despierto para colaborar durante la cirugía. como la rigidez. asegurarse de que no hay interferencias con la implantación de electrodos de estimulación. el aumento de forma prolongada algunos autores las denominan “síndrome de la transmisión dopaminérgica. discinesias bifásicas. te presenta de nuevo déficit de movilidad a las 3-4 horas de la última debiendo considerarse en primer lugar una atrofia multisistema (de.. pero se especifica la ausencia de temblor Ambos procedimientos tienen como diana ciertos núcleos de los de reposo. estamos ante un temblor de características postura- tálamo (núcleo ventral intermedio o VIM). nueva dosis. La diferencia es que aumentan la biodisponibilidad de la misma para su paso a través de los datos de afectación estructural se encuadran en ganglios basales y la barrera hematoencefálica. de la evolución es frecuente que se asocie con un cierto componen- te de temblor postural. pero con casos en que predomina el temblor. dosis (aparecen en el pico máximo de levodopa. que sólo está indicado en aque. miento de las técnicas quirúrgicas destinadas al control de la ante qué tipo de temblor nos encontramos.R: 1 Las discinesias pueden clasificarse en discinesias de beneficio de Hoy día la diana de elección en la cirugía funcional del Parkinson. discinesias bifásicas (movimientos rítmicos alternan- donde más mejora tanto la bradicinesia. El tratamiento quirúrgico puede plantearse en pacientes relativa.R: 2 lógica. casi todos los pacien- la otra. misma patología. no aparece coincidiendo con el fin del efecto de la levodopa) tener datos incompatibles con la enfermedad. Con la mente la combinación de distintos fármacos. El ideal es no produ. Los movimientos que aparecen en el pico plasmático de dosis. plasmáticos de levodopa.. enfermedad por cuerpos de niveles plasmáticos de levodopa. que mejora con alcohol: son En el momento actual puede decirse que la técnica quirúrgica de los datos características del temblor esencial. A las complicaciones derivadas del tratamiento con levodopa de El tratamiento busca. periférica (carbidopa-benseracida) sigue siendo el tratamiento de ciones en la función de los ganglios basales. de la acción de la levodopa) y distonía off (postura distónica que llos pacientes que cumplen criterios de inclusión. Se ha visto que es de tipo coreico). movilidad y discinesias. en las que aparecen cambios de personali- dad y cognitivos. La asociación de L-dopa El temblor característico de la enf. de forma directa o indirecta. ha motivado un resurgi. NEUROLOGÍA CTO Medicina 1ª Vuelta y si aparecen en fases iniciales hay que considerar otros diagnósti. o fin de dosis.. la falta de respuesta a L-Dopa habla en favor lares a los que se producen en la enfermedad por cuerpos de Lewy. enfermedad de Parkinson... permitiendo reducir la dosis y. tes de las extremidades inferiores que aparecen al inicio y/o al final blor. les. Es un tratamiento sintomático. respuesta cutáneo-plantar extensora) Las fluctuaciones motoras pueden ser simples. y generalmente son es la estimulación bilateral del núcleo subtalámico. hipertonía elástica. sin una clara relación temporal con la ingesta de dosis. de beneficio de dosis se basa en la distribución de la dosis de levodo- cir lesiones. que no responden mínimos movimientos. SPECT) que detecten altera. y que las pruebas complemen. La carbidopa y quedando en discusión si ambas entidades no forman parte de una benseracida. puede llegar a precisar Pregunta 75.. Puede presentar rueda dentada (de hecho. la diana de elección puede ser un claro componente familiar. ginar las situaciones en las que aparece el temblor: cos en el campo de la estereotaxia y neurofisiología que permiten • El temblor de reposo aparece en apoyo y sin hacer ninguna acti- localizar las dianas con precisión. primera línea y es especialmente útil en el tratamiento de la El estudio anatomopatológico muestra cambios degenerativos simi. Para la bradicinesia y la rigidez las que descartar fundamentalmente dos cosas: la ingesta de algún fár- dianas preferidas son el núcleo subtalámico y el globus pálido. elección es la estimulación bilateral del núcleo subtalámico. los efectos secundarios. ni demasiada edad (más de 70 años). que empeorarían los síntomas. En los Esta es una enfermedad de etiología no determinada. vidad. la rueda dentada Los parkinsonismos secundarios a causas metabólicas. y el tem. no presente degeneración cortical. res hasta fases avanzadas. wearing-off es el deterioro fin de dosis que aparece cuando caen los cos (parálisis supranuclear progresiva. maco que pueda causar temblor (el temblor farmacológico es más frecuentemente de tipo postural) y la existencia de focalidad neuro- Pregunta 74. no tener deterioro El manejo de las fluctuaciones motoras simples y de las discinesias cognitivo. Es la tes que inicialmente mejoran pierden su respuesta a la L-Dopa en 3- afectación cortical la que se relaciona con la demencia. en general. por lo que es de elección la pa a lo largo del día. El control de las fluctuaciones motoras erráticas requiere general- zó en los años 40. de Parkinson es el temblor de con agonistas dopaminérgicos permite un control parcial de las mis- reposo. de evitar altibajos en los niveles plasmáticos del fármaco. En ocasiones es difícil determinar desde el punto de vista teórico. por lo que en efecto no hay que in. que luego son irreversibles. bradicinesia y la rigidez. errático. más vasculares. La presencia de una rueda dentada podría hacer sospechar una cia. en la enfermedad de Parkinson se achaca al temblor de reposo. En el caso de la pregunta. y suele aparecer tras 3-5 años de trata- cluir antagonistas dopaminérgicos. por tan- tronco del encéfalo en la enfermedad de Parkinson. salvo La L-Dopa asociada a un inhibidor de la dopadecarboxilasa las pruebas de imagen funcionales (PET.. Debido a esto es especial- exploración se aprecian alteraciones de la sensibilidad profunda (la mente importante el control de la hipercolesterolemia. los ictus sufridos por pacientes diabéticos tienen mayor morbimortalidad. por ser el más frecuente y por el hecho de ser modi- ma 9. barbital). La diabetes aumenta el riesgo de padecer infarto principalmente con lo que los problemas cardiacos pueden estar asociados a pro. con herencia autosómica recesiva ligada al cromoso. en la cede a la inversa con el HDL-colesterol. minuye el flujo sanguíneo cerebral y favorece la hiperviscosidad san- En casi todos los casos hay afectación cardiaca. mente en anticonceptivos con alto contenido de estrógenos. debido a complicaciones cardiacas (más frecuente) o respiratorias. tela de juicio si la terapia hormonal sustitutiva en la menopausia plantar es extensora. de la vida. el riesgo se iguala con los miocardiopatía hipertrófica. degeneración de la vía corticoespinal. Sin embargo. ya que la recepción del estímulo superficial cutáneo El tabaquismo promueve la formación de placas de ateroma. ya que la degeneración de las neuronas sensitivas que for. de la aterosclerosis cerebral. con los niveles de colesterol total y LDL-colesterol. asociado a la microangiopatía y al aumento blemas restrictivos. en numerosas ocasiones se detecta di. se El riesgo relativo de padecer un ictus isquémico está directamente produce un desarrollo completo de la enfermedad. que es la ataxia. especial- man los cordones posteriores interrumpe la vía aferente de los refle. PATOLOGÍA VASCULAR CEREBRAL Pregunta 77. más importante. Factores de riesgo del ictus isquémico. A los cinco años de dejar de fumar. lo que conlleva a multiplicar por dos el riesgo de ictus isqué- suele limitar el pronóstico vital. latación..R: 1 La enfermedad de Friedreich es.. aunque posteriormente también se afec. que debería cursar con hiper.R: 1 El curso evolutivo de la Enfermedad de Friedreich es completo una vez que comienzan los síntomas. la ataxia hereditaria De entre los factores de riesgo del ictus isquémico (ver tabla). no fumadores. corticoespinal. tanto en pre- que depende de cordones posteriores) y arreflexia (a pesar de la vención primaria como en secundaria del ictus isquémico.. como corresponde a la degeneración de la vía incrementa o no el riesgo. relacionado con las cifras elevadas de tensión arterial. mientras que su- Además del cuadro clínico más llamativo. embólicos o por arritmias. El momento de comenzar el tratamiento lo da la limitación que tenga el paciente para su vida diaria. sistólica aislada también tienen más riesgo de sufrir un ictus isquémi- tan los cordones laterales. con trombos intracavitarios o anomalías de la conducción. tras un desarrollo normal. Los síntomas siempre comienzan en las dos primeras décadas ficable. llevando al fallecimiento de los Pregunta 80. la respuesta cutáneo. y una vez que aparecen. de forma discreta con el tratamiento con riluzol. de tipo aterotrombótico. el más frecuente. tabaquismo). por lo que los pacientes con hipertensión de los cordones posteriores. Está en jos de estiramiento muscular. Pregunta 78. que es la que más guínea. bre todo si se asocia con otros factores (HTA. Además. 12 24565789 . dis- sí se conserva. aunque típicamente se describe una mico. en efecto. es la hipertensión arterial. pacientes en la cuarta-quinta década. y se La supervivencia espontánea de la enfermedad puede alargarse utilizan los betabloqueantes o la primidona (un precursor del feno. tanto la diastó- El defecto anatomopatológico fundamental es la degeneración lica como la sistólica. con vías descendentes corticoespinal y co. aunque el El grado y la progresión de la aterosclerosis carotídea se relaciona cerebelo no está afectado desde el punto de vista anatómico. El tratamiento con anticonceptivos orales aumenta el riesgo. espinocerebelosa ascendente: esto da lugar a la ataxia. NEUROLOGÍA 1ª Vuelta CTO Medicina El tratamiento de esta patología es meramente sintomático.R: 5 Pregunta 80. so- reflexia). que 324565789 . En prácticamente todos los enfermos hay miocardiopatía. Es la causa más 1234546 12   2 12 56 12#. puede ser hipertrófica o dilatada. detectables en el EKG. y que con frecuencia asocian tras- tornos de la conducción. 6 1275856 2225 56 123. . 129 .6 12# 4546 frecuente de muerte. 6 125  2 6 12 5 2!. 12  56 125.2 12$5%454 Existen alteraciones esqueléticas adquiridas por la disfunción neu. . 5254556 2225. . básicamente cifoescoliosis y pies cavos. aunque la limita. 1255.6 222&56 rológica. 6 1255 . 2". . un infarto en este territorio provoca una afectación contralateral aunque en fases avanzadas también se afecta. podemos apostar por una etiología cardioembólica como ciones sensitivas. en la que el paciente también tiene signos Hay que recordar que los émbolos cardiacos tienen como desti- de afectación de primera motoneurona (con hiperreflexia y Babinski no más frecuente la arteria cerebral media. progresiva. Suele presentarse diabetes. diferenciándose (entre otros datos) en que los ojos miran hacia el miembros inferiores. nos servimos del reflejo de pérdida de motoneurona superior (reflejos vivos. Un tumor en la médula dorsal. En un paciente del que conocemos como antecedente un IAM neran). que consiste en aproximar rápidamente hacia los ojos un neo-plantar extensora) y en otras lesión de motoneurona inferior objeto por el campo visual que queremos valorar. puede aparecer por diversas lesiones. ra. cierre inconsciente de los párpados. Las pérdidas de fuerza asociadas a crisis epilépticas apare- Se describe un cuadro de debilidad progresiva de predominio cen tras una crisis motora (parálisis postcrítica o de Todd) o en las Comentarios TEST distal en las extremidades. en miembros inferiores) y segunda motoneurona en brazos.R: 4 vascular. y nos permite descartar un proceso lacunar. (hiperreflexia). Pregunta 81.R: 3 Una pérdida de fuerza de instauración brusca sugiere un origen Pregunta 79. pero prácticamente nunca hay altera- ción de las funciones superiores.. no se dege. sin amiotrofias. hacia el sano en las frontales. con fatigabilidad. la musculatura extraocular está preservada. Esto provoca un (atrofia muscular marcada. y de segunda motoneurona (amiotrofia y Cuando sospechamos un cuadro vascular. Por lo que respecta a las funciones sensitiva y moto- En las fases iniciales.R: 1 afectación sensitiva y de esfínteres por compresión de otros cordones La arteria cerebral anterior irriga la porción medial de los lóbulos medulares sirven para el diagnóstico. se trata de establecer fasciculaciones). fasciculaciones. La atrofia muscular espinal es selectiva de segun. sin otros datos de afectación sensitiva. reciente. que son generalizadas.. La desviación conjugada de dominio proximal. En este caso. que existe una pérdida de visión por esa área. y probablemente. respuesta cutá. que cursaría con paciente con clínica de pérdida de fuerza. subaguda. ni con altera. sólo afectaría a les. la presencia de hemianopsia asociada al defecto La esclerosis lateral amiotrófica es la enfermedad degenerativa motor automáticamente nos debe hacer pensar en una lesión hemis- de la motoneurona más frecuente. Esta distribución característica y la posible Pregunta 82. con datos de primera motoneurona crisis atónicas. con la mielopatía cervical.56 12'5(5 ción en la marcha la da la ataxia y no estas anomalías. 17 . La exploración del campo visual en un que en la exploración muestre en unas áreas corporales signos de paciente no colaborador no es sencilla. deducimos En ningún momento de la evolución presenta alteración del con. Hay que sospecharla ante un férica. ni con menoscabo de las funciones superiores. frontal y parietal. y característicamente presenta un curso fluctuante. no sólo a la vía motora. La hemicuerpo parético en el caso de las lesiones protuberanciales y miastenia gravis presenta reflejos miotáticos normales. con atro- fia y reflejos disminuidos. amenaza. si no se produce.. trol de esfínteres (esas neuronas. más probable si aparecieran en otras opciones infarto en la arteria El diagnóstico diferencial más frecuente que hay que hacer es cerebral media por otras causas. y un defecto motor aislado. sin que se sepa la razón. reflejos disminuidos). los ojos se presenta en lesiones frontales y en lesiones protuberancia- da motoneurona. La enfermedad de Kugelberg-Welander es de pre. esfinteriana el área de lesión dependiendo de los síntomas: un cuadro motor ni disautonómica. la prueba de imagen de elección será una TAC (la RM es en efecto para fosa posterior. mientras que la coroidea da una hemiparesia- Pueden aparecer los reflejos de liberación frontal (prensión. también con dificultad para la emisión. común de ambas arterias cerebrales anteriores. lo que sucede pasadas las primeras horas. arteria cerebral media o sistema vertebrobasilar. como en la base de la protuberancia.mano torpe. Comentarios TEST Una vez etiquetado el déficit en el lenguaje como disartria por paresia facial. pero no el hematoma descrito en la primera opción. de lesión del territorio completo de esta arteria. tiene una paresia facial izquierda central (o supranuclear). porque no es muy detallada y hay que considerar distintos datos: • Dificultad para sostener cosas con la mano: esto podría corres- ponder a una paresia. de inicio brusco. des.. Ante una hemianopsia de instauración brusca. Pregunta 85. será porque aún no se han produci- do cambios estructurales tras la isquemia. En los casos representan el 20% de toda la patología vascular. pero parece más apro- piado que para una disfonía se refieran alteraciones en la voz. Quizá también una disfonía podría describirse de esta manera. Dado que nuestra sospecha clínica es isquemia en lóbulo occipi- tal y tálamo. La ataxia-hemiparesia y la disartria-mano torpe. y ésta a su vez de etiología cerebelosa o sensitiva (lesión talámica o en cordones posteriores). A) Territorios vasculares cerebrales. originadas por una lesión en territorio de la arteria cerebral media. por lesión talámica. lo más frecuente es que se originen en la bifurcación de la basilar. predominio faciobraquial y probablemente se acompañará de des- cuentemente). por lo que la patología ateromatosa carotídea que pue- da estar asociada no justifica isquemia en este territorio. en este caso derecho. con apra. en el caso del déficit sensitivo será de sensibi. sea de causa cerebelosa o por algún déficit motor. sin desviación de la mirada. puro. o bien a una ataxia.R: 1 bral anterior. la lesión ha de ser occipital contralateral. sí puede ser una lesión lacunar. viación oculocefálica. Si la hemianopsia no se acompaña de otros déficits.3 y 2 cm de diámetro. de la coroidea anterior o de la cerebral posterior. mal. Tenemos un cuadro típico que asocia dificultad para hablar y dificultad con una mano: la disartria .. y aunque en ocasiones estas arterias pueden originarse a partir de las comunican- tes posteriores. como sucede en este caso. El hecho de encontrar déficit de fuerza facial apunta que la dificultad del habla será una disartria motivada por esta debilidad. Los infartos lacunares ble es que sea en territorio de la arteria cerebral posterior. palmomentoniano). bien con inhibición (abulia. La exploración describe una desviación de la comisura hacia la derecha. menos fre. una paresia facial izquierda. Los antecedentes en los que se describen distintos factores de riesgo cerebrovascular no se corresponden con un cuadro vascular. La hipertensión 18 . la lesión se encuentra en tálamo. aparece hemianop- lidades combinadas. co del encéfalo y del polígono de Willis. con lo cual. La primera es más frecuente. tanto de brazo posterior de cápsula xia de la marcha. puesto que el la malformación más frecuente del polígono de Willis es el origen déficit es izquierdo. Si es capaz de cerrar el ojo izquierdo es que no es una parálisis completa. no nuclear ni perifé- rica. presenta un déficit de interés. Pregunta 84. lacunar: un hematoma en cápsula interna derecha podría dar este Las alteraciones del campo visual de causa vascular pueden estar cuadro clínico. lo que cursa de forma asintomática debido a la suplencia de siempre que tengan la misma localización que esperamos del infarto riego por esta última. aunque también puede encontrarse en lesiones unilaterales. con torpeza por inco- ordinación. anterior. mutismo acinético) o con desinhibición (moria. podemos estar ante una lesión de arteria cerebral media o de tornos del comportamiento. Si la hemianopsia se acompaña de hemiparesia y hemihipoeste- Por afectación de áreas prefrontales se puede acompañar de tras. apatía. causada por Un infarto frontal bilateral también puede producirse dado que una infarto lacunar capsular. en brazo anterior de cápsula interna o protu- Pregunta 83. ción. la opción de la cerebral media está La incontinencia de esfínteres es más frecuente en las lesiones bila. y pensa- Pregunta 82. pero no de cere. suc. de pequeñas arterias perforantes procedentes del polígono de Willis. Para encontrar en una lesión vascular una hemianopsia homónima Las lesiones frontales bilaterales pueden cursar con un cuadro aislada. faltando en una de De las otras opciones. pero por mala articulación. cursará con anestesia ni con pérdida de sensibilidades aisladas. es decir. sia. la dificultad con la mano izquierda parece menos probable que se trate de una ataxia (ya que el habla no es por lesión cerebelosa y se describe una exploración sensitiva normal). y no sia y hemianestesia. • Dificultad para hablar: esto puede referirse a una afasia motora (con dificultad para emitir lenguaje) o a una disartria. hemihipoestesia completas. NEUROLOGÍA CTO Medicina 1ª Vuelta de predominio crural. esto es.R: 4 Atención a la descripción clínica del cuadro. quizá merezca la pena recordar que los ellas el segmento arterial entre la carótida interna y la comunicante hematomas pueden cursar clínicamente con un síndrom. pero no es este caso) que si aparece normal en un primer momento. En el caso de la pregunta. de Son pequeños infartos (lagunas) de un tamaño comprendido en- probable origen vascular. como corresponde a un déficit cortical. coroidea anterior. remos antes en una paresia.e lacunar. dado que el déficit visual es derecho. secundarios a aterosclerosis o lipohialinosis da el diagnóstico.R: 3 berancia anterior. prácticamente descartada al conservar el paciente un lenguaje nor- terales. El ictus motor puro clínico que semeja el de la hidrocefalia a presión normal. B) Vascularización del tron. y por lo tanto la lesión es izquierda: un paciente con lesión de cerebral media que le produce hemianopsia asociaría trastornos del lenguaje. incontinencia urinaria y deterioro de funciones interna. El mecanismo más frecuente de oclusión del flujo sanguíneo en el territorio de la arteria cerebral posterior es el embólico.. En el ictus sensitivo superiores. es el resto de síntomas asociados lo que nos tre 0. lo más proba. ceptos claros: • Arteriografía: con el grado de estenosis de certeza. ante el hallazgo de una malformación en Pregunta 86. la exploración complementaria de elección en el Puede asociarse a la Enfermedad de Alzheimer. • Patología grave concomitante (endocarditis. topografía lesional habitualmente localizada en el brazo posterior • Presentación clínica sugestiva de HSA incluso con TC normal. y su curso es presentar hematomas tamaño inferior a 2 cm atribuible a la oclusión de una arteria espontáneos de repetición en esas localizaciones. sin afec. La rotura de estas arteriolas de de los infartos lacunares supratentoriales de un tamaño mayor a 5 pared rígida por estos depósitos origina hematomas lobares. porcionada (similar afectación en cara. que intervención (endarterectomía carotídea). Los síndromes lacunares más frecuentes son: • Presencia de hemorragia en el TC craneal previo a la administra- 1) Ictus motor puro. y se pueden considerar una dilatación anómala de la origen aterotrombótico carotídeo izquierdo. anterior – comunicante anterior). cuyas paredes son las fía para determinar con la mayor exactitud posible el grado de que más sufren la presión arterial elevada. que esto puede suceder). lares. • Eco-doppler de TSA: si fuera normal o sugerente de estenosis no La hipertensión de forma crónica es la causa más frecuente de significativa.R: 4 65% de pacientes. no como tratamien. tación de otras áreas ni del nivel de conciencia o las funciones tratamiento actual con anticoagulantes. que se forman depósitos de material amiloide en las paredes arteriolares.v. llamar embolia artero-arterial o aterotromboembolia. • Existencia de diátesis hemorrágica (trombopenia < 100.R: 2 TC. Otros diagnósticos etiológicos incluyen la diabe. mento actual. Es el síndrome lacunar más frecuente con una ción del fármaco. generalmente pro.. Las malformaciones vasculares. y su localización más habitual es nosis significativa. El pronóstico. Probablemente dentro de poco se pueda determinar el grado con más exactitud en angiografías no convencionales (angio. y da. agresivo para reducirla. Pregunta 87. • Antecedente de lesión de SNC: hemorragia. No tiene tratamiento. • Convulsiones al inicio del ictus. Criterios de exclusión para la fibrinolisis i. La existencia de múltiples infartos lacunares puede conducir a un síndrome pseudobulbar. caben dos • El mecanismo de génesis de trombo a nivel vascular es por la opciones: por encima del 70% de estenosis hay indicación de agregación de plaquetas y fibrina sobre una estenosis previa. sino que el agregado se libere y circule pla en carótidas que están a la espera de cirugía. aplicándose en las tres primeras horas tras el inicio de los Suelen ocasionar un déficit neurológico de más de 24 horas de síntomas. parálisis hemicorporal facio-braquio-crural. en el mo. 19 .. resto del émbolo fragmentado para acabar de completar las carótidas (la localización más frecuente: bifurcación cerebral Comentarios TEST la definición. Cuando se localizan en la rodilla de la cápsula interna durante 48 horas previas y TTPA aumentado). hemorragia intraparenquimatosa. cirugía… peribucal y de la mano ipsilateral). con riesgo hemorrágico.. hepatopatía grave). La Resonancia Magnética es. tanto de recurrencias como de un primer sangrado. parte más posterior de la misma pueden dar lugar a una paresia • Glucosa sanguínea >400 o <50 mg/dl. pero la anticoagulación sólo se contem. sobre la placa de ateroma no obstruya el vaso en ese punto (aun- hay dudas sobre qué hacer.R: 4 base a la anamnesis y exploración neurológica. disposición. déficit sensitivo que afecta a un hemicuerpo incluida la cara. Lo importante es tener los con- arteriografía de troncos supraaórticos. cardiopatía. quizá. aneurismas arteriales. El episodio descrito corresponde a una amaurosis fugaz del ojo Los aneurismas saculares se forman en las bifurcaciones vascu- izquierdo. debe establecerse de forma individualizada. neoplasias 4) Disartria-mano torpe. se antiagregaría ante el cuadro clínico.000/mm3. Son más frecuentes en pacientes jóvenes. Hasta un 50% de los pacientes presentan accidentes tes criterios de exclusión: isquémicos transitorios previos de una duración aproximada de 30 minutos y una latencia entre el AIT y el infarto lacunar de 24-72 horas. de la cápsula interna. que se desprendiera de la carótida izquierda. hasta obstruir un vaso distalmente: a este fenómeno lo podemos to de mantenimiento a medio plazo. Resulta de un infarto lacunar a nivel del nú..R: 4 principio asintomática. crural aislada. estenosis. aneuris- siendo menos frecuente la distribución queiro-oral (afectación ma. una técnica pendiente de consolidarse. estaría indicada la realización de una angiogra. adonde podría ir un fragmento de trombo saculares. aun- El episodio de hormigueos en la mano derecha de días antes es que lo que en realidad lo es la debilidad en las paredes arteriales muy posible que corresponda a un ataque isquémico transitorio en esas bifurcaciones. pero no es útil para los infartos lacunares localizados en te cortico-subcortical de los hemisferios. La clínica consta de un • Hemorragia grave o peligrosa manifiesta o reciente. porque la fosa posterior. 3) Ataxia-hemiparesia. Consiste en la paresia o • Escala NIH > 25 puntos. • Historia de ictus previo y DM concomitante. misma. isquémico. debutar a cualquier edad. en las técnicas de neuroimagen. Se establece el diagnóstico ante una lesión de los depósitos ya no se movilizan. angio-RM) pero de momento la prueba gold standard es la ¡Cuidado con la nomenclatura!. cristales de dentro de éste son más frecuentes en los vasos que dependen de colesterol. Se denominan congénitos. brazo y pierna). cleo ventral posterolateral del tálamo. y el tratamiento debe endarterectomía abierta está dando buenos resultados. porción anterior de la protuberancia. aunque pueden de utilidad. neoplasia. Lo más frecuente es que el trombo formado En estenosis carotídeas altas pero menores del 70%. El diagnóstico se efectúa clínicamente en el 80% de los casos en Pregunta 88. observándose en un Pregunta 87. aunque también puede localizarse en la • Déficit neurológico escaso o síntomas que mejoran rápidamente. pancreatitis. y que puede dar problemas El proceso diagnóstico a seguir con una carótida sintomática es el como otra lesión ocupante de espacio a ese nivel. La TC demuestra la mayor parte depósitos de amiloide extravasculares. Si sugiere este. A este fenómeno le podemos denominar atero. sintomáticas. • Cirugía mayor o traumatismo importante en los tres últimos meses. y se confirma con La angiopatía amiloide es un proceso de personas mayores. que también presenta estudio de los infartos lacunares. 2) Ictus sensitivo puro. perforante. Está aprobada la administración de rTPA intravenoso en el ictus tes mellitus. lo que apunta directamente a patología carotídea izquier. en el lími- milímetros. En los infartos lacunares de mayor sangrados subaracnoideos o intraparenquimatosos (superficiales o pro- tamaño lesional el tratamiento antiagregante plaquetario puede ser fundos). etc. con menos estenosis generalmente está a su vez condicionada por el depósito de co- está indicado mantener la antiagregación. pueden ser la causa de estricto de la hipertensión arterial. NEUROLOGÍA 1ª Vuelta CTO Medicina arterial es el factor de riesgo más importante. tamaño y localización. Distintos son los aneurismas fusiformes: su localización típica es la Luego estamos ante un paciente con dos AIT que suponemos de arteria basilar. y cuando afectan a la • TA > 185/110 o necesidad de manejo i. que con los años dan lugar a las dilataciones hemisférico izquierdo. teniendo en cuenta su distinta na- En el tratamiento del infarto lacunar es fundamental el control turaleza. que no se rompe ni sangra. o tratamiento con heparina superiores. pero el La angioplastia carotídea como sustitución endovascular de la mecanismo de génesis es la aterotrombosis. pero aún es ir encaminado a evitar la aterotrombosis. • Ictus en los 3 últimos meses. puede cursar con paresia facial aislada.v. siempre que el paciente no cumpla alguno de los siguien- duración.(placa de ateroma) trombosis. en territorios vasculares de arterias perforantes. gastropatía ulcerativa reciente. La indicación de descrito: intervenir depende del tamaño. Pregunta 89. En el fondo de ojo se podrían haber apreciado. lesterol. más frecuentemente en arterias del polígono de Willis. pero no obstante pueden aparecer síntomas de otros sistemas (sensi- Comentarios TEST ción del sistema nervioso periférico suele tener un mecanismo inmu. aunque en los casos más graves también puede verse síndrome de Guillain-Barré. Existen ictus indeterminados. Es la forma más frecuente y general- mente presenta una mezcla de estos síntomas en distintos grados. y recuperan en gran medida los síntomas. y la prevención de las infecciones intercurrentes. velocidades de conducción y de las amplitudes. Pregunta 93. damos por correcta la PL. pie neuro- distancia. o por efectos secundarios del tratamiento. Pregunta 90. Pregunta 94. Es sugerente del diag- da a la diabetes mellitus. aunque también pueden aparecer en extremidades. vegetativo). SISTEMA NERVIOSO PERIFÉRICO. La fección previa. dado que la desmielinización con distribución “en guante y calcetín”. tivo.. diarreas y disautonomía cardiaca. El diagnóstico de síndrome de Guillain-Barré es eminentemente Pregunta 91. pero si no se encuentra no lo descarta. detec. puede hacerlo mediante su circulación y la oclusión distal. las velocidades de conducción son variables. con disminución de las valora la conducción motora en las zonas más proximales del axón. por otra parte. y que puede responder al ganciclovir. la primera sospecha clínica ha de ser conducción desproporcionadamente marcada respecto al resto (blo. alteraciones de ble es que se trate de un mecanismo aterotrombótico. no se suele plantear minio proximal. de miembros inferiores. la afecta. dad de predominio en miembros inferiores. arreflexia. Cursa con dolor en raíz nóstico. La prueba complementaria que establece el diagnóstico En fases avanzadas predominan las neuropatías carenciales y tóxi. que acude por debili- velocidades de conducción. De forma general. ya que hay que completar el estudio con pruebas insulínico. lo más fre. plasmaféresis. afectados. Miller-Fisher. Ante un varón. La diabetes mellitus es responsable de un gran número de patolo.. que en principio se conducirá con una velo. úlceras). con una distribución “en guante y calcetín”. encaminado a distinguir las afectaciones axonales de las desmielini- zantes. ría la forma más frecuente de afectación en el comienzo de la infec. que afectación mixta axonal y desmielinizante. en las fases iniciales de la infección. En la forma crónica sí son Los patrones de afectación del sistema nervioso periférico por efectivos los corticoides. en ocasiones el enlentecimiento se pro. Se produce una afectación directa debida al propio virus. El estudio demostrar en la biopsia. con episo- La afectación desmielinizante produce un enlentecimiento de las dio días previos gastrointestinal o respiratorio. son los hallazgos neurofisiológicos: el estudio del nervio periférico cas. ya que la típica disociación albúmino-citológica puede tardar La mayor parte de las polineuropatías de causa metabólica tienen incluso varios días tras el inicio de la clínica. por eso es correcta la opción 2. LCR se puede encontrar un aumento de proteínas. y puede ser reversible si se trata de forma agresiva. aumento de células inflamatorias (disociación albúmino-citológica) co dentro de las afectaciones del sistema nervioso periférico asocia. corticoides). (las raíces motoras). Tam. de predo- un proceso isquémico cerebral en un paciente. presentan dolor en miembros infe- En la práctica. y rápidamente progresiva. postulado que esta forma de neuropatía tendría un mecanismo cuente es que el ictus haya tenido un origen embólico cardiaco. detecta en el nervio periférico tras un estímulo que recorre el nervio En fases avanzadas puede aparecer también la polirradiculopatía hacia atrás y de nuevo hacia adelante. siempre en menor intensidad que la paresia. sobre las pruebas complementarias. junto con pérdida de fuerza y de masa muscular.R: 4 cardiorrespiratorias.. Generalmente tienen buen pronós- flujo lento (venas profundas. en la forma aguda no están indicados dada culpa de la DM son básicamente de tres tipos: su mala respuesta. pático por torceduras recurrentes) y las autonómicas (trastornos tróficos cutáneos. con trastornos del ritmo y posible muerte súbita. plementarias aparece alterada antes (entre el LCR o el estudio tándose distintos potenciales en lugar de uno solo tras un estímulo neurofisiológico). una polineuropatía desmielinizante inflamatoria aguda. debido a la lesión. NEUROLOGÍA CTO Medicina 1ª Vuelta • El mecanismo de formación de un coágulo es la activación de más frecuente es la parálisis incompleta del III par (con conserva- factores de coagulación generalmente en una zona vascular de ción de la actividad pupilar). aunque existe el denominado síndrome de (gammaglobulinas. lo más proba. el tratamiento intensivo de la diabetes El estudio neurofisiológico es básico para determinar las altera. o cuando no se encuentra justificación al proceso. y nos preguntan cidad normal si se conserva la mielina. • Mononeuropatías: son características las mononeuropatías cra. incluso aunque no tuvieran un mal control con anti- en ambos sentidos. • Neuropatía motora proximal (denominada amiotrofia diabética): generalmente en DM tipo 2. el primero en ser positivo. la respuesta correcta no sería la punción lumbar (dispersión). neurofisiológico es. en el que predomina la debilidad superior (craneofacial). Dan por hecho que reconocemos el cuadro. no suele mostrar hallazgos patológicos. Estos pacientes suelen responder al tratamiento a priori la causa. Tras varios días de evolución. y que está abolida en el que se asocia a la infección concomitante por CMV. Si nos preguntaran cuál de las siguientes pruebas com- desmielinizado. sin que haya La amiotrofia proximal en miembros inferiores es un cuadro típi. que recuerda a un será la ocular. Se ha Ante el hallazgo de patología cardíaca embolígena.R: 4 clínico. ausencia 20 .. desmielinización muy proximal en las raíces anteriores medulares tas. En el Guillain-Barré clásico. con alteración de las fibras finas (dolor. diabéticos orales. las fibras gruesas (alteraciones en la posición. vejiga neurógena. La desmielinización produce otro efecto aña. corazón) y la isquemia cerebral sólo tico y se recuperan sin secuelas. no consigue revertir sintomatología. Ante un cuadro clínico típico de síndrome de Guillain-Barré (in- neales. Sería la onda F.R: 1 Salvo escasas excepciones. (PL). la musculatura que menos se afectará ción: la polineuropatía desmilinizante aguda. debilidad de curso ascendente. y la más frecuente es la polineuropatía sensitiva simétrica distal. con lo que el cuadro es casi únicamente motor. con arreflexia generali- duce en puntos concretos en los que se detecta una dificultad en la zada. la típica dismi- dido: el potencial que conduce el nervio generalmente es único. que se puede síndrome de Guillain-Barré. que- en su génesis mecanismos aterotrombóticos con suelta de trombos a mazón). Pueden producirse por rales en las que aún no hay debilidad significativa. nución de las velocidades de conducción las encontramos en la mientras que si la conducción se produce por un segmento neurografía. parestesias. pero hay una onda (denominada onda F) que se causa multifactorial.R: 3 La afectación axonal produce una disminución de la amplitud Se trata de una pregunta modelo típica de síndrome de Guillain- del potencial nervioso. En este contexto esta. por infecciones oportunis. ante la presencia de una debilidad ascendente con aboli- Las manifestaciones neurológicas asociadas al VIH son múltiples y ción de los reflejos de estiramiento muscular. En el bién el VIH de forma directa puede causar una polirradiculitis. incluso en áreas corpo- no pocas corresponden al nervio periférico. con intubación y ventilación mecánica si fuera preciso en las formas gías en el sistema nervioso periférico. cuando el paciente tiene ambas cosas. que probablemente es de es muy proximal. electromiografía y electroneurografía no es lo mismo. El curso nológico. Barré. seguido de pérdida de fuerza y de masa muscular. joven. la sudoración. o con una rápida progresión. La queo de conducción). mediadas por microangiopatía más graves.. La plasmaféresis se reserva para los pacientes severamente y microinfartos en los nervios periféricos. en especial la tipo 2. y lo que más puede hacer es ciones estructurales básicas del nervio periférico: fundamentalmente prevención. a la hora de abordar el diagnóstico etiológico de riores. • Neuropatía autonómica: hipotensión ortostática. con lo que puede responder a terapias en ese sentido clásico es ascendente.R: 5 Lo más importante es mantener el soporte de las funciones Pregunta 92. afectada. porque está producido por la desmielinización. Estadísticamente • Polineuropatía difusa: existe alteración de las fibras sensitivas pe- la mayor parte de los procesos cerebrovasculares isquémicos tienen riféricas. y distinto al del resto de las polineuropatías asociadas a la DM ante la existencia de lesiones carotídeas estenosantes. La los reflejos están inicialmente conservados. Estos métodos son tanto más eficaces cuanto más preco.. la miocardiopatía y un bajo cociente intelectual. un solo intercurrentes. la muerte se produce alrededor de los 20 años por problemas respiratorios. cientes es característico: ptosis palpebral. automáticamente convierte la resección del timo en una medida Con la progresión de la enfermedad se afecta la musculatura fa. enfermedad en determinados pacientes. entre los 20 – 30 años. puesto que debe atrofiarse antes) la indicación quirúrgica la da ducción cardíaca (en el 70% de los pacientes). trastornos esfinterianos. con fatigabilidad músculo estriado. No hay tratamiento.R: 2 Cuando un receptor de acetilcolina es bloqueado por anticuerpos. Pregunta 96. con muta. Con la electroneurografía se estudia la conducción motora fases avanzadas no consiguen beneficios. familiar. Establecido el diagnóstico. con lo que el paciente puede tener restos tímicos en una persona adulta hay quien lo considera patoló- disfagia. El estudio genético (cromosoma 19) de latencia entre el proceso infeccioso y el debut del cuadro es diagnóstico. hace más evidente a medida que se repite la transmisión de potencia- les. puede producirse una La miastenia gravis se produce por un bloqueo de los receptores de remielinización si el paciente no fallece por complicaciones asocia. son eficaces. Debilidad muscular. en marcha y debilidad progresiva de la musculatura proximal de los un paciente miasténico no deben aparecer signos de otros sistemas miembros y flexora del cuello. evolución rápida. Este bloqueo no es completo. Una esclerosis Es especialmente importante el diagnóstico de mujeres portado. Los corticoides y otros inmunosupresores pueden dis. control de trastornos cardíacos. sensitivo. lo que. por tanto. y para levan. con trastorno en la relajación muscular. Este empeoramiento progresivo relacionado con la llegada repeti- Pregunta 95. si se con una distrofia miotónica de Steinert un fenotipo muy típico. lo que el funcionamiento de la sinapsis no vuelve a ser correcto por cuyo tratamiento de elección es la fenitoína. biopsia muscular y una buena historia una miastenia gravis. deberíamos recordar que se trata de El diagnóstico se establece por la clínica.. o sensitiva de un nervio periférico. estímulo) pero la estimulación repetitiva (como si se hiciera un ejerci- debido al reemplazamiento de musculo por grasa y tejido conectivo. empieza a faltar membrana en la sinapsis. en la membrana muscular. los impide el normal funcionamiento de la placa neuromuscular. res afectados. Hay importante aumento con fatigabilidad. pero Nos están describiendo una miastenia gravis. queda poco espacio para receptores.R: 4 da de estímulos tiene su correlato clínico en la fatigabilidad: la fuerza y La distrofia muscular de Duchenne es una distrofinopatía. con lo que tiene que ser en la edad adulta con fenómeno miotónico y debilidad facial y en reabsorbido y sustituido. 21 . desmielinizante con poco daño axonal. otorga a los pacientes de tener más superficie donde situar receptores de acetilcolina. es preferentemente ocular. ción en la miotonina proteinquinasa. estando los miembros inferiores más afectados: autonómico. de edad es incapaz de caminar. si aparecen otros datos en la exploración. La medida más útil es retirar de la circulación los anticuerpos circulantes: hay varios sistemas. Un estudio que la musculatura extraocular y a extremidades. Se trata de una forma generalizada. cataratas y calvicie.. De todas las que se presentan. Se trata de una afectación sólo de la fuerza. to (tras unas pocas contracciones voluntarias) y empeora en los días La sinapsis habitualmente tiene pliegues complicados. y nos. Cuando tenemos clínica es biopsia muscular. es la única se asocia a franca tumoración de origen tímico. Los reflejos miotáticos son normales. pero una repetición del distrofina es una proteina codificada por un gen situado en el brazo acto motor va empobreciendo la transmisión y aparece la debilidad. CPK seriadas. del cuello. NEUROLOGÍA 1ª Vuelta CTO Medicina de síntomas no motores) debería establecerse el diagnóstico con es. administración de gammaglobulina rando con el ejercicio.. neurológico. la afectación es predominantemente proximal. voz gangosa. siendo la más frecuente la hiperplasia. Es característica la pseudohipertrofia de pantorrillas. (peor por la noche). entreabierta (por debilidad facial).R: 3 en un 10 % de los casos es por problemas cardíacos. y la primera manifes. empeo- eficaces (inmunoadsorción. antebrazos y piernas. sonrisa invertida con boca tudio neurofisiológico y/o con estudio del LCR. El cuadro completo de esta enfermedad incluye la miopatía de Típicamente fluctúa (unos días mejor que otros). hipogonadismo. Es autosómica dominante. pero en un 10% de los casos nómeno de anticipación). que replantear el diagnóstico. En torno a los 10 años extremidades. permite asegurar el diagnóstico de portadora al 95%. que mejora con el calentamien. No hay datos de motoneurona. Pero se trata o bien un timo de apariencia normal (el hecho de que se detecten de una enfermedad sistémica. y los reflejos están abolidos. dado que la mayor parte son asintomáticas (la coincidencia de ser sólo motores. con el fin fríos. hendidura sináptica. la reposición no se hace con la misma velocidad. trastornos de con. Este recambio se hace por recaptación de manos de forma progresiva. En motora. Aparecen trastornos de la Dado que la afectación es exclusivamente de la placa motora. que no tendrían que Comentarios TEST ras. hay otros sistemas menos agresivos para el paciente e igualmente de predominio al final del día. ya que afecta a los dos cromosomas X anómalos es letal intraútero). ya que están intentando impedir la génesis de más lesión. El quita mucha membrana. que muestra en efecto atrofia de fibras tipo I. cerebelo. que se sueros antitoxina son útiles en el botulismo o en el tétanos. de la masticación. al tratarse de un proceso valora la típica respuesta decremental. Sólo un déficit motor severamente afectados que los superiores. sobre los receptores de acetilcolina. gico. en el tiempo está normal o levemente elevada. acetilcolina de la placa muscular. indicada. dado que la distrofia miotónica de Steinert segmentos de membrana en los que van incluidos los receptores. el estudio electromiográfico un proceso autoinmune en el que circulan autoanticuerpos contra (que muestra un patrón miopático con descargas miotónicas). pero en el caso del ciones intercurrentes. y los reflejos policlonal). que no suele ser incapacitante. y suelen fallecer Este mecanismo se objetiva con el estudio neurofisiológico: el pri- en la segunda década de la vida por infecciones pulmonares mer potencial de acción es normal (como lo son los reflejos. La electromiografía de fibra única La medida que más ha influido en la supervivencia de estos enfer. bamboleándose hacia La distribución anatómica de la afectación en la miastenia gravis los lados por insuficiencia muscular en cintura pélvica. corto del cromosoma X. mala respuesta al tratamiento médi- cardíacas y deterioro cognitivo progresivo. edades. cio) conlleva una pérdida de la intensidad de la transmisión (respuesta El tratamiento con prednisona ha demostrado alterar el curso de la decremental). Todo esto orienta hacia incluya EKG. cuyo caso el paciente no podría realizar ningún movimiento. sólo Pregunta 98. Comienzan a los 3-5 años. y la miotonía. y la la mielina del sistema nervioso periférico. nos mostrará la alteración del jitter. en das (dificultad respiratoria o en la deglución). puede ya se han producido los anticuerpos.. si el recambio es muy intenso porque haya muchos recepto- tación muscular es precisamente la miotonía de predominio en ma. alteraciones valvulares el cuadro clínico: debut precoz. intentando evitar infec- minuir e incluso frenar la síntesis de anticuerpos. los corticoides no controlarse con fenitoína o procainamida. porque a la aparición de los síntomas si es preciso. La calvicie frontal. co. con marcapasos Guillain-Barré no son eficaces. es lo que se denomina miotonía. La miastenia se asocia frecuentemente con alteraciones tímicas. miotáticos son normales. No hay tratamiento salvo el sintomático. El aspecto de estos pa. habrá de la CPK. Con la estimulación repetitiva se mos es el soporte vital. pero No hay toxina involucrada en el síndrome de Guillain-Barré. y del síndrome de Eaton-Lambert.R: 4 plasmaféresis (sustitución del suero por un compuesto artificial). tam. múltiple característicamente cursaría en brotes. cia del botulismo. fluctuando. dado que. con fatigabilidad. La CPK que se ha producido la síntesis de dichos anticuerpos. no de la sensibilidad. siendo el clásico la Pregunta 97. a diferen- No hay alteraciones sensitivas. En los otros casos en los que se detecta bien una hiperplasia cial. La miastenia es una alteración de la placa ces.. El paciente tendrá una marcha dificultosa. Los anticuerpos responsables de la miastenia gravis se sitúan en la bién puede realizarse el estudio genético de los cromosomas X. y si hay una generalización a tronco y tarse del suelo lo tiene que hacer apoyándose. que rara vez requiere tratamiento. y la debilidad facial. Suele debutar a estas cambios estructurales y no sólo por el bloqueo de los receptores. Hay que tener presente el diagnóstico de un paciente que debute queda inutilizado de forma definitiva. por es la distrofia más frecuente de la edad adulta. enfermedad por tripletes (fe. lo que que presenta miotonía. o tipo cluster. El verapamilo es el tratamiento profilác- tico de elección. se valoran los tiempos de latencia entre el estímulo y el potencial de acción muscular en una misma fibra muscular. El fármaco de elección para prevenir los ataques. y generalmente debuta entre los 25-45 años. Ninguna de estas pruebas complementarias es diagnóstica per se. pero puede apreciarse como los potenciales de acción disminuyen de tama- ño cuando se somete al músculo a una salva de estímulos. 22 . episodios diarios de cefalea unilateral.R: 5 La cefalea de Horton.. el tiempo de latencia permanece constante. Si el paciente es un varón. Se acompaña de lagrimeo. como causa más frecuente de síndrome miastenifor- me. Comentarios TEST CEFALEAS. quizá habrá que pensar primero en un síndrome de Eaton-Lambert. porque aparece en periodos. • Detección de anticuerpos antirreceptor de acetilcolina: no es específico al 100%. permaneciendo asintomático el resto del año. Aparecen varias veces al día durante un periodo de tiempo (cluster). rinorrea. y el hecho de encontrar alguna de ellas negativa no descarta el diag- nóstico de miastenia gravis. en determinadas épocas del año. con tumor conocido o síndro- me constitucional. periocular. mayor. congestión ocular. Pregunta 99. Pregunta 100. se detecta amplia variabilidad en esos intervalos de tiempo. es el verapamilo. • Estudio de fibra aislada: con un electrodo especial. Los ataques se ven precipitados por el alcohol y otros vasodilatadores. Tratamiento de las distintas formas de miastenia gravis. dado que pueden aparecer asociados a otros tipos de miastenia (inducida) o a otras patologías autoinmunes. y sumatriptán subcu- táneo. Se llama así. • Estimulación repetitiva: el estudio basal es normal. Suelen ser por la noche. El dolor periorbitario típico de la cefalea de Horton es unilateral. de entre 30-120 minutos. pero el tratamiento sintomático en la fase aguda del dolor. pero en la fase aguda.R: 3 El diagnóstico de miastenia gravis debe sospecharse ante una clí- nica sugerente (opción 1). apoya la sospecha de que la debilidad la cause un bloqueo en la placa motora. en una persona normal. predomina en varones 10:1. es inhalación de oxígeno a flujo elevado. NEUROLOGÍA CTO Medicina 1ª Vuelta Pregunta 99. Toda debilidad fluctuante es sugerente de miastenia gravis. Ante la sospecha clínica existen una serie de pruebas comple- mentarias que confirman el diagnóstico: • Test de edrofonio: una recuperación inmediata y transitoria de fuerza tras la administración de este anticolinesterásico de vida media corta i. mientras que si existe bloqueo de la placa motora. El sumatriptán es útil. Son episodios que duran entre media hora y dos horas aproximada- mente.v.. generalmente siempre a la misma hora.
Copyright © 2024 DOKUMEN.SITE Inc.